You are on page 1of 263

All rights reserved.

No part of Orthopaedic In-Training Examination may be reproduced, stored in a


retrieval system, or transmitted in any form or by any means (electronic, mechanical, photocopying,
recording, or otherwise) without the prior written permission of the publisher.

Published 2013
American Academy of Orthopaedic Surgeons
6300 North River Road
Rosemont, IL 60018

Copyright  2013 by the American Academy of Orthopaedic Surgeons

Requests for permission to reproduce any part of the work should be mailed to:
Attention: Examinations Department
American Academy of Orthopaedic Surgeons
6300 North River Road
Rosemont, IL 60018

Printed in the USA

© 2013 American Academy of Orthopaedic Surgeons 2013 Orthopaedic In-Training Examination


 2013 American Academy of Orthopaedic Surgeons 2013 Orthopaedic In-Training Examination

Dear Residents and Colleagues,

On behalf of the Central Evaluation Committee, I would like to thank you for participating in
this year’s examination.

Being the first such examination among all the medical specialties, this year marks the 51st
administration of the Orthopaedic In-training Examination™.

Every year, the OITE™ is created by a group of orthopaedic surgeons who are deeply committed
to education through a rigorous peer-review process. While many topics in orthopaedic surgery
lack conclusive evidence, many of our practices are based on sound medical principles and a
consensus of experts. Sometimes, there may appear several correct answers to a test item, but
each question has been vetted through this peer-review process to select the one best preferred
response.

Last year, we transitioned to a new web-based format. With this and each subsequent
administration, we strive to make the OITE™ better. Thank you for being part of this year’s
examination, and I hope that you find this to be an enriching educational experience.

© 2013 American Academy of Orthopaedic Surgeons 2013 Orthopaedic In-Training Examination


ACCREDITATION
The American Academy of Orthopaedic Surgeons is accredited by the Accreditation Council for
Continuing Medical Education to provide continuing medical education for physicians.

AMA PRA CREDIT


U.S. Physicians: The American Academy of Orthopaedic Surgeons designates this enduring
material for a maximum of 20 AMA PRA Category 1 Credits™. Physicians should claim only the
credit commensurate with the extent of their participation in the activity.

International Physicians: AMA PRA credit may only be claimed by, and awarded to, physicians,
defined by the AMA as individuals who have completed an allopathic (MD), osteopathic (DO) or
an equivalent medical degree from another country.

Allied Health Professionals: The AAOS is not accredited to offer credit for nurses and other allied
health professionals. To determine if activities offering AMA PRA Category 1 Credits™ are
acceptable for your licensing or certification needs please contact the relevant organizations
directly.

EDUCATIONAL OBJECTIVES

As a result of taking the 2013 Orthopaedic In-Training Examination, I am able to


 interpret patient data for making clinical decisions.
 determine the appropriate patient management procedures in orthopaedic surgery.
 integrate relevant evidence-based information regarding treatment procedures into my patient
practice.
 apply basic science principles in diagnosing and developing patient management plans for
musculoskeletal diseases.

Inquiries
Questions regarding completing this CME activity or other comments can be sent to
exams@aaos.org or write to: Attention Examinations Department, American Academy of
Orthopaedic Surgeons, 6300 North River Road, Rosemont, IL 60018.

Release Date: November 9, 2013


Expiration Date: June 1, 2014
No CME credit will be awarded for this activity after June 1, 2014.

 2013 American Academy of Orthopaedic Surgeons 2013 Orthopaedic In-Training Examination

© 2013 American Academy of Orthopaedic Surgeons 2013 Orthopaedic In-Training Examination


Produced by the American Academy of Orthopaedic Surgeons
Central Evaluation Committee

Montri D. Wongworawat, MD, Editor-in-chief


Professor, Program Director, Orthopaedic Surgery Residency, Loma Linda University, Department of
Orthopaedic Surgery, Loma Linda, California

Contributors
Animesh Agarwal, MD
Professor, Division of Orthopaedic Trauma, University of Texas Health Science Center at San Antonio,
Department of Orthopaedics, San Antonio, Texas

Rahul Banerjee, MD, FACS


Assistant Professor, Department of Orthopaedic Surgery, University of Texas Southwestern Medical
Center, Dallas, Texas

Albert Bolanos, MD

Samuel J. Chmell, MD
Professor, Department of Orthopaedic Surgery, University of Illinois School of Medicine,
Chicago,Illinois

Bruce E. Cohen, MD
Fellowship Director, Foot and Ankle Fellowship, OrthoCarolina Foot and Ankle Institute, Charlotte,
North Carolina

Sheila Ann Conway, MD


Program Director, Assistant Professor, Department of Orthopaedic Surgery, University of Miami Miller
School of Medicine, Miami, Florida

Gregory J. Della Rocca, MD, PhD, FACS


Associate Professor of Orthopaedic Surgery, Co-Director, Orthopaedic Trauma Service, University of
Missouri, Columbia, Missouri

David G. Dennison, MD
Assistant Professor of Orthopaedics, Mayo Clinical College of Medicine, Department of Orthopedic
Surgery, Rochester, Minnesota

James C. Dreese, MD
Assistant Professor, Orthopaedic Surgery, University of Maryland School of Medicine, Baltimore,
Maryland

Kenneth A. Egol, MD
Professor and Vice Chairman, Department of Orthopaedic Surgery, NYU Hospital for Joint Diseases,
New York, New York

Robert J. Esther, MD
Program Director, Department of Orthopaedics, University of North Carolina, Chapel Hill, North
Carolina

 2013 American Academy of Orthopaedic Surgeons 2013 Orthopaedic In-Training Examination

© 2013 American Academy of Orthopaedic Surgeons 2013 Orthopaedic In-Training Examination


Keith R. Gabriel, MD
Associate Professor of Surgery, Orthopaedic Residency Program Director, Southern Illinois University
School of Medicine, Springfield, Illinois

Charles L. Getz, MD
Associate Professor, Thomas Jefferson University Hospital, Department of Orthopaedic Surgery;
Division of Shoulder and Elbow Surgery, Rothman Institute, Philadelphia, Pennsylvania

Shawn R. Gilbert, MD
Associate Professor, The Children’s Hospital of Alabama, Division of Orthopaedic Surgery, University
of Alabama at Birmingham, Birmingham, Alabama

Jonathan N. Grauer, MD
Co-Director Yale New Haven Hospital Spine Center and Co-Director Yale Orthopaedic Spine Service,
Associate Professor, Department of Orthopaedics and Rehabilitation, Yale University School of
Medicine, New Haven, Connecticut

Konrad I. Gruson, MD
Assistant Professor, Department of Orthopaedic Surgery, Albert Einstein College of Medicine, Bronx,
New York

David J. Hak, MD, MBA


Professor, Denver Health/University of Colorado, Denver, Colorado

Brian R. Hallstrom, MD
Clinical Assistant Professor, Department of Orthopaedic Surgery, University of Michigan, Ann Arbor,
Michigan

Syed Ashfaq Hasan, MD


Associate Professor of Orthopaedic Surgery, University of Maryland School of Medicine, Department of
Orthopaedics, Baltimore, Maryland

Kathleen A. Hogan, MD
Orthopaedic Surgeon, NH Orthopaedic Center, Nashua, New Hampshire

Jeffrey Einer Johnson, MD


Professor, Washington University School of Medicine, Department of Orthopaedic Surgery; Chief, Foot
and Ankle Service, Barnes-Jewish Hospital and Washington University Medical Center; Director,
Orthopaedic Foot and Ankle Fellowship Program, Washington University School of Medicine, St. Louis,
Missouri

Amy L. Ladd, MD
Professor and Chief, Robert A Chase Hand and Upper Limb Center, Department of Orthopaedic
Surgery, Stanford University School of Medicine, Palo Alto, California

Joshua Langford, MD
Director, Limb Deformity Service; Attending Orthopaedic Traumatologist, Orlando Health Orthopaedic
Residency Program; Instructor of Orthopaedic Surgery, University of Central Florida College of
Medicine, Orlando, Florida; Affiliate Assistant Professor, University of South Florida College of
Medicine, Tampa, Florida

 2013 American Academy of Orthopaedic Surgeons 2013 Orthopaedic In-Training Examination

© 2013 American Academy of Orthopaedic Surgeons 2013 Orthopaedic In-Training Examination


C. Michael LeCroy, MD
Director of Orthopaedic Trauma Services, Mission Hospitals, Asheville, North Carolina

James J. McCarthy, MD
Director of Pediatric Orthopaedic Surgery, Alvin Crawford Chair in Pediatric Orthopaedics Professor
Department of Orthopaedic Surgery, University of Cincinnati College of Medicine, Cincinnati
Children’s Hospital Medical Center, Cincinnati, Ohio

Alfonso Mejia, MD, MPH


Program Director, Vice Head, Department of Orthopaedic Surgery, University of Illinois, Chicago,
Illinois

Bradley R. Merk, MD
Associate Professor of Orthopaedic Surgery, Director of Orthopaedic Trauma, Feinberg School of
Medicine, Northwestern University, Chicago, Illinois

Fred R. T. Nelson, MD
Clinical Associate Professor, Wayne State University, Detroit, Michigan

Timothy J. O’Mara, MD
Orthopaedic Surgeon, Private Practice, Reno, Nevada

Matthew E. Oetgen, MD
Assistant Professor, Department of Orthopaedic Surgery and Sports Medicine, Children’s National
Medical Center, Washington, DC

Norman Y. Otsuka, MD
Joseph E Milgram Professor of Orthopaedic Surgery, NYU Langone Hospital for Joint Diseases, New
York, New York

Selene G. Parekh, MD, MBA


Associate Professor, Department of Orthopaedic Surgery, Duke University Health System; Adjunct
Faculty, Fuqua Business School, Duke University, Durham, North Carolina

Alpesh A. Patel, MD, FACS


Associate Professor, Department of Orthopaedic Surgery, Northwestern University Feinberg School of
Medicine, Chicago, Illinois
Adjunct Associate Professor, Department of Orthopaedics, University of Utah School of Medicine, Salt
Lake City, Utah

Darryl Peterson, MD
Orthopaedic Hand Surgeon, Greater Baton Rouge Musculoskeletal Group, Baton Rouge, Louisiana

Richard W. Pope, MD
Colonel, US Army; Chief, Department of Orthopaedics and Rehabilitation, Dwight David Eisenhower
Army Medical Center; Assistant Professor of Clinical Surgery, Uniformed Services University of the
Health Sciences, Fort Gordon, Georgia

Sheeraz A. Qureshi, MD
Assistant Professor, Orthopaedic Surgery, Spinal Surgery, Mount Sinai Hospital, Mount Sinai School of
Medicine, New York, New York

 2013 American Academy of Orthopaedic Surgeons 2013 Orthopaedic In-Training Examination

© 2013 American Academy of Orthopaedic Surgeons 2013 Orthopaedic In-Training Examination


Kent A. Reinker, MD
Clinical Professor, University of Texas Health Sciences Center, San Antonio, Texas

Jason W. Roberts, MD
Orthopaedic Trauma Surgery, Bronson Orthopedics and Sports Medicine; Clinical Faculty, Western
Michigan University Orthopedic Surgery Residency, Kalamazoo, MI

J. R. Rudzki, MD
Clinical Assistant Professor, Department of Orthopaedic Surgery, The George Washington University
School of Medicine, Washington Orthopaedics and Sports Medicine, Washington, DC

Andrew J. Schoenfeld, MD
Assistant Professor, Director of Orthopaedic Research, Department of Orthopaedic Surgery, William
Beaumont Army Medical Center, Texas Tech University Health Sciences Center, El Paso, Texas

J. Milo Sewards, MD
Program Director, Orthopaedic Surgery Residency; Assistant Professor of Orthopaedic Surgery and
Sports Medicine; Team Physician, Temple University School of Medicine, Philadelphia, Pennsylvania

Michael J. Taunton, MD
Instructor in Orthopedics, College of Medicine, Mayo Clinic, Rochester, Minnesota

Anthony D. Watson, MD
Orthopaedic Surgeon, Private Practice, Pittsburgh, Pennsylvania

Peter G. Whang, MD, FACS


Associate Professor, Spine Service, Department of Orthopaedics and Rehabilitation, Yale University
School of Medicine, New Haven, Connecticut

Bruce H. Ziran, MD
Director of Orthopaedic Trauma, Atlanta Medical Center, Orthopaedic Residency, Atlanta, Georgia

Robert D. Zura, MD
Associate Professor, Orthopaedic Trauma Services, Department of Surgery, Division of Orthopaedic
Surgery, Duke University Medical Center, Durham, North Carolina

 2013 American Academy of Orthopaedic Surgeons 2013 Orthopaedic In-Training Examination

© 2013 American Academy of Orthopaedic Surgeons 2013 Orthopaedic In-Training Examination


ACKNOWLEDGEMENTS

OITE Software Development


Internet Testing Systems, LLC, Baltimore, Maryland

Medical Editing
Brenda Moss Feinberg, ELS, AAOS Examinations Consultant

Digital Media Specialist


John Cisco, AAOS Examinations Consultant
Katie Hovany, Medical Illustrator

American Academy of Orthopaedic Surgeons


Department of Electronic Media, Examinations, CME Course Operations, & Practice Management
Howard Mevis, Director
Laura Hruska, M. Ed., Manager, Examinations
Anna M. Scheer, Senior Coordinator, Examinations
Marcie L. Lampert, Senior Coordinator, Examinations
Irene Bogdal, Administrative Assistant, Examinations

 2013 American Academy of Orthopaedic Surgeons 2013 Orthopaedic In-Training Examination

© 2013 American Academy of Orthopaedic Surgeons 2013 Orthopaedic In-Training Examination


Disclosure Information for the 2013 Orthopaedic In-Training Examination (OITE)

Montri D Wongworawat, MD (Chair): 8 (Clinical Orthopaedics and Related Research); 9 (American


Society for Surgery of the Hand; AAOS; San Bernardino County Medical Society; Health Services
International; Alumni Association School of Medicine Loma Linda University; Neufeld Society
(Alumni Association)); Submitted on: 05/16/2013

Animesh Agarwal, MD: 3B (Smith & Nephew; KCI); 5 (Smith & Nephew); 8 (Journal of Orthopaedic
Trauma); 9 (AAOS; Orthopaedic Research Society; Orthopaedic Trauma Association; Clinical
Orthopaedic Society); Submitted on: 04/25/2013

Rahul Banerjee, MD, FACS: 9 (AAOS; AAOS); Submitted on: 06/03/2013

Samuel J Chmell, MD (Member): No Conflicts Reported; Submitted on: 04/01/2013

Bruce E Cohen, MD (Member): 1 (Arthrex, Inc; DJ Orthopaedics; Wright Medical Technology, Inc.); 3B
(Amniox; Arthrex, Inc; Wright Medical Technology, Inc.); 5 (Arthrex, Inc); 9 (American Orthopaedic
Foot and Ankle Society); Submitted on: 06/05/2013

Sheila Ann Conway, MD (Member): No Conflicts Reported; Submitted on: 06/04/2013

Gregory John Della Rocca, MD, PhD (Member): 2 (Synthes); 3B (LifeNet Health; Intellectual Ventures;
Synthes; Bioventus); 4 (Amedica; The Orthopaedic Implant Company; MergeNet); 5 (Wound Care
Technologies; Eli Lilly; Sonoma Orthopaedics); 8 (Journal of Bone and Joint Surgery - American;
Journal of Orthopaedic Trauma; Journal of the American Academy of Orthopaedic Surgeons); 9 (AAOS;
Orthopaedic Trauma Association; American College of Surgeons); Submitted on: 06/03/2013

David G Dennison, MD (Member): 2 (AO - instructor for AO hand fracture course); 5 (DePuy, A Johnson
& Johnson Company; DePuy, A Johnson & Johnson Company); 9 (American Society for Surgery of the
Hand; AAOS); Submitted on: 06/01/2013

Kenneth A Egol, MD: 1 (Exactech, Inc); 3B (Exactech, Inc); 5 (Synthes; OREF, OTA, OMEGA); 7
(SLACK Incorporated; Wolters Kluwer Health Lippincott Williams & Wilkins); 8 (Journal of
Orthopaedic Trauma) Submitted on: 04/11/2013

Robert J Esther, MD (Member): 6 (MTF Medical Board); Submitted on: 01/17/2013

Keith Robert Gabriel, MD (Member): 9 (American Orthopaedic Association; Pediatric Orthopaedic


Society of North America; American Academy of Pediatrics Section on Orthopaedics; State of Illinois
Division of Specialized Care for Children); Submitted on: 04/15/2013

Charles L Getz, MD (Member): 2 (Mitek); 3C (Zimmer); 5 (Zimmer); Submitted on: 05/31/2013

Shawn R Gilbert, MD (Member): 9 (AAOS, POSNA); Submitted on: 05/07/2013

Konrad Izumi Gruson, MD (Member): 4 (Amgen Co; Bristol-Myers Squibb; Eli Lilly; GlaxoSmithKline;
Johnson & Johnson; Medtronic; Merck; Pfizer; Procter & Gamble; Stryker; Zimmer); Submitted on:
01/21/2013

 2013 American Academy of Orthopaedic Surgeons 2013 Orthopaedic In-Training Examination

© 2013 American Academy of Orthopaedic Surgeons 2013 Orthopaedic In-Training Examination


David J Hak, MD (Member): 3B (RTI Biologics, Baxter, Invibio); 4 (Emerge); 5 (Synthes; Stryker); 8
(Orthopedics;Journal of Orthopaedic Trauma; European Journal of Orthopaedic Surgery); 9 (Orthopaedic
Trauma Association, International Society for Fracture Repair); Submitted on: 04/03/2013

Brian Richard Hallstrom, MD: 9 (AAOS; Orthopaedic Research and Education Foundation); Submitted
on: 04/04/2013

Syed Ashfaq Hasan, MD (Member): 9 (AAOS); Submitted on: 05/01/2013

Kathleen Anne Hogan, MD (Member): 2 (Cadence Pharmaceuticals; Ferring Pharmaceuticals); 3A


(Cadence Pharmaceuticals (sister is an employee)); Submitted on: 01/27/2013

Jeffrey Einer Johnson, MD (Member): 1 (OrthoHelix Surgical Designs, Inc. Division of Tornier;); 4
(OrthoHelix Surgical Designs, Inc. Division of Tornier; Midwest Therapy, LLC); 8 (Foot and Ankle
International; American Journal of Orthopedics; Techniques in Foot and Ankle Surgery); 9 (AAOS;
American Orthopaedic Foot and Ankle Society, MidAmerica Orthopaedic Association; International
Federation of Foot and Ankle Societies (IFFAS)); Submitted on: 05/30/2013

Amy L Ladd, MD: 1 (Extremity Medical; Orthohelix); 4 (Articulinx LLC; Extremity Medical LLC;
Illuminoss; OsteoSpring Medical, Inc.); 5 (National Institutes of Health (NIAMS & NICHD); OREF); 8
(Orthopedics Today); 9 (Ruth Jackson Orthopaedic Society); Submitted on: 05/31/2013

Joshua Langford, MD (Member): 3B (Stryker); 4 (Institute for Better Bone Health, LLC); Submitted on:
05/02/2013

Alfonso Mejia, MD: 3C (BloxR); 4 (BloxR); 5 (Synthes; Smith & Nephew; Arthrex, Inc; Acumed, LLC);
9 (AAOS, AALOS, IAOS, CMS, ISMS); Submitted on: 06/01/2013

Bradley Robert Merk, MD: 2 (Synthes; Stryker); 3B (Stryker); 5 (Synthes; Stryker); 8 (American Journal
of Orthopedics); 9 (Orthopaedic Trauma Association-Research Committee); Submitted on: 05/29/2013

Fred R T Nelson, MD (Member): 3C (Bioiberica); 7 (Elsevier for book royalty); Submitted on:
04/02/2013

Matthew Oetgen, MD (Member): 9 (AAOS; Pediatric Orthopaedic Society of North America; Scoliosis
Research Society); Submitted on: 04/05/2013

Timothy J O'Mara, MD: 2 (AO North America); 4 (Orthopaedic Implant Company); 8 (Journal of
Orthopaedic Trauma); 9 (AAOS); Submitted on: 02/05/2013

Norman Yoshinobu Otsuka, MD (Member): 8 (Journal of Children's Orthopaedics; Journal of


Orthopaedic Surgical Advances; Journal of Pediatric Orthopedics, Part B); 9 (American Academy of
Pediatrics; Bone and Joint Decade, U.S.A.; Pediatric Orthopaedic Society of North America; Pediatric
Orthopaedic Society of North America; Pediatric Orthopaedic Society of North America); Submitted on:
04/13/2013

Alpesh Ashwin Patel, MD (Member): 1 (Amedica); 3B (Amedica; Biomet; GE Healthcare; Stryker;


Zimmer); 4 (Amedica; Cytonics; Nocimed; Trinity Orthopaedics); 7 (Springer); 8 (Surgical Neurology
International; Wolters Kluwer Health - Lippincott Williams & Wilkins); 9 (AAOS; American College of
Surgeons; American Orthopaedic Association; AO Spine North America; Cervical Spine Research
Society; Indo-American Spine Alliance; Lumbar Spine Research Society; North American Spine

 2013 American Academy of Orthopaedic Surgeons 2013 Orthopaedic In-Training Examination

© 2013 American Academy of Orthopaedic Surgeons 2013 Orthopaedic In-Training Examination


Society); Submitted on: 04/15/2013

Darryl W Peterson, MD: 3C (Ultimate Athletic Equipment); 4 (Ultimate Athletic Equipment); 8 (J Robert
Gladden Orthopaedic Society Newsletter); 9 (J Robert Gladden Orthopaedic Society; Diversity Advisory
Board); Submitted on: 02/13/2013

Richard W Pope, MD (Member): 9 (AAOS Evaluations Committee (Hand/Wrist) starting in MAR11);


Submitted on: 01/17/2013

Sheeraz Qureshi, MD (Member): 1 (Zimmer); 2 (Medtronic; Stryker Zimmer); 3B (Stryker; Zimmer;


Medtronic; Orthofix, Inc.); 8 (Spine (reviewer); Spine Journal (reviewer)); 9 (AAOS; Cervical Spine
Research Society; Musculoskeletal Transplant Foundation; NASS); Submitted on: 04/29/2013

Kent A Reinker, MD (Member): 8 (Journal of Pediatric Orthopedics); 9 (AAOS); Submitted on:


01/17/2013

Jason W Roberts, MD: 9 (AAOS-Trauma Evaluation Committee member,); Submitted on: 01/28/2013

Jonas R Rudzki, MD (Member): 2 (Arthrex, Inc); 3B (Arthrex, Inc); 9 (AAOS Evaluation Committee &
BOC, Washington Orthopaedic Society); Submitted on: 02/01/2013

Andrew J Schoenfeld, MD: 8 (Saunders/Mosby-Elsevier- The Spine Journal); 9 (AAOS); Submitted on:
04/03/2013

Joseph Milo Sewards, MD (Member): 2 (Mitek); Submitted on: 05/21/2013

Michael J Taunton, MD (Member): 3B (DJ Orthopaedics); 5 (Stryker); Submitted on: 05/30/2013

Anthony D Watson, MD (Member): 8 (Foot and Ankle International); Submitted on: 04/02/2013

Peter G Whang, MD (Member): 2 (Medtronic; Stryker); 3B (Cerapedics; Medtronic; Paradigm Spine;


Relievant; Stryker; Trans1); 3C (DiFusion); 4 (DiFusion); 6 (Vertiflex); Submitted on: 09/10/2013

Bruce Ziran, MD (Member): 3B (Synthes); 4 (Powers Medical Group; Symbod; Tekartis); 8 (Clinical
Orthopaedics and Related Research; Journal of Bone and Joint Surgery - American; Journal of
Orthopaedics and Traumatology; journal of trauma; patient safety in surgery); 9 (AAOS; AAOS;
American Board of Orthopaedic Surgery, Inc.; American College of Surgeons; Orthopaedic Trauma
Association); Submitted on: 04/08/2013

Robert D Zura, MD (Member): 2 (Smith & Nephew, bioventus); 3B (Smith & Nephew, bioventus); 5
(Synthes); 6 (Synthes fellowship); Submitted on: 06/07/2013

American Academy of Orthopaedic Surgeons Staff

Howard Mevis: 4 (GE Healthcare; 3M; Gilead Sciences); 9 (Orthopaedic Learning Center); Submitted on:
06/06/2013

Laura Hruska (Staff Liaison): No Conflicts Reported; Submitted on: 02/18/2013

 2013 American Academy of Orthopaedic Surgeons 2013 Orthopaedic In-Training Examination

© 2013 American Academy of Orthopaedic Surgeons 2013 Orthopaedic In-Training Examination


* Disclosure Items Answered: (n) = Respondent answered 'No' to all items indicating no conflicts.
1= Royalties from a company or supplier; 2= Speakers bureau/paid presentations for a company or supplier; 3A=
Paid employee for a company or supplier; 3B= Paid consultant for a company or supplier; 3C= Unpaid consultant
for a company or supplier; 4= Stock or stock options in a company or supplier; 5= Research support from a
company or supplier as a PI; 6= Other financial or material support from a company or supplier; 7= Royalties,
financial or material support from publishers; 8= Medical/Orthopaedic publications editorial/governing board; 9=
Board member/committee appointments for a society.

 2013 American Academy of Orthopaedic Surgeons 2013 Orthopaedic In-Training Examination

© 2013 American Academy of Orthopaedic Surgeons 2013 Orthopaedic In-Training Examination


Disclaimer
The material presented in this self-assessment examination has been made available by the
American Academy of Orthopaedic Surgeons for educational purposes only. This material is not
intended to present the only, or necessarily the best, methods or procedures for the medical
situations discussed, but rather is intended to represent an approach, view, statement, or opinion of
the author(s) or producer(s), that may be helpful to others who face similar situations.

Some drugs or medical devices demonstrated in Academy courses or described in Academy print or
electronic publications have not been cleared by the Food and Drug Administration (FDA) or have
been cleared for specific uses only. The FDA has stated that it is the responsibility of the physician
to determine the FDA clearance status of each drug or device he or she wishes to use in clinical
practice.

Furthermore, any statements about commercial products are solely the opinion(s) of the author(s)
and do not represent an Academy endorsement or evaluation of these products. These statements
may not be used in advertising or for any commercial purpose.

 2013 American Academy of Orthopaedic Surgeons 2013 Orthopaedic In-Training Examination

© 2013 American Academy of Orthopaedic Surgeons 2013 Orthopaedic In-Training Examination


2013 Orthopaedic In-Training Examination

TABLE OF CONTENTS

EXAMINATION QUESTIONS Page


SECTION 1: Basic Science and Orthopaedic Diseases Questions .................................... 17
SECTION 2: Foot and Ankle ............................................................................................. 37
SECTION 3: Hand .............................................................................................................. 57
SECTION 4: Hip and Knee Reconstruction ....................................................................... 76
SECTION 5: System Based Practice .................................................................................. 98
SECTION 6: Oncology....................................................................................................... 106
SECTION 7: Pediatric Orthopaedics .................................................................................. 127
SECTION 8: Shoulder and Elbow ...................................................................................... 161
SECTION 9: Spine ............................................................................................................. 178
SECTION 10: Sports Medicine ............................................................................................ 199
SECTION 11: Musculoskeletal Trauma............................................................................... 212

APPENDICES
Non-Exclusive License for Use of AAOS Material .............................................................. 255
Score Key............................................................................................................................... 257
Content Domain Item Numbers ............................................................................................. 259
Worksheet (for personal use) ................................................................................................ 261

 2013 American Academy of Orthopaedic Surgeons 2013 Orthopaedic In-Training Examination

© 2013 American Academy of Orthopaedic Surgeons 2013 Orthopaedic In-Training Examination


SECTION 1: Basic Science and Orthopaedic Diseases Questions • 17

SECTION 1: Basic Science and Orthopaedic Diseases

Question 9
Which treatment technique for segmental nerve defects leads to the best motor recovery?

1. Silicon tube
2. Collagen tube
3. Nerve allograft
4. Nerve autograft
5. Direct repair with tension

PREFERRED RESPONSE: 4

RECOMMENDED READINGS
Giusti G, Willems WF, Kremer T, Friedrich PF, Bishop AT, Shin AY. Return of motor function after
segmental nerve loss in a rat model: comparison of autogenous nerve graft, collagen conduit, and
processed allograft (AxoGen). J Bone Joint Surg Am. 2012 Mar 7;94(5):410-7. PubMed PMID: 22398734.
Deal DN, Griffin JW, Hogan MV. Nerve conduits for nerve repair or reconstruction. J Am Acad Orthop
Surg. 2012 Feb;20(2):63-8. Review. PubMed PMID: 22302443.

© 2013 American Academy of Orthopaedic Surgeons 2013 Orthopaedic In-Training Examination


18 • American Academy of Orthopaedic Surgeons

Figure 22

Question 22
Figure 22 is the sagittal MRI scan of the spine of a 68-year-old woman with a history of chronic back pain
radiating into her right lower extremity. She reports paresthesias involving the right great toe and parts of
her shin. Nonsurgical treatment consisting of pain medications and epidural steroid injections has failed to
provide relief. Assessment of ambulation is most likely to reveal which pathologic gait pattern?

1. Quadriceps avoidance gait attributable to L5 compression and radiculopathy


2. Scissor gait attributable to spondylolisthesis at L4-5 and bilateral radiculopathy
3. Trendelenburg gait attributable to L4 compression and weakness of the gluteus medius
4. Trendelenburg gait attributable to L5 compression and weakness of the gluteus medius
5. Varus thrust gait during stance attributable to L5 compression and posterolateral corner
weakness

PREFERRED RESPONSE: 4

RECOMMENDED READINGS
Bono CM, Schoenfeld A, Garfin SR. Lumbar disc herniations. In: Herkowitz HN, Garfin SR, Eismont
FJ, Bell GR, Balderston R, eds. Rothman-Simeone The Spine. 6th ed. Philadelphia, PA: WB Saunders;
2011:887-914.
Keenan MA. Normal and pathologic gait. In: Lieberman JR, ed. AAOS Comprehensive Orthopaedic
Review. Vol 1. Rosemont, IL: American Academy of Orthopaedic Surgeons; 2009:163-170.

© 2013 American Academy of Orthopaedic Surgeons 2013 Orthopaedic In-Training Examination


SECTION 1: Basic Science and Orthopaedic Diseases Questions • 19

Question 24
Following laceration, peripheral nerves heal in which manner?

1. Direct repair across the gap by Schwann cells


2. Direct repair of the axon
3. Antegrade growth of the axon
4. Retrograde growth of the axon
5. Formation of collagen bridges by fibroblasts

PREFERRED RESPONSE: 3

RECOMMENDED READINGS
Dodds SD. Peripheral nervous system. In: Lieberman JR, ed. AAOS Comprehensive Orthopaedic Review.
Rosemont, IL: American Academy of Orthopaedic Surgeons; 2009:71-81.
Lee SK, Wolfe SW. Peripheral nerve injury and repair. J Am Acad Orthop Surg. 2000;8(4):243-52. PMID:
10951113.
Gupta R, Mozaffar T. Neuromuscular diseases. In: Einhorn TA, O’Keefe RJ, Buckwalter JA, eds.
Orthopaedic Basic Science: Foundations of Clinical Practice. 3rd ed. Rosemont, IL: American Academy
of Orthopaedic Surgeons; 2007:427-443.

Question 38
A 7-year-old boy with short stature, bowed legs, and back pain has 2 sisters whose appearances
are unremarkable. Laboratory studies show low serum phosphate; his serum calcium, and 25
hydroxycholecalciferol levels findings are within defined limits, but his 1,25-dihydroxycholecalciferol
levels are below normal. The boy has a male cousin with the same disorder. What type of rickets is the
most likely diagnosis?

1. Renal
2. Vitamin D-deficient
3. X-linked hypophosphatemic
4. Type I vitamin D-dependent
5. Type II vitamin D-dependent

PREFERRED RESPONSE: 3

RECOMMENDED READINGS
Carpenter TO, Imel EA, Holm IA, Jan de Beur SM, Insogna KL. A clinician's guide to X-linked
hypophosphatemia. J Bone Miner Res. 2011 Jul;26(7):1381-8. doi:10.1002/jbmr.340. Epub 2011 May 2.
Review. PubMed PMID: 21538511.
Dietz FR, Murray JC. Genetic basis of disorders with orthopaedic manifestations. In: Einhorn TA, O'Keefe
RJ, Buckwalter JA, eds. Orthopaedic Basic Science: Foundations of Clinical Practice. 3rd ed. Rosemont,
IL: American Academy of Orthopaedic Surgeons; 2007:25-47.

© 2013 American Academy of Orthopaedic Surgeons 2013 Orthopaedic In-Training Examination


20 • American Academy of Orthopaedic Surgeons

Question 52
A 5-year-old boy has superficial heterotopic bone around his chest and difficulty breathing. He has one
sibling who died of respiratory distress at age 13. The most likely genetic cause of this disorder is a
variant in

1. SOX9.
2. matrilin 3.
3. the FGF3 receptor.
4. the BMP type I receptor.
5. the PTH-PTHrP receptor.

PREFERRED RESPONSE: 4

RECOMMENDED READINGS
Carvalho DR, Navarro MM, Martins BJ, Coelho KE, Mello WD, Takata RI, Speck-Martins CE.
Mutational screening of ACVR1 gene in Brazilian fibrodysplasia ossificans progressiva patients. Clin
Genet. 2010 Feb;77(2):171-6. Epub 2009 Oct 1. PubMed PMID: 19796185.
Seo SW, Lee FY. Cellular and molecular biology, immunology, and genetics. In: Lieberman JR, ed. AAOS
Comprehensive Orthopaedic Review. Vol 1. Rosemont, IL: American Academy of Orthopaedic Surgeons;
2009:3-13.
Medvecky MJ. Skeletal muscle. In: Lieberman JR, ed. AAOS Comprehensive Orthopaedic Review. Vol 1.
Rosemont, IL: American Academy of Orthopaedic Surgeons; 2009:83-91.

Question 61
Postnatal skeletal linear growth occurs most rapidly during

1. puberty.
2. prepuberty.
3. preschool years.
4. first year of life.
5. early elementary school years.

PREFERRED RESPONSE: 4

RECOMMENDED READINGS
Dimeglio A. Growth in pediatric orthopaedics. In: Morrissey RT, Weinstein SL, eds. Lovell and Winter’s
Pediatric Orthopaedics. 6th ed. Philadelphia, PA: Lippincott Williams & Wilkins; 2006:35-65.
Herring JA. Limb length discrepancy. In: Herring JA, ed. Tachdjian’s Pediatric Orthopaedics. 4th ed.
Philadelphia, PA; WB Saunders; 2008:1191-1263.

© 2013 American Academy of Orthopaedic Surgeons 2013 Orthopaedic In-Training Examination


SECTION 1: Basic Science and Orthopaedic Diseases Questions • 21

Question 72
In the setting of skeletal muscle lacerations, which effect does transforming growth factor-beta have on
scar tissue formation?

1. Stimulates the proliferation of myocytes and enhances healing


2. Stimulates the proliferation of myofibroblasts and increases fibrosis
3. Enhances the differentiation of myoblasts and enhances healing
4. Decreases the proliferation of myofibroblasts and reduces fibrosis
5. Decreases the proliferation of myocytes and reduces healing

PREFERRED RESPONSE: 2

RECOMMENDED READINGS
Gulotta LV, Rodeo SA. Muscle, tendon, and ligament. In: Flynn JM, ed. Orthopaedic Knowledge Update
10. Rosemont, IL: American Academy of Orthopaedic Surgeons; 2011:37-48.
Pereira BP, Tan BL, Han HC, Zou Y, Aung KZ, Leong DT. Intramuscular nerve damage in lacerated
skeletal muscles may direct the inflammatory cytokine response during recovery. J Cell Biochem. 2012
Jul;113(7):2330-45. doi: 10.1002/jcb.24105. PubMed PMID: 22573552.

Question 84
What cell is responsible for the production of receptor activator of nuclear factor kappa beta ligand and
osteoprotegerin?

1. Osteocyte
2. Osteoclast
3. Osteoblast
4. Macrophage
5. Preosteoclast

PREFERRED RESPONSE: 3

RECOMMENDED READINGS
Boyle WJ, Simonet WS, Lacey DL. Osteoclast differentiation and activation. Nature. 2003 May
15;423(6937):337-42. Review. PubMed PMID: 12748652.
Miller JD, McCreadie BR, Alford AI, Hankenson KD, Goldstein SA. Form and function of bone. In:
Einhorn TA, O’Keefe RJ, Buckwalter JA, eds. Orthopaedic Basic Science: Foundations of Clinical
Practice. 3rd ed. Rosemont, IL: American Academy of Orthopaedic Surgeons; 2007:129-159.

© 2013 American Academy of Orthopaedic Surgeons 2013 Orthopaedic In-Training Examination


22 • American Academy of Orthopaedic Surgeons

Question 100
Which change, if any, will most likely occur within the articular cartilage of the knee as a result of
nonweight bearing for 2 months?

1. No change
2. Cartilage thinning
3. Decreased water content
4. Increased collagen synthesis
5. Increased proteoglycan concentration

PREFERRED RESPONSE: 2

RECOMMENDED READINGS
Hinterwimmer S, Krammer M, Krötz M, Glaser C, Baumgart R, Reiser M, Eckstein F. Cartilage atrophy
in the knees of patients after seven weeks of partial load bearing. Arthritis Rheum. 2004 Aug;50(8):2516-
20. PubMed PMID: 15334465.
Sun HB. Mechanical loading, cartilage degradation, and arthritis. Ann N Y Acad Sci. 2010 Nov;1211:37-
50. doi: 10.1111/j.1749-6632.2010.05808.x. Review. PubMed PMID: 21062294.
Millward-Sadler SJ, Wright MO, Davies LW, Nuki G, Salter DM. Mechanotransduction via integrins and
interleukin-4 results in altered aggrecan and matrix metalloproteinase 3 gene expression in normal, but not
osteoarthritic, human articular chondrocytes. Arthritis Rheum. 2000 Sep;43(9):2091-9. PubMed PMID:
11014361.

Question 110
At the time of heel strike in normal gait, what is the pattern of activation of the anterior tibialis?

1. Relaxation
2. Isotonic contraction until midstance
3. Concentric contraction followed by relaxation
4. Eccentric contraction throughout the stance phase
5. Eccentric contraction followed by concentric muscle activity

PREFERRED RESPONSE: 5

RECOMMENDED READINGS
Perry J. Normal gait. In: Smith DG, Michael JW, Bowker JH, eds. Atlas of Amputations and Limb
Deficiencies: Surgical, Prosthetic, and Rehabilitation Principles. 3rd ed. Rosemont, IL: American
Academy of Orthopaedic Surgeons; 2004:353-365.
Winter DA. Energy generation and absorption at the ankle and knee during fast, natural, and slow
cadences. Clin Orthop Relat Res. 1983 May;(175):147-54. PubMed PMID: 6839580.

© 2013 American Academy of Orthopaedic Surgeons 2013 Orthopaedic In-Training Examination


SECTION 1: Basic Science and Orthopaedic Diseases Questions • 23

Question 116
In performing a study comparing infection rates for patients undergoing elective orthopaedic surgery who
received irrigation with or without the addition of antibiotics in the solution, what is the most appropriate
statistical test to determine whether the proportion of patients developing infection is significant?

1. ANOVA
2. Paired t test
3. Unpaired t test
4. Chi-square test
5. Bonferroni correction

PREFERRED RESPONSE: 4

RECOMMENDED READINGS
Bhandari M. Evidence-based orthopaedics: issues in research design, analysis, and critical appraisal.
In: Einhorn TA, O’Keefe RJ, Buckwalter JA, eds. Orthopaedic Basic Science: Foundations of Clinical
Practice. 3rd ed. Rosemont, IL: American Academy of Orthopaedic Surgeons; 2007:87-101.
Kocher MS, Zurakowski D. Clinical epidemiology and biostatistics: a primer for orthopaedic surgeons. J
Bone Joint Surg Am. 2004 Mar;86-A(3):607-20. Review. PubMed PMID: 14996892.

Question 119
What component of bone is most responsible for tensile strength?

1. Hydroxyapatite
2. Type I collagen
3. Type II collagen
4. Type X collagen
5. Tricalcium phosphate

PREFERRED RESPONSE: 2

RECOMMENDED READINGS
Miller JD, McCreadie BR, Alford AI, Hankenson KD, Goldstein SA. Form and function of bone. In:
Einhorn TA, O’Keefe RJ, Buckwalter JA, eds. Orthopaedic Basic Science: Foundations of Clinical
Practice. 3rd ed. Rosemont, IL: American Academy of Orthopaedic Surgeons; 2007:129-159.
Clohisy JC, Lindskog D, Abu-AmerY. Bone and joint biology. In: Lieberman JR, ed. AAOS
Comprehensive Orthopaedic Review. Rosemont, IL: American Academy of Orthopaedic Surgeons;
2009:41-51.

© 2013 American Academy of Orthopaedic Surgeons 2013 Orthopaedic In-Training Examination


24 • American Academy of Orthopaedic Surgeons

Question 126
A randomized placebo-controlled study of 100 patients was performed to test the hypothesis that Drug
A would improve the rate of fracture healing. The study found that patients who received Drug A had a
shorter time to fracture healing (P < .05). A subsequent multicenter follow-up study with 1000 patients
showed that there was no real difference in fracture healing rates when comparing the drug to the placebo
(P = .25). Which kind of error occurred in the first study?

1. Type 1
2. Type 2
3. Design
4. Surgeon bias
5. Underpowered study

PREFERRED RESPONSE: 1

RECOMMENDED READINGS
Fosgate GT. Practical sample size calculations for surveillance and diagnostic investigations. J Vet Diagn
Invest. 2009 Jan;21(1):3-14. Review. PubMed PMID: 19139495.
Petrisor BA, Bhandari M. Orthopaedic research: health research methodology. In: Flynn JM, ed.
Orthopaedic Knowledge Update 10. Rosemont, IL: American Academy of Orthopaedic Surgeons;
2011:167-177.

Question 134
What is the etiology of fretting corrosion?

1. Oxidation
2. Differences in oxygen tension
3. Micromotion at contact sites under load
4. Impurities entering metal during manufacturing
5. Electrochemical potential between 2 metals in a conductive medium

PREFERRED RESPONSE: 3

RECOMMENDED READINGS
Singh K. Biomaterials. In: Lieberman JR, ed. AAOS Comprehensive Orthopaedic Review. Rosemont, IL:
American Academy of Orthopaedic Surgeons; 2009:21-28.
Srinivasan A, Jung E, Levine BR. Modularity of the femoral component in total hip arthroplasty. J Am
Acad Orthop Surg. 2012 Apr;20(4):214-22. Review. PubMed PMID: 22474091.

© 2013 American Academy of Orthopaedic Surgeons 2013 Orthopaedic In-Training Examination


SECTION 1: Basic Science and Orthopaedic Diseases Questions • 25

Question 142
An 85-year-old woman with osteoarthritis of the knee wants to discuss treatment options for her pain.
Which drug is most likely to result in serious renal impairment?

1. Tramadol
2. Ibuprofen
3. Oxycodone
4. Hydrocodone
5. Acetaminophen

PREFERRED RESPONSE: 2

RECOMMENDED READINGS
Horlocker TT, Kopp SL, Pagnano MW, Hebl JR. Analgesia for total hip and knee arthroplasty: a
multimodal pathway featuring peripheral nerve block. J Am Acad Orthop Surg. 2006 Mar;14(3):126-35.
PubMed PMID: 16520363.
Griffin MR, Yared A, Ray WA. Nonsteroidal antiinflammatory drugs and acute renal failure in elderly
persons. Am J Epidemiol. 2000 Mar 1;151(5):488-96. PubMed PMID: 10707917.

© 2013 American Academy of Orthopaedic Surgeons 2013 Orthopaedic In-Training Examination


26 • American Academy of Orthopaedic Surgeons

Figure 152

Question 152
Figure 152 is an arthroscopic view of a reconstructive procedure performed on a 28-year-old man who had
a medial femoral osteochondral lesion that measured 1.5 cm x 1.5 cm. The grafts were harvested from
the superolateral aspect of the trochlea. What is the most accurate description for the process of graft
incorporation?

1. The bone graft will be incorporated into the subchondral bone, and the overlying cartilage
remains viable.
2. The bone graft will be incorporated into the subchondral bone, but the overlying cartilage is
nonviable.
3. The bone graft will be incorporated while the overlying cartilage is used as a scaffold to
generate a new cartilage layer.
4. The bone graft will not be incorporated; healing will occur only within the cartilage layer.
5. The bone graft and cartilage will both be resorbed and replaced by a fibrocartilaginous matrix.

PREFERRED RESPONSE: 1

RECOMMENDED READINGS
O’Driscoll SW, Saris DBF. Articular cartilage repair. In: Einhorn TA, O’Keefe RJ, Buckwalter JA, eds.
Orthopaedic Basic Science: Foundations of Clinical Practice. 3rd ed. Rosemont, IL: American Academy
of Orthopaedic Surgeons; 2007:349-364.
Hangody L, Dobos J, Baló E, Pánics G, Hangody LR, Berkes I. Clinical experiences with autologous
osteochondral mosaicplasty in an athletic population: a 17-year prospective multicenter study. Am J Sports
Med. 2010 Jun;38(6):1125-33. Epub 2010 Apr 1. PubMed PMID: 20360608.

© 2013 American Academy of Orthopaedic Surgeons 2013 Orthopaedic In-Training Examination


SECTION 1: Basic Science and Orthopaedic Diseases Questions • 27

Question 154
Rivaroxaban is a novel anticoagulant approved for use in thromboprophylaxis after total hip and knee
arthroplasty. What is its mechanism of action?

1. Blocks factor V
2. Activates antithrombin III
3. Directly inhibits thrombin
4. Directly inhibits factor Xa
5. Inhibits vitamin K-dependent factors

PREFERRED RESPONSE: 4

RECOMMENDED READINGS
Eriksson BI, Borris LC, Friedman RJ, Haas S, Huisman MV, Kakkar AK, Bandel TJ, Beckmann H,
Muehlhofer E, Misselwitz F, Geerts W; RECORD1 Study Group. Rivaroxaban versus enoxaparin for
thromboprophylaxis after hip arthroplasty. N Engl J Med. 2008 Jun 26;358(26):2765-75. PubMed PMID:
18579811.
Della Valle CJ. Coagulation and thromboembolism. In: Lieberman JR, ed. AAOS Comprehensive
Orthopaedic Review. Vol 1. Rosemont, IL: American Academy of Orthopaedic Surgeons; 2009:149-156.

Question 165
Shoulder examination under anesthesia reveals symmetric forward elevation and external rotation in the
abducted position. However, external rotation with the arm by the side is decreased by 25 degrees when
compared to the contralateral shoulder. Which structure is contracted?

1. Biceps tendon
2. Anterior-inferior capsule
3. Anterior-superior capsule
4. Posterior-superior capsule
5. Acromioclavicular joint capsule

PREFERRED RESPONSE: 3

RECOMMENDED READINGS
Harryman DT 2nd, Sidles JA, Harris SL, Matsen FA 3rd. The role of the rotator interval capsule in passive
motion and stability of the shoulder. J Bone Joint Surg Am. 1992 Jan;74(1):53-66. PubMed PMID:
1734014.
Kim KC, Rhee KJ, Shin HD, Kim YM. Estimating the dimensions of the rotator interval with use of
magnetic resonance arthrography. J Bone Joint Surg Am. 2007 Nov;89(11):2450-5. PubMed PMID:
17974888.

© 2013 American Academy of Orthopaedic Surgeons 2013 Orthopaedic In-Training Examination


28 • American Academy of Orthopaedic Surgeons

Question 169
Rheumatoid factor as clinically measured detects antibodies to which antigen?

1. IgG
2. Scl-70
3. Histone
4. Nuclear antigens
5. Ribonucleoprotein

PREFERRED RESPONSE: 1

RECOMMENDED READINGS
Gardner GC, Kadel NJ. Ordering and interpreting rheumatologic laboratory tests. J Am Acad Orthop Surg.
2003 Jan-Feb;11(1):60-7. Review. PubMed PMID: 12699372.
Mortazavi JSM, Parvizi J. Arthritis. In: Flynn JM, ed. AAOS Orthopaedic Knowledge Update 10.
Rosemont, IL: American Academy of Orthopaedic Surgeons; 2011:213-224.

Figure 179

Question 179
Figure 179 is a recent pelvic radiograph of a 73-year-old woman who cannot ambulate. She developed
blindness as an infant and has a history of difficulty with healing of limb fractures. What defect is
associated with her disorder?

1. PEX gene
2. Fibrillin 1
3. Collagen I
4. Collagen oligomeric protein
5. Carbonic anhydrase

PREFERRED RESPONSE: 5

© 2013 American Academy of Orthopaedic Surgeons 2013 Orthopaedic In-Training Examination


SECTION 1: Basic Science and Orthopaedic Diseases Questions • 29

RECOMMENDED READINGS
Dietz FR, Mathews KD. Update on the genetic bases of disorders with orthopaedic manifestations. J Bone
Joint Surg Am. 1996 Oct;78(10):1583-98. Review. PubMed PMID: 8876589.
Einhorn TA. Metabolic bone disease. In: Einhorn TA, O’Keefe RJ, Buckwalter JA, eds. Orthopaedic
Basic Science: Foundations of Clinical Practice. 3rd ed. Rosemont, IL: American Academy of Orthopaedic
Surgeons; 2007:415-426.

Figure 185a Figure 185b Figure 185c

Question 185
Figures 185a through 185c are the biopsy specimen and radiographs of a 43-year-old man with a slowly
enlarging mass in his right proximal tibia. He has normal kidney function. Examination reveals a firm
mass with slight tenderness. Which laboratory value is most likely elevated?

1. Serum calcium
2. Serum phosphorous
3. Serum gamma globulin
4. Serum creatine phosphokinase
5. Prostate-specific antigen

PREFERRED RESPONSE: 1

RECOMMENDED READINGS
Singhal S, Johnson CA, Udelsman R. Primary hyperparathyroidism: what every orthopedic surgeon
should know. Orthopedics. 2001 Oct;24(10):1003-9; quiz 1010-1. Review. PubMed PMID: 11688768.
Mankin HJ, Mankin CJ. Metabolic bone disease: a review and update. Instr Course Lect. 2008;57:575-93.
Review. PubMed PMID: 18399611.

© 2013 American Academy of Orthopaedic Surgeons 2013 Orthopaedic In-Training Examination


30 • American Academy of Orthopaedic Surgeons

Question 192
Dessication of the nucleus pulposus results from loss or degradation of which molecule?

1. Matrilin
2. Proteoglycan
3. Type I collagen
4. Type II collagen
5. Type X collagen

PREFERRED RESPONSE: 2

RECOMMENDED READINGS
An HS, Masuda K, Cs-Szabo G, Zhang Y, Chee A, Andersson GB, Im HJ, Thonar EJ, Kwon YM. Biologic
repair and regeneration of the intervertebral disk. J Am Acad Orthop Surg. 2011 Jul;19(7):450-2. Review.
PubMed PMID: 21724924.
Kepler CK, Anderson DG, Tannoury C, Ponnappan RK. Intervertebral disk degeneration and emerging
biologic treatments. J Am Acad Orthop Surg. 2011 Sep;19(9):543-53. Review. PubMed PMID: 21885700.

Question 195
A study was undertaken to compare outcomes of 3 different treatment alternatives. The study was
initiated after treatments had been completed. What is the best term for this type of study?

1. Case-control
2. Prospective cohort
3. Retrospective cohort
4. Single blinded
5. Prospective randomized blinded and controlled

PREFERRED RESPONSE: 3

RECOMMENDED READINGS
Petrisor BA, Bhandari M. Orthopaedic research: health research methodology. In: Flynn JM, ed.
Orthopaedic Knowledge Update 10. Rosemont, IL: American Academy of Orthopaedic Surgeons;
2011:167-177.
Cox CL, Spindler KP. The design of clinical investigations: randomized, cohort and case studies. In:
O’Keefe RJ, Jacobs JJ, Chu CR, Einhorn TA, eds. Orthopaedic Basic Science: Foundations of Clinical
Practice. 4th ed. Rosemont, IL: American Academy of Orthopaedic Surgeons; 2013:437-443.

© 2013 American Academy of Orthopaedic Surgeons 2013 Orthopaedic In-Training Examination


SECTION 1: Basic Science and Orthopaedic Diseases Questions • 31

Question 199
What is the inheritance pattern of Duchenne muscular dystrophy?

1. Mitochondrial
2. X-linked recessive
3. X-linked dominant
4. Autosomal recessive
5. Autosomal dominant

PREFERRED RESPONSE: 2

RECOMMENDED READINGS
Zane MSM: Pediatric neuromuscular disorders. In: Lieberman JR, ed. AAOS Comprehensive Orthopaedic
Review. Rosemont, IL: American Academy of Orthopaedic Surgeons; 2009:293-309.
Dietz FR, Murray JC. Genetic basis of disorders with orthopaedic manifestations. In: Einhorn TA,
O’Keefe RJ, Buckwalter JA, eds. Orthopaedic Basic Science: Foundations of Clinical Practice. 3rd ed.
Rosemont, IL: American Academy of Orthopaedic Surgeons; 2007:25-47.

Question 203
What cell type is responsible for repair of cartilage defects with fibrocartilage following microfracture?

1. Synovial fibroblasts
2. Marrow mesenchymal cells
3. Deep-zone chondrocytes
4. Middle-zone chondrocytes
5. Superficial-zone chondrocytes

PREFERRED RESPONSE: 2

RECOMMENDED READINGS
Biswas D, Bible JE, Grauer JN. Articular cartilage. In: Lieberman JR, ed. AAOS Comprehensive
Orthopaedic Review. Rosemont, IL: American Academy of Orthopaedic Surgeons; 2009:53-63.
Shapiro F, Koide S, Glimcher M. Cell origin and differentiation in the repair of full-thickness defects of
articular cartilage. J Bone Joint Surg Am 1993 75A:532–553. PMID: 8478382.
Mithoefer K, Williams RJ III, Warren RF, Potter HG, Spock CR, Jones EC, Wickiewicz TL, Marx RG.
The microfracture technique for the treatment of articular cartilage lesions in the knee. A prospective
cohort study. J Bone Joint Surg Am. 2005 Sep;87(9):1911-20. PubMed PMID: 16140804.

© 2013 American Academy of Orthopaedic Surgeons 2013 Orthopaedic In-Training Examination


32 • American Academy of Orthopaedic Surgeons

Question 206
A weightlifter is engaged in a curling exercise using a free barbell. He is focused on resisting the weight
while his elbow extends. During this maneuver, he feels a snap in his right arm and drops the weight
suddenly. He is seen in the emergency department with a palpable prominence in his right biceps. During
which type of muscle contraction did this injury occur?

1. Isotonic eccentric
2. Isotonic concentric
3. Isometric eccentric
4. Isokinetic eccentric
5. Isokinetic concentric

PREFERRED RESPONSE: 1

RECOMMENDED READINGS
Lieber RL. Form and function of skeletal muscle. In: Einhorn TA, O’Keefe RJ, Buckwalter JA, eds.
Orthopaedic Basic Science: Foundations of Clinical Practice. 3rd ed. Rosemont, IL: American Academy
of Orthopaedic Surgeons; 2007:223-243.
Gulotta LV, Rodeo SA. Muscle, tendon, and ligament. In: Flynn JM, ed. Orthopaedic Knowledge Update
10. Rosemont, IL: American Academy of Orthopaedic Surgeons; 2011:37-48.

© 2013 American Academy of Orthopaedic Surgeons 2013 Orthopaedic In-Training Examination


SECTION 1: Basic Science and Orthopaedic Diseases Questions • 33

Video 218 (click to play)

Question 218
The pathologic gait shown in Video 218 is most likely attributable to

1. hamstring weakness.
2. quadriceps weakness.
3. adductor spasticity.
4. triceps surae spasticity.
5. tibialis anterior spasticity.

PREFERRED RESPONSE: 4

RECOMMENDED READINGS
Keenam MA. Normal and pathologic gait. In: Lieberman JR, ed. AAOS Comprehensive Orthopaedic
Review. Rosemont, IL: American Academy of Orthopaedic Surgeons; 2009:163-170.
Rodda JM, Graham HK, Carson L, Galea MP, Wolfe R. Sagittal gait patterns in spastic diplegia. J Bone
Joint Surg Br. 2004 Mar;86(2):251-8. PubMed PMID: 15046442.

© 2013 American Academy of Orthopaedic Surgeons 2013 Orthopaedic In-Training Examination


34 • American Academy of Orthopaedic Surgeons

Question 229
Parents bring in their 8-year-old son for an evaluation. They are concerned because the boy’s paternal
grandfather and uncle both died from osteosarcoma in their 40s. In addition, rhabdomyosarcoma was
recently diagnosed in the boy’s first cousin. The family is requesting genetic testing to ascertain their
son’s risk for developing a malignancy. Which gene is most likely responsible for the increased risk for
the malignancy?

1. EXT1
2. tX,18
3. t12,22
4. NF1
5. p53

PREFERRED RESPONSE: 5

RECOMMENDED READINGS
Jones KB. Musculoskeletal oncology. In: Flynn JM, ed. Orthopaedic Knowledge Update 10. Rosemont,
IL: American Academy of Orthopaedic Surgeons; 2011:193-212.
Seo SW, Lee FY. Cellular and molecular biology, immunology, and genetics. In: Lieberman JR, ed. AAOS
Comprehensive Orthopaedic Review. Vol 1. Rosemont, IL: American Academy of Orthopaedic Surgeons;
2009:3-13.

Question 244
Patients with factor V Leiden are at increased risk for deep vein thrombosis following surgery. What is
the effect of the mutation in factor V?

1. Blocks antithrombin III


2. Increases the activity of factor Xa
3. Prevents inhibition by activated protein C
4. Prevents the factor Xa conversion of prothrombin to thrombin
5. Makes it highly susceptible to inhibition by activated protein C

PREFERRED RESPONSE: 3

RECOMMENDED READINGS
Seligsohn U, Lubetsky A. Hereditary thrombophilia. In: Lichtman MA, Kipps TJ, Seligsohn U,
Kaushansky K, Prchal JT, eds. Williams Hematology. 8th ed. New York, NY: McGraw-Hill; 2010.chap 131.
Konkle B. Bleeding and thrombosis. In: Longo D, Fauci A, Kasper D, Hauser S, Jameson J, Loscalzo J,
eds. Harrison's Principles of Internal Medicine. 18th ed. New York, NY: McGraw-Hill; 2012:chap 58.

© 2013 American Academy of Orthopaedic Surgeons 2013 Orthopaedic In-Training Examination


SECTION 1: Basic Science and Orthopaedic Diseases Questions • 35

Question 254
Infliximab (antitumor necrosis factor-alpha antibody) is associated with which adverse condition?

1. Osteolysis
2. Osteoporosis
3. Hyperglycemia
4. Atypical fracture
5. Opportunistic infection

PREFERRED RESPONSE: 5

RECOMMENDED READINGS
Mortazavi JSM, Parvizi J. Arthritis. In: Flynn JM, ed. Orthopaedic Knowledge Update 10. Rosemont, IL:
American Academy of Orthopaedic Surgeons; 2011:213-224.
Bongartz T, Sutton AJ, Sweeting MJ, Buchan I, Matteson EL, Montori V. Anti-TNF antibody therapy
in rheumatoid arthritis and the risk of serious infections and malignancies: systematic review and meta-
analysis of rare harmful effects in randomized controlled trials. JAMA. 2006 May 17;295(19):2275-85.
Review. Erratum in: JAMA. 2006 Jun 7;295(21):2482. PubMed PMID: 16705109.

© 2013 American Academy of Orthopaedic Surgeons 2013 Orthopaedic In-Training Examination


36 • American Academy of Orthopaedic Surgeons

Figure 270a Figure 270b

Figure 270c

Question 270
Figures 270a through 270c are the MRI scans of a 65-year-old man who has pain in his right groin and
buttocks. He had a metal-on-metal hip replacement 3 years ago. What is the most likely cause of his
pain?

1. Infection
2. Lymphoma
3. Sarcomatous change
4. Large-particle wear debris disease
5. Pseudotumor hypersensitivity response

PREFERRED RESPONSE: 5

RECOMMENDED READINGS
Campbell P, Ebramzadeh E, Nelson S, Takamura K, De Smet K, Amstutz HC. Histological features of
pseudotumor-like tissues from metal-on-metal hips. Clin Orthop Relat Res. 2010 Sep;468(9):2321-7.
PubMed PMID: 20458645.
Kwon YM, Thomas P, Summer B, Pandit H, Taylor A, Beard D, Murray DW, Gill HS. Lymphocyte
proliferation responses in patients with pseudotumors following metal-on-metal hip resurfacing
arthroplasty. J Orthop Res. 2010 Apr;28(4):444-50. PubMed PMID: 19834954.

© 2013 American Academy of Orthopaedic Surgeons 2013 Orthopaedic In-Training Examination


SECTION 2: Foot and Ankle • 37

SECTION 2: Foot and Ankle

Question 8
A 49-year-old weekend athlete has a 4-week history of pain in his unilateral plantar heel that is most
severe for the first 20 steps upon arising in the morning. He has an area of maximal tenderness on the
plantar medial aspect of the heel pad at the origin of the plantar fascia. He has only improved 30% after
a 3-week course of physical therapy with toe intrinsic muscle strengthening and arch- and tendo-Achilles
stretching. What is the best next treatment step?

1. Release the plantar fascia.


2. Inject the plantar fascia with platelet-rich plasma.
3. Prescribe a night splint and continue physical therapy.
4. Administer extracorporeal shockwave therapy to the heel.
5. Perform a series of 3 steroid injections into the plantar fascia.

PREFERRED RESPONSE: 3

RECOMMENDED READINGS
Neufeld SK, Cerrato R. Plantar fasciitis: evaluation and treatment. J Am Acad Orthop Surg. 2008
Jun;16(6):338-46. Review. PubMed PMID: 18524985.
League AC. Plantar heel pain. In: Pinzur MS, ed. Orthopaedic Knowledge Update: Foot and Ankle 4.
Rosemont, IL: American Academy of Orthopaedic Surgeons; 2008:341-349.

© 2013 American Academy of Orthopaedic Surgeons 2013 Orthopaedic In-Training Examination


38 • American Academy of Orthopaedic Surgeons

Figure 16a Figure 16b

Question 16
Figures 16a and 16b are the radiographs of a 38-year-old carpenter with progressively worsening ankle
pain; 14 years ago, he was involved in an all-terrain vehicle collision. Anti-inflammatory medication,
corticosteroid injections, and bracing no longer effectively control his pain. The pain now interferes with
his work and family responsibilities. Examination reveals an antalgic limp, varus deformity, limited ankle
motion, limited eversion, and normal strength. Treatment should now consist of

1. ankle arthrodesis.
2. total ankle arthroplasty.
3. distal tibia osteotomy.
4. lateral ligament repair.
5. deltoid ligament release.

PREFERRED RESPONSE: 1

RECOMMENDED READINGS
Saltzman CL. Ankle arthritis. In: Coughlin MJ, Mann RA, Saltzman CL, eds. Surgery of the Foot and
Ankle. 8th ed. Philadelphia, PA: Mosby/Elsevier; 2007:923-983.
Easley ME, Adams SB Jr, Hembree WC, DeOrio JK. Results of total ankle arthroplasty. J Bone Joint Surg
Am. 2011 Aug 3;93(15):1455-68. Review. PubMed PMID: 21915552.

© 2013 American Academy of Orthopaedic Surgeons 2013 Orthopaedic In-Training Examination


SECTION 2: Foot and Ankle • 39

Question 30
A 48-year-old woman had total knee arthroplasty. She is unable to “lift her toes or ankle to her nose.”
After 2 months of physical therapy, she has a slapping gait. What is the best next treatment step?

1. Ankle fusion
2. Ankle-foot orthosis
3. Sural nerve graft
4. Medial heel post
5. Laminectomy of L4/5

PREFERRED RESPONSE: 2

RECOMMENDED READINGS
Yokoyama O, Sashika H, Hagiwara A, Yamamoto S, Yasui T. Kinematic effects on gait of a newly
designed ankle-foot orthosis with oil damper resistance: a case series of 2 patients with hemiplegia. Arch
Phys Med Rehabil. 2005 Jan;86(1):162-6. PubMed PMID: 15641008.
Sutlive TG, Mitchell SD, Maxfield SN, McLean CL, Neumann JC, Swiecki CR, Hall RC, Bare AC,
Flynn TW. Identification of individuals with patellofemoral pain whose symptoms improved after a
combined program of foot orthosis use and modified activity: a preliminary investigation. Phys Ther. 2004
Jan;84(1):49-61. PubMed PMID: 14992676.

© 2013 American Academy of Orthopaedic Surgeons 2013 Orthopaedic In-Training Examination


40 • American Academy of Orthopaedic Surgeons

Figure 46a Figure 46b Figure 46c

Question 46
Figures 46a through 46c are the CT scans of an 18-year-old who sustained an injury 3 weeks ago and
now has ankle pain. Examination reveals an ankle effusion and painful range of motion. Recommended
treatment should consist of

1. transtalar drilling.
2. fixation of the fragment.
3. osteochondral autograft.
4. weight bearing in a boot with early range of motion.
5. cast immobilization and nonweight-bearing activity for 6 weeks.

PREFERRED RESPONSE: 2

RECOMMENDED READINGS
Cuttica DJ, Smith WB, Hyer CF, Philbin TM, Berlet GC. Osteochondral lesions of the talus: predictors of
clinical outcome. Foot Ankle Int. 2011 Nov;32(11):1045-51. PubMed PMID: 22338953.
Choi WJ, Park KK, Kim BS, Lee JW. Osteochondral lesion of the talus: is there a critical defect size
for poor outcome? Am J Sports Med. 2009 Oct;37(10):1974-80. Epub 2009 Aug 4. PubMed PMID:
19654429.
Choi WJ, Kim BS, Lee JW. Osteochondral lesion of the talus: could age be an indication for arthroscopic
treatment? Am J Sports Med. 2012 Feb;40(2):419-24. Epub 2011 Oct 7. PubMed PMID: 21984689.

© 2013 American Academy of Orthopaedic Surgeons 2013 Orthopaedic In-Training Examination


SECTION 2: Foot and Ankle • 41

Question 54
A 47-year-old woman has a closed, displaced, Weber C bimalleolar ankle fracture. Past medical history
includes diabetes mellitus for 7 years controlled with diet and an oral hypoglycemic agent. Semmes-
Weinstein sensory testing reveals absence of sensation to the 5.07/10-gm monofilament on the plantar
aspect of both feet. The skin is intact with 2+ pedal pulses. Treatment should include

1. open reduction with limited internal fixation.


2. closed reduction and application of an external fixator.
3. closed reduction and total contact cast immobilization.
4. retrograde intramedullary rod fixation with ankle fusion.
5. internal fixation and an extended period of immobilization.

PREFERRED RESPONSE: 5

RECOMMENDED READINGS
Castro MD, Maurer JP: Fractures of the ankle. In: Pinzur MS, ed. Orthopaedic Knowledge Update: Foot
and Ankle 4. Rosemont, IL: American Academy of Orthopaedic Surgeons; 2008:59-73.
Jani MM, Ricci WM, Borrelli J Jr, Barrett SE, Johnson JE. A protocol for treatment of unstable ankle
fractures using transarticular fixation in patients with diabetes mellitus and loss of protective sensibility.
Foot Ankle Int. 2003 Nov;24(11):838-44. PubMed PMID: 14655888.
Johnson JE: Surgical treatment for neuropathic arthropathy of the foot and ankle. In: Trepman E, ed.
Instructional Course Lectures Foot and Ankle. Rosemont, IL: American Academy of Orthopaedic
Surgeons; 2009:51-59.
Costigan W, Thordarson DB, Debnath UK. Operative management of ankle fractures in patients with
diabetes mellitus. Foot Ankle Int. 2007 Jan;28(1):32-7. PubMed PMID: 17257535.

© 2013 American Academy of Orthopaedic Surgeons 2013 Orthopaedic In-Training Examination


42 • American Academy of Orthopaedic Surgeons

Figure 68a Figure 68b

Question 68
Figures 68a and 68b are the clinical photographs of a 55-year-old woman who had a right hindfoot fusion
3 years ago for a pes planovalgus deformity. Since the surgery, she has had lateral hindfoot pain and
places most of the weight-bearing load on the lateral border of her foot when walking. What is the most
likely cause of her symptoms?

1. Deltoid insufficiency
2. Excessive forefoot abduction
3. Residual heel valgus
4. Residual Achilles tendon contracture
5. Malposition of the transverse tarsal joint

PREFERRED RESPONSE: 5

RECOMMENDED READINGS
Johnson JE, Yu JR. Arthrodesis techniques in the management of stage II and III acquired adult flatfoot
deformity. Instr Course Lect. 2006;55:531-42. Review. PubMed PMID: 16958486.
Flemister AS Jr. Hindfoot osteoarthritis and fusion. In: Pinzur MS, ed. Orthopaedic Knowledge Update:
Foot and Ankle 4. Rosemont, IL: American Academy of Orthopaedic Surgeons; 2008:195-214.

© 2013 American Academy of Orthopaedic Surgeons 2013 Orthopaedic In-Training Examination


SECTION 2: Foot and Ankle • 43

Question 69
A 23-year-old hiker experienced a twisting injury to his right ankle 10 days ago. His dorsiflexion external
rotation test is negative and he is able to hop on his right ankle, but he has pain over the anterior talofibular
ligament. His peroneal strength is 4/5. What is the best next treatment step?

1. A modified Brostrom procedure


2. Rest, ice, compression, and elevation
3. Physical therapy with proprioceptive training
4. Casting of the right ankle in a neutral position
5. Surgical arthroscopy of the right ankle with anterolateral ankle debridement

PREFERRED RESPONSE: 3

RECOMMENDED READINGS
Janssen KW, van Mechelen W, Verhagen EA. Ankles back in randomized controlled trial (ABrCt): braces
versus neuromuscular exercises for the secondary prevention of ankle sprains. Design of a randomized
controlled trial. BMC Musculoskelet Disord. 2011 Sep 27;12:210. PubMed PMID: 21951559.
Lin C. Proprioceptive training reduces the risk of ankle sprain recurrence in athletes. Aust J Physiother.
2009;55(4):283. PubMed PMID: 19929772.

Question 81
Abnormal gait attributable to deformity after partial-foot amputation through the talonavicular and
calcaneocuboid joints is the result of the unbalanced pull of which structure?

1. Achilles tendon
2. Anterior tibialis
3. Posterior tibialis
4. Peroneus brevis
5. Flexor digitorum longus

PREFERRED RESPONSE: 1

RECOMMENDED READINGS
Ng VY, Berlet GC. Evolving techniques in foot and ankle amputation. J Am Acad Orthop Surg. 2010
Apr;18(4):223-35. Review. PubMed PMID: 20357231.
Early JS. Transmetatarsal and midfoot amputations. Clin Orthop Relat Res. 1999 Apr;(361):85-90.
PubMed PMID: 10212600.

© 2013 American Academy of Orthopaedic Surgeons 2013 Orthopaedic In-Training Examination


44 • American Academy of Orthopaedic Surgeons

Figure 92a Figure 92b

Question 92
Figures 92a and 92b are the current radiographs of a 47-year-old man with an 8-year history of diabetes
mellitus treated for 3 months with total contact casting for an erythematous, swollen, warm foot without
ulceration. He has had 2 episodes of plantar ulceration that have healed with repeat total contact casting.
What is the best next treatment step?

1. Midfoot osteotomy
2. Reapplication of a total contact cast
3. Achilles tendon lengthening procedure
4. Needle biopsy for culture and sensitivity
5. Application of an external bone growth stimulator

PREFERRED RESPONSE: 1

RECOMMENDED READINGS
Rogers LC, Frykberg RG, Armstrong DG, Boulton AJ, Edmonds M, Van GH, Hartemann A, Game F,
Jeffcoate W, Jirkovska A, Jude E, Morbach S, Morrison WB, Pinzur M, Pitocco D, Sanders L, Wukich
DK, Uccioli L. The Charcot foot in diabetes. Diabetes Care. 2011 Sep;34(9):2123-9. PubMed PMID:
21868781.
Pinzur MS. Current concepts review: Charcot arthropathy of the foot and ankle. Foot Ankle Int. 2007
Aug;28(8):952-9. Review. PubMed PMID: 17697664.
Pinzur MS, Sostak J. Surgical stabilization of nonplantigrade Charcot arthropathy of the midfoot. Am J
Orthop (Belle Mead NJ). 2007 Jul;36(7):361-5. PubMed PMID: 17694183.
Philbin TM. The diabetic foot. In: Pinzur MS, ed. Orthopaedic Knowledge Update: Foot and Ankle 4.
Rosemont, IL: American Academy of Orthopaedic Surgeons; 2008:273-290.

© 2013 American Academy of Orthopaedic Surgeons 2013 Orthopaedic In-Training Examination


SECTION 2: Foot and Ankle • 45

Figure 106a Figure 106b

Question 106
Figures 106a and 106b are the radiographs of a 36-year-old woman who had hallux valgus reconstruction
2 years ago. She has difficulty with shoe wear and pain with activity. Examination reveals moderate
pain with range of motion and medial tightness at the hallux metatarsophalangeal joint. What is the
recommended treatment?

1. Keller resection arthroplasty


2. Hallux metatarsophalangeal arthrodesis
3. Distal metatarsal osteotomy
4. Proximal metatarsal osteotomy with medial soft-tissue release
5. Metatarsophalangeal reconstruction with extensor hallucis brevis transfer

PREFERRED RESPONSE: 2

RECOMMENDED READINGS
Grimes JS, Coughlin MJ. First metatarsophalangeal joint arthrodesis as a treatment for failed hallux valgus
surgery. Foot Ankle Int. 2006 Nov;27(11):887-93. PubMed PMID: 17144948.
Skalley TC, Myerson MS. The operative treatment of acquired hallux varus. Clin Orthop Relat Res. 1994
Sep;(306):183-91. PubMed PMID: 8070193.

© 2013 American Academy of Orthopaedic Surgeons 2013 Orthopaedic In-Training Examination


46 • American Academy of Orthopaedic Surgeons

Question 130
A 62-year-old woman with diabetes mellitus and neuropathy has had a plantar foot ulcer at the second
metatarsal head for 2 months. Her dorsalis pedis pulse is palpable. Erythema surrounds the ulcer but
there is no drainage. The metatarsal head is palpable with a cotton-tipped applicator placed in the wound.
Treatment should consist of

1. dressing changes.
2. oral ciprofloxacin.
3. total-contact casting.
4. surgical debridement.
5. transmetatarsal amputation.

PREFERRED RESPONSE: 4

RECOMMENDED READINGS
Lavery LA, Armstrong DG, Peters EJ, Lipsky BA. Probe-to-bone test for diagnosing diabetic foot
osteomyelitis: reliable or relic? Diabetes Care. 2007 Feb;30(2):270-4. PubMed PMID: 17259493.
Grayson ML, Gibbons GW, Balogh K, Levin E, Karchmer AW. Probing to bone in infected pedal ulcers.
A clinical sign of underlying osteomyelitis in diabetic patients. JAMA. 1995 Mar 1;273(9):721-3. PubMed
PMID: 7853630.
Pinzur MS, Slovenkai MP, Trepman E, Shields NN; Diabetes Committee of American Orthopaedic
Foot and Ankle Society. Guidelines for diabetic foot care: recommendations endorsed by the Diabetes
Committee of the American Orthopaedic Foot and Ankle Society. Foot Ankle Int. 2005 Jan;26(1):113-9.
PubMed PMID: 15680122.

© 2013 American Academy of Orthopaedic Surgeons 2013 Orthopaedic In-Training Examination


SECTION 2: Foot and Ankle • 47

Figure 146a Figure 146b

Question 146
Figures 146a and 146b are the MRI scans of a 67-year-old tennis player who has had intermittent pain in
his posterior ankle for 15 years. He felt a “pop” while playing tennis 1 week ago and now has weakness
and increased pain. What is the most appropriate surgical option?

1. A mini-open primary Achilles repair


2. An allograft reconstruction of the Achilles tendon
3. A primary Achilles tendon repair reinforced with xenograft tissue
4. A peroneus longus tendon transfer with Achilles tendon repair
5. A flexor hallucis longus tendon transfer with Achilles tendon repair

PREFERRED RESPONSE: 5

RECOMMENDED READINGS
Arastu MH, Partridge R, Crocombe A, Solan M. Determination of optimal screw positioning in flexor
hallucis longus tendon transfer for chronic tendoachilles rupture. Foot Ankle Surg. 2011 Jun;17(2):74-8.
Epub 2010 Mar 7. PubMed PMID: 21549976.
Mahajan RH, Dalal RB. Flexor hallucis longus tendon transfer for reconstruction of chronically ruptured
Achilles tendons. J Orthop Surg (Hong Kong). 2009 Aug;17(2):194-8. PubMed PMID: 19721151.
Den Hartog BD. Flexor hallucis longus transfer for chronic Achilles tendonosis. Foot Ankle Int. 2003
Mar;24(3):233-7. PubMed PMID: 12793486.

© 2013 American Academy of Orthopaedic Surgeons 2013 Orthopaedic In-Training Examination


48 • American Academy of Orthopaedic Surgeons

Figure 157a Figure 157b Figure 157c

Question 157
Figures 157a through 157c are the radiographs and MRI scan of a 53-year-old woman who has had medial
ankle pain and swelling for 7 months. Examination reveals a pes planovalgus deformity. The hindfoot
and forefoot deformities are passively correctable. A single-limb toe raise on the affected leg reproduces
her pain. Surgical treatment should consist of

1. posterior tibialis tendon repair.


2. posterior tibialis tendon debridement.
3. flexor digitorum longus tendon transfer.
4. triple arthrodesis and flexor digitorum longus tendon transfer.
5. corrective osteotomies and flexor digitorum longus tendon transfer.

PREFERRED RESPONSE: 5

RECOMMENDED READINGS
Deland JT. Adult-acquired flatfoot deformity. J Am Acad Orthop Surg. 2008 Jul;16(7):399-406. Review.
PubMed PMID: 18611997.
Haddad SL, Myerson MS, Younger A, Anderson RB, Davis WH, Manoli A II. Symposium: Adult acquired
flatfoot deformity. Foot Ankle Int. 2011 Jan;32(1):95-111. PubMed PMID: 21288442.

© 2013 American Academy of Orthopaedic Surgeons 2013 Orthopaedic In-Training Examination


SECTION 2: Foot and Ankle • 49

Figure 175a Figure 175b

Question 175
Figures 175a and 175b are the radiographs of a 68-year-old man who has had hallux pain for several years.
Corticosteroid injections and orthotics no longer provide relief. He wants to continue his daily 2-mile
walk. What is the best next treatment step?

1. Arthrodesis
2. Cheilectomy
3. Implant arthroplasty
4. Proximal phalanx osteotomy
5. Keller resection arthroplasty

PREFERRED RESPONSE: 1

RECOMMENDED READINGS
Deland JT, Williams BR. Surgical management of hallux rigidus. J Am Acad Orthop Surg. 2012
Jun;20(6):347-58. Review. PubMed PMID: 22661564.
Yee G, Lau J. Current concepts review: hallux rigidus. Foot Ankle Int. 2008 Jun;29(6):637-46. Review.
PubMed PMID: 18549766.

© 2013 American Academy of Orthopaedic Surgeons 2013 Orthopaedic In-Training Examination


50 • American Academy of Orthopaedic Surgeons

Figure 191a Figure 191b

Question 191
Figures 191a and 191b are the radiographs of an 18-year-old man who had an ankle fracture requiring
open reduction and internal fixation 2 years ago. He has a progressive symptomatic ankle deformity.
Surgical intervention should consist of

1. ankle arthrodesis.
2. total ankle arthroplasty.
3. supramalleolar tibial osteotomy.
4. valgus-producing calcaneal osteotomy.
5. epiphyseodesis of the distal tibial physis.

PREFERRED RESPONSE: 3

RECOMMENDED READINGS
Becker AS, Myerson MS. The indications and technique of supramalleolar osteotomy. Foot Ankle Clin.
2009 Sep;14(3):549-61. Review. PubMed PMID: 19712889.
Pagenstert, GI; Hintermann B; Barg A; Leumann A;Valderrabano V. Realignment surgery as alternative
treatment of varus and valgus ankle osteoarthritis. Clin Orthop Relat Res. 462:156-68,2007. PubMed
PMID: 17563701.

© 2013 American Academy of Orthopaedic Surgeons 2013 Orthopaedic In-Training Examination


SECTION 2: Foot and Ankle • 51

Figure 217

Question 217
What is the foot orthosis/footwear prescription for management of the passively correctable deformity
seen in Figure 217?

1. Solid ankle cushion heel with lateral flare


2. 3/8” heel lift with firm heel counter
3. Lateral heel and lateral forefoot posting
4. Medial heel wedge with lateral forefoot posting
5. Metatarsal pad for global metatarsal head offloading

PREFERRED RESPONSE: 3

RECOMMENDED READINGS
Jeng CL, Logue J. Shoes and orthotics. In: Pinzur MS, ed. Orthopaedic Knowledge Update: Foot and
Ankle 4. Rosemont, IL: American Academy of Orthopaedic Surgeons; 2008:15-24.
Perry MD, Manoli A II. Foot and ankle reconstruction. In: Flynn JM, ed. Orthopaedic Knowledge Update
10. Rosemont, IL: American Academy of Orthopaedic Surgeons; 2011:523-535.
Alexander IJ. Pes cavus. In: Nunley JA, Pfeffer GB, Sanders RW, Tripmen E, eds. Advanced
Reconstruction Foot and Ankle. Rosemont, IL: American Academy of Orthopaedic Surgeons. American
Orthopaedic Foot and Ankle Society. 2004:495-502.

© 2013 American Academy of Orthopaedic Surgeons 2013 Orthopaedic In-Training Examination


52 • American Academy of Orthopaedic Surgeons

Question 231
A 67-year-old woman has a persistent foot drop 18 months after right total hip arthroplasty. Examination
reveals passive ankle joint dorsiflexion to 0 degrees. Muscle strength testing results are listed below.
Which treatment will provide the highest level of function?

R L
Anterior tibialis 0/5 5/5
EHL/EDLa 0/5 5/5
Peroneal 2/5 5/5
Posterior tibialis 3/5 5/5
FHL/FDL 3/5 5/5
b

Gastrocsoleus 4/5 5/5

a
Extensor hallucis longus/extensor digitorum longus
b
Flexor hallucis longus/flexor digitorum longus

1. Ankle foot orthosis


2. Gastrocsoleus lengthening
3. Jones extensor hallucis longus tendon transfer
4. Posterior tibialis tendon transfer to the dorsum of the foot
5. Flexor hallucis longus tendon transfer to the dorsum of the foot

PREFERRED RESPONSE: 1

RECOMMENDED READINGS
Prahinski JR, McHale KA, Temple HT, Jackson JP. Bridle transfer for paresis of the anterior and lateral
compartment musculature. Foot Ankle Int. 1996 Oct;17(10):615-9. PubMed PMID: 8908487.
Posterior tibial and anterior tibial tendon transfers for rebalancing the foot in neuromuscular disorders.
In: Hsu JD, ed. Techniques in Foot and Ankle Surgery. Philadelphia, PA: Lippincott Williams & Wilkins;
2009;8(4):172-177.

© 2013 American Academy of Orthopaedic Surgeons 2013 Orthopaedic In-Training Examination


SECTION 2: Foot and Ankle • 53

Figure 245a Figure 245b Figure 245c

Figure 245d Figure 245e

Question 245
Figures 245a through 245e are the radiographs and MRI scans of a 50-year-old ice hockey referee with
a 3-year history of progressive anterolateral ankle pain, a history of multiple ankle sprains, and a fibular
fracture he sustained 30 years ago. Examination reveals mild bilateral pes planovalgus feet with passive
ankle joint dorsiflexion range of motion of 10 degrees and plantar flexion of 45 degrees without pain. The
physician should recommend

1. ankle joint arthrodesis.


2. ankle ligament reconstruction.
3. supramalleolar osteotomy.
4. total ankle joint arthroplasty.
5. corticosteroid injection into the ankle.

PREFERRED RESPONSE: 1

© 2013 American Academy of Orthopaedic Surgeons 2013 Orthopaedic In-Training Examination


54 • American Academy of Orthopaedic Surgeons

RECOMMENDED READINGS
Abidi NA, Neufeld SK, Brage ME, Reese KA, Savharwal S, Paley D. Ankle arthritis. In: Pinzur MS, ed.
Orthopaedic Knowledge Update: Foot and Ankle 4. Rosemont, IL: American Academy of Orthopaedic
Surgeons; 2008:159-193.
Pagenstert GI, Hintermann B, Barg A, Leumann A, Valderrabano V. Realignment surgery as alternative
treatment of varus and valgus ankle osteoarthritis. Clin Orthop Relat Res. 2007 Sep;462:156-68. PubMed
PMID: 17563701.

Video 252 (click to play)

Question 252
Video 252 shows the intrasurgical examination of a 36-year-old woman 3 weeks after an acute ankle
sprain. Surgical treatment should include

1. oblique fibular osteotomy.


2. peroneal tendon tenodesis.
3. calcaneofibular ligament release.
4. lateral ankle ligament reconstruction.
5. superior peroneal retinacular reconstruction.

PREFERRED RESPONSE: 5

RECOMMENDED READINGS
Philbin TM, Landis GS, Smith B. Peroneal tendon injuries. J Am Acad Orthop Surg. 2009 May;17(5):306-
17. Review. PubMed PMID: 19411642.
Walther M, Morrison R, Mayer B. Retromalleolar groove impaction for the treatment of unstable peroneal
tendons. Am J Sports Med. 2009 Jan;37(1):191-4. Epub 2008 Oct 16. PubMed PMID: 18927253.

© 2013 American Academy of Orthopaedic Surgeons 2013 Orthopaedic In-Training Examination


SECTION 2: Foot and Ankle • 55

Figure 264a Figure 264b

Question 264
Figures 264a and 264b are the radiographs of a 55-year-old woman who has pain at the first
metatarsophalangeal joint that has not responded to a change in footwear and she desires surgery.
The deformity has been present for 40 years, and she has painless passive range of motion at the
metatarsophalangeal joint. The first metatarsophalangeal joint has 70 degrees of dorsiflexion and 20
degrees of plantarflexion and the deformity is partially passively correctable. Which procedure(s) should
be recommended to correct her deformity?

1. First metatarsophalangeal arthrodesis


2. Lapidus procedure with Akin osteotomy
3. Distal Chevron osteotomy with Akin phalanx osteotomy
4. Proximal first metatarsal osteotomy with modified McBride bunionectomy
5. Double metatarsal osteotomy with proximal and distal metatarsal correction

PREFERRED RESPONSE: 5

RECOMMENDED READINGS
Coughlin MJ, Mann RA: Hallux valgus. In: Coughlin MJ, Mann RA, Saltzman CL, eds. Surgery of the
Foot and Ankle. 8th ed. St Louis, MO: Mosby Elsevier; 2007:183-362.
Chou LB, Dieter AA, Aronson J, Kelly DM. Hallux valgus. In: Pinzur MS, ed. Orthopaedic Knowledge
Update: Foot and Ankle 4. Rosemont, IL: American Academy of Orthopaedic Surgeons; 2008:233-244.
Coughlin MJ, Carlson RE: Treatment of hallux valgus with an increased distal metatarsal articular angle:
evaluation of double and triple first ray osteotomies. Foot Ankle Int 1999 Dec;20(12): 762-70. PubMed
PMID: 10609703.
Johnson AE, Georgopoulos G, Erickson MA, Eilert R: Treatment of adolescent hallux valgus with the first
metatarsal double osteotomy: the denver experience. J Pediatr Orthop 2004; 24:358-62. PubMed PMID:
15205615.

© 2013 American Academy of Orthopaedic Surgeons 2013 Orthopaedic In-Training Examination


56 • American Academy of Orthopaedic Surgeons

Figure 273a Figure 273b Figure 273c

Question 273
Figures 273a through 273c are the clinical photograph, radiograph, and coronal MRI scan through the
forefoot of a 24-year-old otherwise healthy man with a 1-week history of increasing pain, fevers, and
swelling of his right foot. Examination reveals right groin tenderness and adenopathy. Laboratory studies
show a peripheral blood leukocyte count of 22.5X109 cells/L (reference range, 4.5 to 11X 109 cells/L),
C-reactive protein of 60.3 mg/L (reference range 0.08-3.1 mg/L), and erythrocyte sedimentation rate of 15
mm/h (reference range 0-20 mm/h). What is the best next treatment step?

1. Transmetatarsal amputation
2. Wound culture and intravenous antibiotics
3. Incision, debridement, and fifth-toe amputation
4. Incision, debridement, and partial fifth-ray resection
5. Incision, soft-tissue debridement, and open packing

PREFERRED RESPONSE: 5

RECOMMENDED READINGS
Farber DC, Henry S: Nondiabetic foot infections. In: Pinzur MS, ed. Orthopaedic Knowledge Update:
Foot and Ankle 4. Rosemont, IL: American Academy of Orthopaedic Surgeons; 2008:331-340.
Cayce KO IV, Galloway MT. Infection. In: Fischgrund JS, ed. Orthopaedic Knowledge Update 9.
Rosemont, IL: American Academy of Orthopaedic Surgeons; 2008:241-257.

© 2013 American Academy of Orthopaedic Surgeons 2013 Orthopaedic In-Training Examination


SECTION 3: Hand • 57

SECTION 3: Hand

Figure 1

Question 1
A 33-year-old woman has left index fingertip pain that is severely exacerbated by reaching movements.
An intense T2 signal under the nailbed is visible on the MRI scan seen in Figure 1. What is the best
treatment option?

1. Tumor excision
2. Sympathetic digital block
3. Oral calcium channel blockers
4. Tuft amputation with nail ablation
5. Activity modification and glove wear

PREFERRED RESPONSE: 1

RECOMMENDED READINGS
Nazerani S, Motamedi MH, Keramati MR. Diagnosis and management of glomus tumors of the hand.
Tech Hand Up Extrem Surg. 2010 Mar;14(1):8-13. PubMed PMID: 20216046.
Paliogiannis P, Trignano E, Trignano M. Surgical management of the glomus tumors of the fingers: a
single center experience. Ann Ital Chir. 2011 Nov-Dec;82(6):465-8. PubMed PMID: 22229235.
Wang P, Zhou Z. The treatment of finger glomus tumors by raising a full thickness nail bed flap or finger
pulp flap. J Hand Surg Eur Vol. 2011 Jun;36(5):420-2. PubMed PMID: 21685131.

© 2013 American Academy of Orthopaedic Surgeons 2013 Orthopaedic In-Training Examination


58 • American Academy of Orthopaedic Surgeons

Question 15
A 24-year-old man with weakness and atrophy of the thumb for 12 months has very slight numbness on
the radial side of his thumb that is constant and not progressing. He has no other hand or finger numbness.
His 2-point static sensory examination is unremarkable in all digits and there is marked atrophy of the
thenar muscles. His carpal tunnel provocative tests are negative. He has no symptoms on the opposite
hand and otherwise is in excellent health. Which next step will most likely reveal the diagnosis?

1. An MRI scan
2. Muscle biopsy
3. Carpal tunnel diagnostic injection
4. Electrodiagnostic testing
5. Carpal tunnel view radiograph

PREFERRED RESPONSE: 1

RECOMMENDED READINGS
Shimizu A, Ikeda M, Kobayashi Y, Saito I, Oka Y. Carpal tunnel syndrome caused by a ganglion in the
carpal tunnel with an atypical type of palsy: a case report. Hand Surg. 2011;16(3):339-41. PubMed PMID:
22072471.
Chen CH, Wu T, Sun JS, Lin WH, Chen CY. Unusual causes of carpal tunnel syndrome: space occupying
lesions. J Hand Surg Eur Vol. 2012 Jan;37(1):14-9. Epub 2011 Aug 8. PubMed PMID: 21825010.
Mauermann ML, Amrami KK, Kuntz NL, Spinner RJ, Dyck PJ, Bosch EP, Engelstad J, Felmlee JP, Dyck
PJ. Longitudinal study of intraneural perineurioma--a benign, focal hypertrophic neuropathy of youth.
Brain. 2009 Aug;132(Pt 8):2265-76. Epub 2009 Jun 30. PubMed PMID: 19567701.

© 2013 American Academy of Orthopaedic Surgeons 2013 Orthopaedic In-Training Examination


SECTION 3: Hand • 59

Figure 28a Figure 28b

Question 28
Figures 28a and 28b are the pre- and postreduction radiographs of a finger. The rehabilitation protocol
indicated is

1. buddy tape and active motion.


2. static splint in full extension.
3. static splint in 45 degrees of flexion at the proximal interphalangeal joint.
4. static splint in metacarpophalangeal joint flexion and proximal interphalangeal joint extension.
5. extension block splint in 90 degrees of flexion at the proximal interphalangeal joint.

PREFERRED RESPONSE: 1

RECOMMENDED READINGS
Merrel G, Slade JF. Dislocations and ligament injuries in the digits. In: Wolfe SW, Hotchkiss RN,
PedersonWC, Kozin SH, eds. Green’s Operative Hand Surgery. 6th ed. Philadelphia, PA: Elsevier
Churchill Livingstone; 2011:291-295.
Morgan WJ, Slowman LS. Acute hand and wrist injuries in athletes: evaluation and management. J Am
Acad Orthop Surg. 2001 Nov-Dec;9(6):389-400. Review. PubMed PMID: 11730330.

© 2013 American Academy of Orthopaedic Surgeons 2013 Orthopaedic In-Training Examination


60 • American Academy of Orthopaedic Surgeons

Question 37
A 27-year-old jackhammer operator has a 4-month history of hand coldness and severe ischemia that
spares his thumb and index finger. Systemic illnesses have been ruled out. Doppler workup reveals
aneurysmal changes, and digital subtraction arteriogram confirms the findings. Intervention should
consist of

1. excision and vein graft.


2. surgical thrombectomy.
3. systemic anticoagulation.
4. intravascular fibrinolysis.
5. interventional embolectomy.

PREFERRED RESPONSE: 1

RECOMMENDED READINGS
Yuen JC, Wright E, Johnson LA, Culp WC. Hypothenar hammer syndrome: an update with algorithms for
diagnosis and treatment. Ann Plast Surg. 2011 Oct;67(4):429-38. Review. PubMed PMID: 21372671.
Lifchez SD, Higgins JP. Long-term results of surgical treatment for hypothenar hammer syndrome. Plast
Reconstr Surg. 2009 Jul;124(1):210-6. PubMed PMID: 19568083.
Nitecki S, Anekstein Y, Karram T, Peer A, Bass A. Hypothenar hammer syndrome: apropos of six cases
and review of the literature. Vascular. 2008 Sep-Oct;16(5):279-82. PubMed PMID: 19238870.

Question 48
A mechanic sustained a high-pressure injection of cleaning solvent into the tip of his index finger 2 hours
ago. The finger has good capillary refill and his 2-point discrimination is 7 mm. Initial treatment should
include

1. a corticosteroid injection.
2. elevation and observation.
3. elective surgical treatment within 7 days.
4. oral clindamycin for 10 days.
5. emergent surgical debridement.

PREFERRED RESPONSE: 5

RECOMMENDED READINGS
Tan V, Katolik LI. Hand and wrist trauma. In; Flynn JM, ed. Orthopaedic Knowledge Update 10.
Rosemont, IL: American Academy of Orthopaedic Surgeons; 2011:351-362.
Stevanovic MV, Sharpe F. Acute infections. In: Wolfe SW, Hotchkiss RN, Pederson WC, Kozin SH, eds.
Green’s Operative Hand Surgery. 6th ed. Philadelphia, PA: Elsevier Churchill Livingstone; 2011:81.

© 2013 American Academy of Orthopaedic Surgeons 2013 Orthopaedic In-Training Examination


SECTION 3: Hand • 61

Question 58
An otherwise healthy 42-year-old woman is scheduled for carpal tunnel release. The physician should
adhere to routine sterility protocols

1. without local or systemic antibiotics.


2. and irrigate with cefazolin solution.
3. and irrigate with bacitracin solution.
4. and administer cefazolin within 1 hour before incision.
5. and administer cefazolin within 1 hour before incision and continue dosing up to 23 hours
after surgery.

PREFERRED RESPONSE: 1

RECOMMENDED READINGS
Tosti R, Fowler J, Dwyer J, Maltenfort M, Thoder JJ, Ilyas AM. Is antibiotic prophylaxis necessary
in elective soft tissue hand surgery? Orthopedics. 2012 Jun 1;35(6):e829-33. doi: 10.3928/01477447-
20120525-20. PubMed PMID: 22691653.
Bykowski MR, Sivak WN, Cray J, Buterbaugh G, Imbriglia JE, Lee WP. Assessing the impact of
antibiotic prophylaxis in outpatient elective hand surgery: a single-center, retrospective review of 8,850
cases. J Hand Surg Am. 2011 Nov;36(11):1741-7. Epub 2011 Oct 5. PubMed PMID: 21975095.
Harness NG, Inacio MC, Pfeil FF, Paxton LW. Rate of infection after carpal tunnel release surgery and
effect of antibiotic prophylaxis. J Hand Surg Am. 2010 Feb;35(2):189-96. PubMed PMID: 20141890.
Rizvi M, Bille B, Holtom P, Schnall SB. The role of prophylactic antibiotics in elective hand surgery. J
Hand Surg Am. 2008 Mar;33(3):413-20. Review. PubMed PMID: 18343301.

© 2013 American Academy of Orthopaedic Surgeons 2013 Orthopaedic In-Training Examination


62 • American Academy of Orthopaedic Surgeons

Figure 65a Figure 65b

Question 65
Figures 65a and 65b are the magnetic resonance arthrogram and wrist arthroscopic photograph of a
25-year-old man who has wrist pain during extension and ulnar rotation. Treatment should consist of

1. synovectomy.
2. ulnar shortening osteotomy.
3. diagnostic arthroscopy only.
4. triangular fibrocartilage complex tear debridement.
5. triangular fibrocartilage complex repair dorsal ligament.

PREFERRED RESPONSE: 4

RECOMMENDED READINGS
Faber KJ, Iordache S, Grewal R. Magnetic resonance imaging for ulnar wrist pain. J Hand Surg Am. 2010
Feb;35(2):303-7. Review. PubMed PMID: 20141901.
Chen NC, Osterman AL, Culp RW, Chung KC. Wrist arthroscopy. In: Chung KC, Murray PM, eds. Hand
Surgery Update 5. Rosemont, IL: American Society for Surgery of the Hand; 2012:159-170.

© 2013 American Academy of Orthopaedic Surgeons 2013 Orthopaedic In-Training Examination


SECTION 3: Hand • 63

Figure 75

Question 75
The least gliding resistance for a flexor tendon laceration at the thumb palmar-digital crease as shown in
Figure 75 can be achieved with

1. a 6-strand repair.
2. a division and repair.
3. debridement of the partial laceration.
4. no debridement, motion therapy only.
5. multiple-strand core repair with epitendinous repair.

PREFERRED RESPONSE: 3

RECOMMENDED READINGS
Lilly SI, Messer TM. Complications after treatment of flexor tendon injuries. J Am Acad Orthop Surg.
2006 Jul;14(7):387-96. Review. PubMed PMID: 16822886.
Mostofi A, Palmer J, Akelman E. Flexor tendon injury. In: Chung KC, Murray PM, eds. Hand Surgery
Update 5. Rosemont, IL: American Society for Surgery of the Hand; 2012:181-192.

© 2013 American Academy of Orthopaedic Surgeons 2013 Orthopaedic In-Training Examination


64 • American Academy of Orthopaedic Surgeons

Figure 78a Figure 78b

Question 78
A 42-year-old woman has the injury shown in Figures 78a and 78b. The decision to treat the ulnar styloid
surgically is based upon which finding?

1. Patient age
2. Displacement of the radius fracture
3. Displacement of the ulnar styloid fracture
4. Position of the ulnar styloid after open reduction and internal fixation of the radius
5. Stability of the distal radioulnar joint after open reduction and internal fixation of the radius

PREFERRED RESPONSE: 5

RECOMMENDED READINGS
Kim JK, Yun YH, Kim DJ, Yun GU. Comparison of united and nonunited fractures of the ulnar styloid
following volar-plate fixation of distal radius fractures. Injury. 2011 Apr;42(4):371-5. Epub 2010 Oct 20.
PubMed PMID: 20961540.
Sammer DM, Shah HM, Shauver MJ, Chung KC. The effect of ulnar styloid fractures on patient-rated
outcomes after volar locking plating of distal radius fractures. J Hand Surg Am. 2009 Nov;34(9):1595-
602. PubMed PMID: 19896004.

© 2013 American Academy of Orthopaedic Surgeons 2013 Orthopaedic In-Training Examination


SECTION 3: Hand • 65

Video 88 (click to play)

Question 88
A 45-year-old man has lateral elbow pain that worsens with weightlifting and pushups. Video 88 shows a
dynamic examination of the elbow. The anatomic defect is in which ligament?

1. Annular
2. Transverse medial collateral
3. Anterior band of the medial collateral
4. Posterior band of the medial collateral
5. Lateral ulnar collateral

PREFERRED RESPONSE: 5

RECOMMENDED READINGS
Mehta JA, Bain GI. Posterolateral rotatory instability of the elbow. J Am Acad Orthop Surg. 2004 Nov-
Dec;12(6):405-15. Review. PubMed PMID: 15615506.
O’Driscoll SW, Zarkadas PC. Recurrent instability of the elbow. In: Wolfe SW, Hotchkiss RN, Pederson
WC, Kozin SH, eds. Green’s Operative Hand Surgery. 6th ed. Philadelphia, PA: Elsevier Churchill
Livingstone; 2011:890.

© 2013 American Academy of Orthopaedic Surgeons 2013 Orthopaedic In-Training Examination


66 • American Academy of Orthopaedic Surgeons

Question 95
The deformity caused by long-term arthritis of the first carpometacarpal joint of the hand often leads to a
secondary hyperextension arthrosis of which joint?

1. Midcarpal
2. Radiocarpal
3. Scaphotrapezotrapezoidal
4. Thumb interphalangeal
5. Thumb metacarpophalangeal

PREFERRED RESPONSE: 5

RECOMMENDED READINGS
Rozental TD, LaPorte DM. Hand and wrist reconstruction. In: Flynn JM, ed. Orthopaedic Knowledge
Update 10. Rosemont, IL: American Academy of Orthopaedic Surgeons; 2011:363-376.
Van Heest AE, Kallemeier P. Thumb carpal metacarpal arthritis. J Am Acad Orthop Surg. 2008
Mar;16(3):140-51. Review. PubMed PMID: 18316712.

Figure 103

Question 103
Figure 103 is the clinical photograph of a 62-year-old man with numbness and weakness that has been
progressing for 10 years. What is the most appropriate treatment to improve thumb function?

1. Functional splinting
2. Neurotization of the thenar muscles
3. Hypothenar muscle transfer to thumb intrinsic
4. Arthrodesis of the thumb carpometacarpal joint in abduction
5. Transfer of the extensor indicis proprius around the ulnar wrist

PREFERRED RESPONSE: 5

© 2013 American Academy of Orthopaedic Surgeons 2013 Orthopaedic In-Training Examination


SECTION 3: Hand • 67

RECOMMENDED READINGS
Anderson GA, Lee V, Sundararaj GD. Opponensplasty by extensor indicis and flexor digitorum
superficialis tendon transfer. J Hand Surg Br. 1992 Dec;17(6):611-4. PubMed PMID: 1484241.
Burkhalter W, Christensen RC, Brown P. Extensor indicis proprius opponensplasty. J Bone Joint Surg Am.
1973 Jun;55(4):725-32. PubMed PMID: 4283744.
Cawrse NH, Sammut D. A modification in technique of abductor digiti minimi (Huber) opponensplasty. J
Hand Surg Br. 2003 Jun;28(3):233-7. PubMed PMID: 12809655.

Question 115
A 50-year-old man sustained a clavicle fracture after a motorcycle collision. He has no sensation or motor
function in the biceps and triceps; however, he has very weak thenar and finger flexion and extension.
Which finding would suggest a postganglionic as opposed to a preganglionic injury?

1. Preservation of C8, T1 function


2. Preserved sensory nerve action potential
3. Pseudomeningocele on CT myelogram
4. Ptosis and miosis on the same side as the injury
5. The cervical paraspinal muscle is normal on electromyography

PREFERRED RESPONSE: 5

RECOMMENDED READINGS
Moran SL, Steinmann SP, Shin AY. Adult brachial plexus injuries: mechanism, patterns of injury, and
physical diagnosis. Hand Clin. 2005 Feb;21(1):13-24. Review. PubMed PMID: 15668062.
Shin AY, Spinner RJ, Steinmann SP, Bishop AT. Adult traumatic brachial plexus injuries. J Am Acad
Orthop Surg. 2005 Oct;13(6):382-96. Review. PubMed PMID: 16224111.

© 2013 American Academy of Orthopaedic Surgeons 2013 Orthopaedic In-Training Examination


68 • American Academy of Orthopaedic Surgeons

Figure 133a Figure 133b

Question 133
Figures 133a and 133b are the clinical photographs of a 34-year-old woman with increasing pain in her
index finger for 3 days. The pain is worse with passive extension. Appropriate treatment should consist of

1. observation.
2. hand therapy.
3. oral antibiotics.
4. intravenous antibiotics.
5. irrigation and debridement of the flexor tendon sheath.

PREFERRED RESPONSE: 5

RECOMMENDED READINGS
Capo JT. Infections. In: Chung KC, Murray PM, eds. Hand Surgery Update 5. Rosemont, IL: American
Society for Surgery of the Hand; 2012:394-399.
Draeger RW, Bynum DK Jr. Flexor tendon sheath infections of the hand. J Am Acad Orthop Surg. 2012
Jun;20(6):373-82. Review. PubMed PMID: 22661567.

© 2013 American Academy of Orthopaedic Surgeons 2013 Orthopaedic In-Training Examination


SECTION 3: Hand • 69

Figure 140

Question 140
A 38-year-old woman had a distal radius fracture treated with a short-arm cast 3 months ago. The fracture
healed in good alignment. Figure 140 shows her attempt to extend her thumb. What is the best treatment
option?

1. Static splinting
2. Dynamic splinting
3. Transfer of the extensor pollicis brevis
4. Transfer of the extensor indicis proprius
5. Arthrodesis of the interphalangeal joint

PREFERRED RESPONSE: 4

RECOMMENDED READINGS
Roth KM, Blazar PE, Earp BE, Han R, Leung A. Incidence of extensor pollicis longus tendon rupture
after nondisplaced distal radius fractures. J Hand Surg Am. 2012 May;37(5):942-7. Epub 2012 Mar 29.
PubMed PMID: 22463927.
Skoff HD. Postfracture extensor pollicis longus tenosynovitis and tendon rupture: a scientific study and
personal series. Am J Orthop (Belle Mead NJ). 2003 May;32(5):245-7. PubMed PMID: 12772876.

© 2013 American Academy of Orthopaedic Surgeons 2013 Orthopaedic In-Training Examination


70 • American Academy of Orthopaedic Surgeons

Figure 213a Figure 213b

Question 213
Figures 213a and 213b are the clinical photograph and biopsy specimen of a 65-year-old man with a lesion
under his thumbnail that was biopsied by a dermatologist. Appropriate treatment should consist of

1. observation.
2. local excision.
3. marginal excision.
4. thumb ray resection.
5. amputation at the interphalangeal joint.

PREFERRED RESPONSE: 5

RECOMMENDED READINGS
Haase SC, Chung KC. Skin tumors. In: Wolfe SW, Hotchkiss RN, Pederson WC, Kozin SH, eds. Green’s
Operative Hand Surgery. 6th ed. Philadelphia, PA: Elsevier Churchill Livingstone; 2011:2131-2133.
Plate AM, Steiner G, Posner MA. Malignant tumors of the hand and wrist. J Am Acad Orthop Surg. 2006
Nov;14(12):680-92. PubMed PMID: 17077340.

© 2013 American Academy of Orthopaedic Surgeons 2013 Orthopaedic In-Training Examination


SECTION 3: Hand • 71

Figure 225

Question 225
Figure 225 is the clinical photograph of a 26-year-old man who fell through a window and sustained a
laceration to his thumb 5 days ago. He is unable to flex his thumb. Treatment should include

1. palmaris longus tendon transfer.


2. reconstruction with a palmaris longus free tendon bridge graft.
3. direct repair of the flexor pollicis longus with core sutures only.
4. repair of the flexor pollicis longus with core and epitendinous sutures.
5. transfer of the flexor digitorum superficialis of the ring finger to the thumb.

PREFERRED RESPONSE: 4

RECOMMENDED READINGS
Gulihar A, Hajipour L, Dias JJ. Comparison of three different peripheral suturing techniques for partial
flexor tendon lacerations: a controlled in-vitro biomechanical study. Hand Surg. 2012 2012;17(2):155-160.
PubMed PMID: 22745077.
Lee SK, Goldstein RY, Zingman A, Terranova C, Nasser P, Hausman MR. The effects of core suture
purchase on the biomechanical characteristics of a multistrand locking flexor tendon repair: a cadaveric
study. J Hand Surg Am. 2010 Jul;35(7):1165-71. Epub 2010 Jun 11. PubMed PMID: 20541326.
Coats RW II, Echevarría-Oré JC, Mass DP. Acute flexor tendon repairs in zone II. Hand Clin. 2005
May;21(2):173-9. Review. PubMed PMID: 15882596.
Barrie KA, Wolfe SW, Shean C, Shenbagamurthi D, Slade JF III, Panjabi MM. A biomechanical
comparison of multistrand flexor tendon repairs using an in situ testing model. J Hand Surg Am. 2000
May;25(3):499-506. PubMed PMID: 10811755.

© 2013 American Academy of Orthopaedic Surgeons 2013 Orthopaedic In-Training Examination


72 • American Academy of Orthopaedic Surgeons

Figure 230a Figure 230b Figure 230c Figure 230d

Question 230
Figures 230a through 230d are the pre- and postreduction radiographs of a 6-year-old boy who had a
fracture of the radius and ulna shafts in the distal diaphyses. Successful reduction of the completely
displaced fractures is achieved. To best maintain reduction while minimizing complications, treatment
should include immobilization in a

1. removable splint.
2. sugar-tong splint.
3. short-arm cast.
4. long-arm cast.
5. long-arm thumb spica cast.

PREFERRED RESPONSE: 3

RECOMMENDED READINGS
Paneru SR, Rijal R, Shrestha BP, Nepal P, Khanal GP, Karn NK, Singh MP, Rai P. Randomized controlled
trial comparing above- and below-elbow plaster casts for distal forearm fractures in children. J Child
Orthop. 2010 Jun;4(3):233-7. Epub 2010 Mar 17. PubMed PMID: 21629372.
Webb GR, Galpin RD, Armstrong DG. Comparison of short and long arm plaster casts for displaced
fractures in the distal third of the forearm in children. J Bone Joint Surg Am. 2006 Jan;88(1):9-17.
PubMed PMID: 16391244.
Bohm ER, Bubbar V, Yong Hing K, Dzus A. Above and below-the-elbow plaster casts for distal forearm
fractures in children. A randomized controlled trial. J Bone Joint Surg Am. 2006 Jan;88(1):1-8. PubMed
PMID: 16391243.

© 2013 American Academy of Orthopaedic Surgeons 2013 Orthopaedic In-Training Examination


SECTION 3: Hand • 73

Figure 240

Question 240
Figure 240 is the clinical photograph of a 33-year-old man who sustained a thumb pulp injury. There is
1-cm necrosis and tissue loss at the distal flap edge. What is the most appropriate treatment option?

1. Cross-finger flap
2. Volar advancement flap
3. Free microvascular pulp reconstruction
4. Local wound care with wet-to-moist dressings
5. Local rotation flap from the dorsal metacarpal vessels

PREFERRED RESPONSE: 2

RECOMMENDED READINGS
Baumeister S, Menke H, Wittemann M, Germann G. Functional outcome after the Moberg advancement
flap in the thumb. J Hand Surg Am. 2002 Jan;27(1):105-14. PubMed PMID: 11810623.
Macht SD, Watson HK. The Moberg volar advancement flap for digital reconstruction. J Hand Surg Am.
1980 Jul;5(4):372-6. PubMed PMID: 7419881.
Horta R, Barbosa R, Oliveira I, Amarante JM, Marques M, Cruz Reis J, Rebelo M. Neurosensible
reconstruction of the thumb in an emergency situation: review of 107 cases. Tech Hand Up Extrem Surg.
2009 Jun;13(2):85-9. PubMed PMID: 19516133.

© 2013 American Academy of Orthopaedic Surgeons 2013 Orthopaedic In-Training Examination


74 • American Academy of Orthopaedic Surgeons

Question 256
An 84-year-old patient who has been hospitalized for pneumonia has developed isolated wrist pain and
swelling with an effusion. The wrist is aspirated, the nucleated cell count is 75,000 cells/mm3, and urate
crystals are identified. What is the most important next treatment step?

1. Begin allopurinol.
2. Begin nonsteroidal anti-inflammatory drugs.
3. Administer a corticosteroid wrist injection.
4. Obtain cultures and begin empiric antibiotics.
5. Obtain radiographs to evaluate for a wrist fracture.

PREFERRED RESPONSE: 4

RECOMMENDED READINGS
Shah K, Spear J, Nathanson LA, McCauley J, Edlow JA. Does the presence of crystal arthritis rule out
septic arthritis? J Emerg Med. 2007 Jan;32(1):23-6. PubMed PMID: 17239729.
Schuind FA, Remmelink M, Pasteels JL. Co-existent gout and septic arthritis at the wrist: a case report.
Hand Surg. 2003 Jul;8(1):107-9. PubMed PMID: 12923944.
Yu KH, Luo SF, Liou LB, Wu YJ, Tsai WP, Chen JY, Ho HH. Concomitant septic and gouty arthritis--an
analysis of 30 cases. Rheumatology (Oxford). 2003 Sep;42(9):1062-6. Epub 2003 Apr 16. PubMed PMID:
12730521.

© 2013 American Academy of Orthopaedic Surgeons 2013 Orthopaedic In-Training Examination


SECTION 3: Hand • 75

Figure 269a Figure 269b

Question 269
Figures 269a and 269b are the MRI scans of a 60-year-old man who has pain and loss of elbow flexion
strength. In addition to the distal biceps tendon injury, what is the most likely diagnosis?

1. Soft-tissue sarcoma
2. Intraneural ganglion cyst
3. Denervation of the biceps muscle
4. Benign peripheral nerve sheath tumor
5. Malignant peripheral nerve sheath tumor

PREFERRED RESPONSE: 4

RECOMMENDED READINGS
Bhargava R, Parham DM, Lasater OE, Chari RS, Chen G, Fletcher BD. MR imaging differentiation
of benign and malignant peripheral nerve sheath tumors: use of the target sign. Pediatr Radiol. 1997
Feb;27(2):124-9. PubMed PMID: 9028843.
Uetani M, Hashmi R, Hayashi K, Nagatani Y, Narabayashi Y, Imamura K. Peripheral nerve intraneural
ganglion cyst: MR findings in three cases. J Comput Assist Tomogr. 1998 Jul-Aug;22(4):629-32. Review.
PubMed PMID: 9676458.

© 2013 American Academy of Orthopaedic Surgeons 2013 Orthopaedic In-Training Examination


76 • American Academy of Orthopaedic Surgeons

SECTION 4: Hip and Knee Reconstruction

Question 5
The ability of bacteria to adhere to orthopaedic implants and elude antimicrobial therapies through the use
of biofilm is attributable to their ability to produce

1. pyrrolidonyl arylamidase.
2. virulence factor exotoxin A.
3. Panton-Valentine leukocidin.
4. exopolysaccharide glycocalyx.
5. glyceraldehyde-3-phosphate dehydrogenase.

PREFERRED RESPONSE: 4

RECOMMENDED READINGS
Nguyen LL, Nelson CL, Saccente M, Smeltzer MS, Wassell DL, McLaren SG. Detecting bacterial
colonization of implanted orthopaedic devices by ultrasonication. Clin Orthop Relat Res. 2002
Oct;(403):29-37. PubMed PMID: 12360004.
Fux CA, Stoodley P, Hall-Stoodley L, Costerton JW. Bacterial biofilms: a diagnostic and therapeutic
challenge. Expert Rev Anti Infect Ther. 2003 Dec;1(4):667-83. PubMed PMID: 15482163.

© 2013 American Academy of Orthopaedic Surgeons 2013 Orthopaedic In-Training Examination


SECTION 4: Hip and Knee Reconstruction • 77

Figure 20a Figure 20b

Question 20
Figures 20a and 20b are the radiograph and MRI scan of a 58-year-old man who had total hip arthroplasty
3 years ago. His hip has been increasingly painful for 6 months. Laboratory studies show an erythrocyte
sedimentation rate of 24 mm/h (reference range [rr], 0-20 mm/h) and a C-reactive protein level of 0.3
mg/L (rr, 0.08-3.1 mg/L). In Figure 20b, which abnormality is indicated by the arrows?

1. Infection
2. Malignancy
3. Pseudotumor
4. Polyethylene debris
5. Heterotopic ossification

PREFERRED RESPONSE: 3

RECOMMENDED READINGS
Daniel J, Holland J, Quigley L, Sprague S, Bhandari M. Pseudotumors associated with total hip
arthroplasty. J Bone Joint Surg Am. 2012 Jan 4;94(1):86-93. Review. PubMed PMID: 22218386.
Hart AJ, Satchithananda K, Liddle AD, Sabah SA, McRobbie D, Henckel J, Cobb JP, Skinner JA, Mitchell
AW. Pseudotumors in association with well-functioning metal-on-metal hip prostheses: a case-control
study using three-dimensional computed tomography and magnetic resonance imaging. J Bone Joint Surg
Am. 2012 Feb 15;94(4):317-25. PubMed PMID: 22336970.

© 2013 American Academy of Orthopaedic Surgeons 2013 Orthopaedic In-Training Examination


78 • American Academy of Orthopaedic Surgeons

Question 31
Which population is least likely to receive total joint arthroplasty?

1. Black men
2. Black women
3. White men
4. White women
5. Hispanic men

PREFERRED RESPONSE: 1

RECOMMENDED READINGS
Skinner J, Weinstein JN, Sporer SM, Wennberg JE. Racial, ethnic, and geographic disparities in rates of
knee arthroplasty among Medicare patients. N Engl J Med. 2003 Oct 2;349(14):1350-9. PubMed PMID:
14523144.
Nelson CL. Disparities in orthopaedic surgical intervention. J Am Acad Orthop Surg. 2007;15 Suppl
1:S13-7. PubMed PMID: 17766783.

Question 44
A 70-year-old healthy man had total knee arthroplasty 18 years ago, and it now is painful. Radiographs
reveal aseptic loosening and the range of motion before surgery is 15 to 85 degrees. The strongest
indication for performing a tibial tubercle osteotomy to aid in exposure in his knee would be

1. patella baja.
2. nonresurfaced patella.
3. isolated femoral revision.
4. noncemented tibial component.
5. previous use of the quadriceps turn-down technique.

PREFERRED RESPONSE: 1

RECOMMENDED READINGS
Whiteside LA. Exposure in difficult total knee arthroplasty using tibial tubercle osteotomy. Clin Orthop
Relat Res. 1995 Dec;(321):32-5. PubMed PMID: 7497683.
Younger AS, Duncan CP, Masri BA. Surgical exposures in revision total knee arthroplasty. J Am Acad
Orthop Surg. 1998 Jan-Feb;6(1):55-64. Review. PubMed PMID: 9692941.
Mendes MW, Caldwell P, Jiranek WA. The results of tibial tubercle osteotomy for revision total knee
arthroplasty. J Arthroplasty. 2004 Feb;19(2):167-74. PubMed PMID: 14973859.

© 2013 American Academy of Orthopaedic Surgeons 2013 Orthopaedic In-Training Examination


SECTION 4: Hip and Knee Reconstruction • 79

Figure 50

Question 50
Figure 50 is the radiograph of a 45-year-old man who has avascular necrosis of the hip attributable to his
sickle cell anemia. He is scheduled for total hip arthroplasty. To prevent the most likely intrasurgical
technical complication, particular attention should be directed toward

1. dislocating the hip.


2. preparing the femur.
3. reaming the acetabulum.
4. inserting the acetabular screws.
5. cutting the short external rotators.

PREFERRED RESPONSE: 2

RECOMMENDED READINGS
Jeong GK, Ruchelsman DE, Jazrawi LM, Jaffe WL. Total hip arthroplasty in sickle cell
hemoglobinopathies. J Am Acad Orthop Surg. 2005 May-Jun;13(3):208-17. Review. PubMed PMID:
15938609.
Hernigou P, Zilber S, Filippini P, Mathieu G, Poignard A, Galacteros F. Total THA in adult osteonecrosis
related to sickle cell disease. Clin Orthop Relat Res. 2008 Feb;466(2):300-8. Epub 2008 Jan 10. PubMed
PMID: 18196410.

© 2013 American Academy of Orthopaedic Surgeons 2013 Orthopaedic In-Training Examination


80 • American Academy of Orthopaedic Surgeons

Question 59
A 63-year-old woman with rheumatoid arthritis is undergoing a knee arthroplasty. Her rheumatoid
arthritis has been well controlled with methotrexate, etanercept, and naproxen. Which medication-related
instructions should be followed 7 days before surgery?

1. Continue all medications


2. Discontinue naproxen
3. Discontinue naproxen and etanercept
4. Discontinue naproxen and methotrexate
5. Discontinue naproxen, etanercept, and methotrexate

PREFERRED RESPONSE: 3

RECOMMENDED READINGS
Howe CR, Gardner GC, Kadel NJ. Perioperative medication management for the patient with rheumatoid
arthritis. J Am Acad Orthop Surg. 2006 Sep;14(9):544-51. Review. PubMed PMID: 16959892.
Giles JT, Bartlett SJ, Gelber AC, Nanda S, Fontaine K, Ruffing V, Bathon JM. Tumor necrosis factor
inhibitor therapy and risk of serious postoperative orthopedic infection in rheumatoid arthritis. Arthritis
Rheum. 2006 Apr 15;55(2):333-7. PubMed PMID: 16583385.
Perhala RS, Wilke WS, Clough JD, Segal AM. Local infectious complications following large joint
replacement in rheumatoid arthritis patients treated with methotrexate versus those not treated with
methotrexate. Arthritis Rheum. 1991Feb;34(2):146-52. PubMed PMID: 1994911.

© 2013 American Academy of Orthopaedic Surgeons 2013 Orthopaedic In-Training Examination


SECTION 4: Hip and Knee Reconstruction • 81

Figure 76a Figure 76b Figure 76c

Question 76
Figures 76a through 76c are the anteroposterior and lateral radiographs and bone scan of a 66-year-old
man with type I diabetes mellitus who had revision right total knee arthroplasty for aseptic loosening 3
years ago. He has pain over the proximal tibia with startup and at the end of the day. He has difficulty
walking on level ground. Laboratory studies reveal an erythrocyte sedimentation rate of 5 mm/h
(reference range [rr], 0-20 mm/h) and C-reactive protein of <3.0 mg/L (rr, 0.08-3.1 mg/L). Synovial fluid
has 389 nucleated cells with 11% neutrophils and cultures are negative. What is the most likely failure
mechanism for this revision total knee arthroplasty?

1. Unrecognized fungal infection


2. Improper component alignment
3. Posterior cruciate ligament insufficiency
4. Aseptic loosening because of inadequate diaphyseal fixation
5. Aseptic loosening because of inadequate metaphyseal fixation

PREFERRED RESPONSE: 5

RECOMMENDED READINGS
Haidukewych GJ, Hanssen A, Jones RD. Metaphyseal fixation in revision total knee arthroplasty:
indications and techniques. J Am Acad Orthop Surg. 2011 Jun;19(6):311-8. Review. PubMed PMID:
21628642.
Bush JL, Wilson JB, Vail TP. Management of bone loss in revision total knee arthroplasty. Clin Orthop
Relat Res. 2006 Nov;452:186-92. Review. PubMed PMID: 16906109.

© 2013 American Academy of Orthopaedic Surgeons 2013 Orthopaedic In-Training Examination


82 • American Academy of Orthopaedic Surgeons

Figure 98a Figure 98b Figure 98c Figure 98d Figure 98e

Question 98
When templating total hip arthroplasty, which figure reveals the best recreation of the proper biomechanics
of the hip joint, assuming that the right leg is 5 mm shorter than the left?

1. Figure 98a
2. Figure 98b
3. Figure 98c
4. Figure 98d
5. Figure 98e

PREFERRED RESPONSE: 3

RECOMMENDED READINGS
Merle C, Waldstein W, Pegg E, Streit MR, Gotterbarm T, Aldinger PR, Murray DW, Gill HS. Femoral
offset is underestimated on anteroposterior radiographs of the pelvis but accurately assessed on
anteroposterior radiographs of the hip. J Bone Joint Surg Br. 2012 Apr;94(4):477-82. PubMed PMID:
22434462.
Della Valle AG, Padgett DE, Salvati EA. Preoperative planning for primary total hip arthroplasty. J Am
Acad Orthop Surg. 2005 Nov;13(7):455-62. Review. PubMed PMID: 16272270.

Question 105
Internal rotation of the femoral component can cause patella maltracking by

1. increasing the Q angle.


2. increasing the medial-directed force vector on the patella.
3. producing valgus malalignment.
4. tightening of the lateral retinaculum.
5. overstuffing the patellofemoral compartment.

PREFERRED RESPONSE: 1

© 2013 American Academy of Orthopaedic Surgeons 2013 Orthopaedic In-Training Examination


SECTION 4: Hip and Knee Reconstruction • 83

RECOMMENDED READINGS
Rhoads DD, Noble PC, Reuben JD, Tullos HS. The effect of femoral component position on the
kinematics of total knee arthroplasty. Clin Orthop Relat Res. 1993 Jan;(286):122-9. PubMed PMID:
8425333.
Malo M, Vince KG. The unstable patella after total knee arthroplasty: etiology, prevention, and
management. J Am Acad Orthop Surg. 2003 Sep-Oct;11(5):364-71. Review. PubMed PMID: 14565758.

Question 108
A 70-year-old man with osteoarthrosis is scheduled to undergo total knee arthroplasty. He inquires about
patellar resurfacing. He should be told that a potential advantage of having the patella resurfaced as
opposed to leaving the patella unresurfaced is

1. increased extensor strength.


2. lower risk for patellar fracture.
3. lower risk for requiring reoperation.
4. lower risk for patellar subluxation.
5. higher chance of achieving desirable range of motion.

PREFERRED RESPONSE: 3

RECOMMENDED READINGS
Meneghini RM. Should the patella be resurfaced in primary total knee arthroplasty? An evidence-based
analysis. J Arthroplasty. 2008 Oct;23(7 Suppl):11-4. Epub 2008 Aug 12. Review. PubMed PMID:
18701250.
Parvizi J, Rapuri VR, Saleh KJ, Kuskowski MA, Sharkey PF, Mont MA. Failure to resurface the patella
during total knee arthroplasty may result in more knee pain and secondary surgery. Clin Orthop Relat Res.
2005 Sep;438:191-6. PubMed PMID: 16131890.

© 2013 American Academy of Orthopaedic Surgeons 2013 Orthopaedic In-Training Examination


84 • American Academy of Orthopaedic Surgeons

Figure 121a Figure 121b

Question 121
Figures 121a and 121b are the current radiographs of a 39-year-old woman who had left total hip
arthroplasty 1 year ago. She is experiencing squeaking from the left hip while ambulating. Which factor
most likely contributes to her symptoms?

1. Activity level
2. Surgical approach
3. Component design
4. Component loosening
5. Component positioning

PREFERRED RESPONSE: 5

RECOMMENDED READINGS
Chevillotte C, Trousdale RT, Chen Q, Guyen O, An KN. The 2009 Frank Stinchfield Award: "Hip
squeaking": a biomechanical study of ceramic-on-ceramic bearing surfaces. Clin Orthop Relat Res. 2010
Feb;468(2):345-50. Epub 2009 Jun 19. PubMed PMID: 19543782.
Finkbone PR, Severson EP, Cabanela ME, Trousdale RT. Ceramic-on-ceramic total hip arthroplasty in
patients younger than 20 years. J Arthroplasty. 2012 Feb;27(2):213-9. Epub 2011 Aug 9. PubMed PMID:
21831576.

© 2013 American Academy of Orthopaedic Surgeons 2013 Orthopaedic In-Training Examination


SECTION 4: Hip and Knee Reconstruction • 85

Question 136
What is the optimal treatment for a Vancouver type B2 fracture in a healthy patient?

1. Retain the stem and fracture fixation with cortical strut graft and cables
2. Revision to a proximal femoral-replacing stem
3. Revision to a long porous-coated stem and cable fixation
4. Revision to a long cemented stem bypassing the fracture site
5. Revision to a proximally coated stem and open reduction and internal fixation of the fracture

PREFERRED RESPONSE: 3

RECOMMENDED READINGS
Corten K, Macdonald SJ, McCalden RW, Bourne RB, Naudie DD. Results of cemented femoral revisions
for periprosthetic femoral fractures in the elderly. J Arthroplasty. 2012 Feb;27(2):220-5. Epub 2011 Jul 12.
PubMed PMID: 21752585.
Mulay S, Hassan T, Birtwistle S, Power R. Management of types B2 and B3 femoral periprosthetic
fractures by a tapered, fluted, and distally fixed stem. J Arthroplasty. 2005 Sep;20(6):751-6. PubMed
PMID: 16139712.
Springer BD, Berry DJ, Lewallen DG. Treatment of periprosthetic femoral fractures following total hip
arthroplasty with femoral component revision. J Bone Joint Surg Am. 2003 Nov;85-A(11):2156-62.
PubMed PMID: 14630846.

© 2013 American Academy of Orthopaedic Surgeons 2013 Orthopaedic In-Training Examination


86 • American Academy of Orthopaedic Surgeons

Figure 153a Figure 153b

Question 153
The failure of total hip arthroplasty using a zirconium-ceramic femoral head as seen in Figures 153a and
153b is most likely the result of

1. infection.
2. aseptic loosening.
3. bony impingement.
4. material properties.
5. component alignment.

PREFERRED RESPONSE: 4

RECOMMENDED READINGS
Traina F, Tassinari E, De Fine M, Bordini B, Toni A. Revision of ceramic hip replacements for fracture of
a ceramic component: AAOS exhibit selection. J Bone Joint Surg Am. 2011 Dec 21;93(24):e147. Review.
PubMed PMID: 22258782.
Hannouche D, Hamadouche M, Nizard R, Bizot P, Meunier A, Sedel L. Ceramics in total hip replacement.
Clin Orthop Relat Res. 2005 Jan;(430):62-71. Review. PubMed PMID: 15662305.

© 2013 American Academy of Orthopaedic Surgeons 2013 Orthopaedic In-Training Examination


SECTION 4: Hip and Knee Reconstruction • 87

Figure 164a Figure 164b Figure 164c

Figure 164d Figure 164e

Question 164
Which figure best shows the femoral component loosening?

1. Figure 164a
2. Figure 164b
3. Figure 164c
4. Figure 164d
5. Figure 164e

PREFERRED RESPONSE: 1

© 2013 American Academy of Orthopaedic Surgeons 2013 Orthopaedic In-Training Examination


88 • American Academy of Orthopaedic Surgeons

RECOMMENDED READINGS
Robbins GM, Masri BA, Garbuz DS, Duncan CP. Evaluation of pain in patients with apparently solidly
fixed total hip arthroplasty components. J Am Acad Orthop Surg. 2002 Mar-Apr;10(2):86-94. Review.
PubMed PMID: 11929203.
Ollivere B, Wimhurst JA, Clark IM, Donell ST. Current concepts in osteolysis. J Bone Joint Surg Br.
2012 Jan;94(1):10-5. Review. PubMed PMID: 22219240.
Hirakawa K, Jacobs JJ, Urban R, Saito T. Mechanisms of failure of total hip replacements: lessons learned
from retrieval studies. Clin Orthop Relat Res. 2004 Mar;(420):10-7. Review. PubMed PMID: 15057073.

Question 173
A 57-year-old woman had right total knee arthroplasty for varus gonarthrosis. Before surgery, her range
of motion was 5 to 110 degrees. At skin closure, her range of motion was 0 to 120 degrees. Her range of
motion at 10 weeks after surgery is 0 to 70 degrees. What is the best next treatment step?

1. Observation
2. Dynamic bracing
3. Manipulation under anesthesia
4. Revision with open adhesiolysis
5. Physical therapy with aggressive range of motion

PREFERRED RESPONSE: 3

RECOMMENDED READINGS
Namba RS, Inacio M. Early and late manipulation improve flexion after total knee arthroplasty. J
Arthroplasty. 2007 Sep;22(6 Suppl 2):58-61. Epub 2007 Jul 26. PubMed PMID: 17823017.
Keating EM, Ritter MA, Harty LD, Haas G, Meding JB, Faris PM, Berend ME. Manipulation after total
knee arthroplasty. J Bone Joint Surg Am. 2007 Feb;89(2):282-6. PubMed PMID: 17272441.

Question 183
When comparing the results of cemented all-polyethylene tibial components to metal-backed components,
the all-polyethylene tibia

1. is more expensive.
2. is more susceptible to fracture.
3. is associated with an elevated risk for polyethylene wear.
4. has an equivalent rate of aseptic loosening.
5. has higher failure rates when used in patients younger than age 70.

PREFERRED RESPONSE: 4

© 2013 American Academy of Orthopaedic Surgeons 2013 Orthopaedic In-Training Examination


SECTION 4: Hip and Knee Reconstruction • 89

RECOMMENDED READINGS
Voigt J, Mosier M. Cemented all-polyethylene and metal-backed polyethylene tibial components
used for primary total knee arthroplasty: a systematic review of the literature and meta-analysis of
randomized controlled trials involving 1798 primary total knee implants. J Bone Joint Surg Am. 2011 Oct
5;93(19):1790-8. Review. PubMed PMID: 22005864.
Toman J, Iorio R, Healy WL. All-polyethylene and metal-backed tibial components are equivalent with
BMI of less than 37.5. Clin Orthop Relat Res. 2012 Jan;470(1):108-16. PubMed PMID: 21997784.
Dalury DF, Tucker KK, Kelley TC. All-polyethylene tibial components in obese patients are associated
with low failure at midterm followup. Clin Orthop Relat Res. 2012 Jan;470(1):117-24. PubMed PMID:
21739322.

Question 189
When the liquid monomer (monomethacrylate) is added to polymer powder (polymethylmethacrylate),
the activator in the liquid monomer (N,N-Dimethyl-p-toluidine) comes in contact with the initiator in the
polymer powder and polymerization is initiated. What is the initiator?

1. Hylamer
2. Polystyrene
3. Barium sulfate
4. Benzoyl peroxide
5. Zirconium dioxide

PREFERRED RESPONSE: 4

RECOMMENDED READINGS
Webb JC, Spencer RF. The role of polymethylmethacrylate bone cement in modern orthopaedic surgery. J
Bone Joint Surg Br. 2007 Jul;89(7):851-7. Review. PubMed PMID: 17673574.
Ahmed AM, Morrey BF. Polymethylmethacrylate. In: Morrey BF, ed. Joint Replacement Arthroplasty. 3rd
ed. Philadelphia, PA: Elsevier Health Sciences; 2003:9.

© 2013 American Academy of Orthopaedic Surgeons 2013 Orthopaedic In-Training Examination


90 • American Academy of Orthopaedic Surgeons

Figure 197

Question 197
Figure 197 is the radiograph of a 62-year-old woman who is seen in the emergency department with a
dislocated left total hip arthroplasty. This is her seventh dislocation during the last 3 months and she most
recently had a liner revision. What is the best next treatment step?

1. Skeletal traction
2. Open reduction
3. Closed reduction
4. Component revision
5. Hip abduction orthosis

PREFERRED RESPONSE: 4

RECOMMENDED READINGS
Alberton GM, High WA, Morrey BF. Dislocation after revision total hip arthroplasty: an analysis of
risk factors and treatment options. J Bone Joint Surg Am. 2002 Oct;84-A(10):1788-92. PubMed PMID:
12377909.
Paterno SA, Lachiewicz PF, Kelley SS. The influence of patient-related factors and the position of the
acetabular component on the rate of dislocation after total hip replacement. J Bone Joint Surg Am. 1997
Aug;79(8):1202-10. PubMed PMID: 9278081.

© 2013 American Academy of Orthopaedic Surgeons 2013 Orthopaedic In-Training Examination


SECTION 4: Hip and Knee Reconstruction • 91

Figure 214

Question 214
Figure 214 is the current radiograph of a 74-year-old man who had right total hip arthroplasty 3 weeks
ago. He stumbled and has increasing pain with weight-bearing activity. What is the best next treatment
step?

1. Revision
2. Resection arthroplasty
3. Routine follow-up at 3 months
4. Open reduction and internal fixation
5. Nonweight bearing activity for 6 weeks

PREFERRED RESPONSE: 1

RECOMMENDED READINGS
Duncan CP, Masri BA. Fractures of the femur after hip replacement. Instr Course Lect. 1995;44:293-304.
Review. PubMed PMID: 7797866.
Springer BD, Berry DJ, Lewallen DG. Treatment of periprosthetic femoral fractures following total hip
arthroplasty with femoral component revision. J Bone Joint Surg Am. 2003 Nov;85-A(11):2156-62.
PubMed PMID: 14630846.

© 2013 American Academy of Orthopaedic Surgeons 2013 Orthopaedic In-Training Examination


92 • American Academy of Orthopaedic Surgeons

Figure 234a Figure 234b Figure 234c

Question 234
Figure 234a is the clinical photograph of an 82-year-old man who had left total knee arthroplasty 1
year ago. He has difficulty with pain and stiffness and recently noted swelling on the medial side. He
had aspiration of the knee 1 month ago with a cell count of 22,000/mm3 nucleated cells. Aerobic and
anaerobic culture and gram stain findings are negative. Laboratory studies reveal the erythrocyte
sedimentation rate and C-reactive protein are within defined limits. He is able to perform a straight-leg
raise. Range of motion is 15 to 80 degrees. Anteroposterior and lateral radiographs are shown in Figures
234b and 234c. What is the best next step?

1. An MRI scan to evaluate for possible vastus medialis oblique disruption


2. Physical therapy with biofeedback focusing on gentle range of motion
3. Reaspirate and send for aerobic, anaerobic, fungal, and acid fast bacilli cultures
4. Resection arthroplasty and placement of vancomycin and gentamicin cement spacer
5. Revision total knee arthroplasty, elevation of joint line for flexion contracture, repair of the
extensor mechanism disruption

PREFERRED RESPONSE: 3

RECOMMENDED READINGS
Phelan DM, Osmon DR, Keating MR, Hanssen AD. Delayed reimplantation arthroplasty for candidal
prosthetic joint infection: a report of 4 cases and review of the literature. Clin Infect Dis. 2002 Apr
1;34(7):930-8. Epub 2002 Feb 26. Review. PubMed PMID: 11880958.
Azzam K, Parvizi J, Jungkind D, Hanssen A, Fehring T, Springer B, Bozic K, Della Valle C, Pulido L,
Barrack R. Microbiological, clinical, and surgical features of fungal prosthetic joint infections: a multi-
institutional experience. J Bone Joint Surg Am. 2009 Nov;91 Suppl 6:142-9. PubMed PMID: 19884422.

© 2013 American Academy of Orthopaedic Surgeons 2013 Orthopaedic In-Training Examination


SECTION 4: Hip and Knee Reconstruction • 93

Question 241
A woman has activity-related right knee pain that is located medially and is sharp in nature. Radiographs
reveal medial compartment degenerative changes. She recently lost 40 pounds (intentionally) and has had
some improvement in symptoms. What other nonsurgical treatment modality has the best evidence for
your recommendation?

1. Acupuncture
2. Valgus off-loader brace
3. Quadriceps strengthening
4. Intra-articular cortisone injection
5. Intra-articular viscosupplementation injection

PREFERRED RESPONSE: 3

RECOMMENDED READINGS
Zhang W, Nuki G, Moskowitz RW, Abramson S, Altman RD, Arden NK, Bierma-Zeinstra S, Brandt
KD, Croft P, Doherty M, Dougados M, Hochberg M, Hunter DJ, Kwoh K, Lohmander LS, Tugwell P.
OARSI recommendations for the management of hip and knee osteoarthritis: part III: Changes in evidence
following systematic cumulative update of research published through January 2009. Osteoarthritis
Cartilage. 2010 Apr;18(4):476-99. Epub 2010 Feb 11. Review. PubMed PMID: 20170770.

© 2013 American Academy of Orthopaedic Surgeons 2013 Orthopaedic In-Training Examination


94 • American Academy of Orthopaedic Surgeons

Figure 253

Question 253
Figure 253 shows the fracture sustained by an otherwise healthy 61-year-old man who was knocked down
by an automobile door that was suddenly opened as he was riding his bicycle. Which treatment will most
likely provide him with the best long-term function?

1. Hemiarthroplasty
2. Total hip arthroplasty
3. Open reduction and internal fixation with a blade plate
4. Open reduction and internal fixation with a dynamic hip screw
5. Closed reduction and percutaneous cannulated screw fixation

PREFERRED RESPONSE: 2

RECOMMENDED READINGS
Lee BP, Berry DJ, Harmsen WS, Sim FH. Total hip arthroplasty for the treatment of an acute fracture
of the femoral neck: long-term results. J Bone Joint Surg Am. 1998 Jan;80(1):70-5. PubMed PMID:
9469311.
Ricci WM, Langer JS, Leduc S, Streubel PN, Borrelli JJ. Total hip arthroplasty for acute displaced femoral
neck fractures via the posterior approach: a protocol to minimize hip dislocation risk. Hip Int. 2011 Jun
8;21(3):344-350. doi: 10.5301/HIP.2011.8401. PubMed PMID: 21698586.

© 2013 American Academy of Orthopaedic Surgeons 2013 Orthopaedic In-Training Examination


SECTION 4: Hip and Knee Reconstruction • 95

Figure 259

Question 259
Figure 259 is the radiograph of an 85-year-old man who had hip arthroplasty 15 years ago. He is now
living in a nursing home, ambulating with a walker, and has dementia. During the past 3 months, his hip,
which had been previously stable, has dislocated 3 times. What is the most likely cause of the recurrent
dislocations?

1. Polyethylene wear
2. Small-diameter femoral head
3. Damage to the locking mechanism of the liner
4. Insufficient anteversion of the acetabular cup
5. Failure to comply with hip dislocation precautions

PREFERRED RESPONSE: 1

RECOMMENDED READINGS
Parvizi J, Picinic E, Sharkey PF. Revision total hip arthroplasty for instability: surgical techniques and
principles. J Bone Joint Surg Am. 2008 May;90(5):1134-42. Review. PubMed PMID: 18451408.
Pulido L, Restrepo C, Parvizi J. Total hip arthroplasty for instability. Clin Med Res. 2007 Jun;5(2):139-42.
PMID 17607050.
von Knoch M, Berry DJ, Harmsn WS, Morrey BF. Late dislocation after total hip arthroplasty. J Bone
Joint Surg Am 2002 Nov;84-A(11):1949-53. PubMed PMID: 12429754.

© 2013 American Academy of Orthopaedic Surgeons 2013 Orthopaedic In-Training Examination


96 • American Academy of Orthopaedic Surgeons

Figure 272a Figure 272b Figure 272c

Figure 272d Video 272e (click to play)

Question 272
Figures 272a through 272c are the current radiographs and CT reconstruction scan of a 58-year-old
woman who has increasing pain with household ambulation. An intrasurgical video is shown in Figure
272e. After undergoing treatment as seen in Figure 272d, what is the most likely complication?

1. Infection
2. Instability
3. Nonunion
4. Aseptic loosening
5. Periprosthetic fracture

PREFERRED RESPONSE: 2

© 2013 American Academy of Orthopaedic Surgeons 2013 Orthopaedic In-Training Examination


SECTION 4: Hip and Knee Reconstruction • 97

RECOMMENDED READINGS
Taunton MJ, Fehring TK, Edwards P, Bernasek T, Holt GE, Christie MJ. Pelvic discontinuity treated with
custom triflange component: a reliable option. Clin Orthop Relat Res. 2012 Feb;470(2):428-34. PubMed
PMID: 21997785.
Christie MJ, Barrington SA, Brinson MF, Ruhling ME, DeBoer DK. Bridging massive acetabular defects
with the triflange cup: 2- to 9-year results. Clin Orthop Relat Res. 2001 Dec;(393):216-27. PubMed
PMID: 11764351.

© 2013 American Academy of Orthopaedic Surgeons 2013 Orthopaedic In-Training Examination


98 • American Academy of Orthopaedic Surgeons

SECTION 5: System Based Practice

Question 18
A 60-year-old Middle Eastern woman with a dark complexion is seen in the emergency department for a
nondisplaced humeral fracture. She has osteoporosis based on a previous bone mineral density test and a
history of fracture. Laboratory studies should include measuring levels of

1. vitamin D2.
2. vitamin D3.
3. 25 hydroxycholecalciferol.
4. 1,25 dihydrocholecalciferol.
5. 24,25-dihydroxycholecalciferol.

PREFERRED RESPONSE: 3

RECOMMENDED READINGS
Patton CM, Powell AP, Patel AA. Vitamin D in orthopaedics. J Am Acad Orthop Surg. 2012
Mar;20(3):123-9. Review. PubMed PMID: 22382284.
Bogunovic L, Kim AD, Beamer BS, Nguyen J, Lane JM. Hypovitaminosis D in patients scheduled to
undergo orthopaedic surgery: a single-center analysis. J Bone Joint Surg Am. 2010 Oct 6;92(13):2300-4.
PubMed PMID: 20926724.

Question 40
A 63-year-old African-American man is scheduled for right shoulder arthroplasty for degenerative
joint disease. He has no history of infection, connective tissue disease, or tobacco and alcohol abuse.
Laboratory studies drawn on the day of surgery show a leukocyte count of 2.2 X 109 cells/L (reference
range, 4.5 X 109 cells/L). All other laboratory and presurgical evaluations are within defined limits. What
is the best appropriate course of action?

1. Perform the surgery after consultation with a hematologist.


2. Perform the surgery because this value is not abnormally low for this patient.
3. Delay surgery and immediately repeat the white blood cell count.
4. Cancel the surgery because of the abnormally low white blood cell count.
5. Cancel the surgery; a delay is needed to consider this value.

PREFERRED RESPONSE: 2

© 2013 American Academy of Orthopaedic Surgeons 2013 Orthopaedic In-Training Examination


SECTION 5: Medically Related Issues • 99

RECOMMENDED READINGS
Haddy TB, Rana SR, Castro O. Benign ethnic neutropenia: what is a normal absolute neutrophil count? J
Lab Clin Med. 1999 Jan;133(1):15-22. Review. PubMed PMID: 10385477.
Eichner ER. Sports medicine pearls and pitfalls: benign neutropenia in athletes. Curr Sports Med Rep.
2009 Jul-Aug;8(4):162-3. Review. PubMed PMID: 19584599.
Hsieh MM, Everhart JE, Byrd-Holt DD, Tisdale JF, Rodgers GP. Prevalence of neutropenia in the U.S.
population: age, sex, smoking status, and ethnic differences. Ann Intern Med. 2007 Apr 3;146(7):486-92.
PubMed PMID: 17404350.

Question 77
What is the leading cause of medication errors, delays in diagnosis and treatment, and wrong-site
surgeries?

1. Not enough sleep


2. Too heavy a caseload
3. Inadequate preparation
4. Communication failures
5. Lack of patient participation

PREFERRED RESPONSE: 4

RECOMMENDED READINGS
Gandhi TK, Kachalia A, Thomas EJ, Puopolo AL, Yoon C, Brennan TA, Studdert DM. Missed and
delayed diagnoses in the ambulatory setting: a study of closed malpractice claims. Ann Intern Med. 2006
Oct 3;145(7):488-96. PubMed PMID: 17015866.
O’Daniel M, Rosenstein AH. Professional communication and team collaboration. In: Hughes RG,
ed. Patient Safety and Quality: An Evidence-Based Handbook for Nurses. Rockville, MD: Agency for
Healthcare Research and Quality (US); 2008 Apr. Chapter 33. PubMed PMID: 21328739.
Joint Commission on Accreditation of Health Care Organizations. The J. C. Guide to Improving Staff
Communication. Joint Commission Resources Memorandum, 2005:27-41.

© 2013 American Academy of Orthopaedic Surgeons 2013 Orthopaedic In-Training Examination


100 • American Academy of Orthopaedic Surgeons

Figure 85a Figure 85b

Figure 85c

Question 85
Figures 85a through 85c are the injury and reconstruction images of a 48-year-old man who had his right
arm amputated 5 cm below his elbow by a machine. His postsurgical course is uneventful. He is pleasant
in all his interactions with those treating him. He is fitted with and has learned to use a myoelectric
prosthesis. He is discharged from physical therapy with the evaluation that he is capable of returning to
his job activities. The best next step should be to

1. return him to work as soon as possible.


2. evaluate independently his ability to use his myoelectric prosthesis.
3. obtain a functional capacity evaluation and compare it to his job description.
4. offer the opportunity to be evaluated by psychology before returning to work.
5. recommend vocational rehabilitation because his prosthesis is too slow to use for work.

PREFERRED RESPONSE: 4

RECOMMENDED READINGS
Richards T, Garvert DW, McDade E, Carlson E, Curtin C. Chronic psychological and functional sequelae
after emergent hand surgery. J Hand Surg Am. 2011 Oct;36(10):1663-8. Epub 2011 Sep 8. PubMed PMID:
21862240.
Mallette P, Ring D. Attitudes of hand surgeons, hand surgery patients, and the general public regarding
psychologic influences on illness. J Hand Surg Am. 2006 Oct;31(8):1362-6. PubMed PMID: 17027800.

© 2013 American Academy of Orthopaedic Surgeons 2013 Orthopaedic In-Training Examination


SECTION 5: Medically Related Issues • 101

Question 96
Which finding is most associated with intimate partner violence?

1. Multiple extremity fractures


2. Isolated abdominal injury
3. Isolated lower-extremity fracture
4. Evidence of drug or alcohol use by the partner
5. Repeated visits to the emergency department

PREFERRED RESPONSE: 5

RECOMMENDED READINGS
Shields G, Baer J, Leininger K, Marlow J, DeKeyser P. Interdisciplinary health care and female victims of
domestic violence. Soc Work Health Care. 1998;27(2):27-48. PubMed PMID: 9606817.
Varvaro FF. Treatment of the battered woman: effective response of the emergency department. Am Coll
Emerg Phyicians 1989;11:8-13.

Question 150
A 25-year-old red-haired healthy woman with no history of substance abuse underwent fixation of a
fractured distal radius under supraclavicular block but required an unusual amount of intravenous sedation
and analgesia. Afterwards, she came to the emergency department in extreme pain. Compartment
syndrome was ruled out; on maximum dose oral opiates, she improved over time. At her postoperative
visit, she comes in with her red-haired mother, who related similar need for high dose pain medications.
Examination is otherwise uneventful. What is the most appropriate course?

1. Perform a drug test


2. Question her drug and alcohol history
3. Refer her to the pain service within a week
4. Give her more high-dose pain medication
5. Discuss a possible inherited lower pain tolerance

PREFERRED RESPONSE: 5

© 2013 American Academy of Orthopaedic Surgeons 2013 Orthopaedic In-Training Examination


102 • American Academy of Orthopaedic Surgeons

RECOMMENDED READINGS
Liem EB, Lin CM, Suleman MI, Doufas AG, Gregg RG, Veauthier JM, Loyd G, Sessler DI. Anesthetic
requirement is increased in redheads. Anesthesiology. 2004 Aug;101(2):279-83. PubMed PMID:
15277908.
Greenspan JD, Craft RM, LeResche L, Arendt-Nielsen L, Berkley KJ, Fillingim RB, Gold MS, Holdcroft
A, Lautenbacher S, Mayer EA, Mogil JS, Murphy AZ, Traub RJ; Consensus Working Group of the Sex,
Gender, and Pain SIG of the IASP. Studying sex and gender differences in pain and analgesia: a consensus
report. Pain. 2007 Nov;132 Suppl 1:S26-45. Epub 2007 Oct 25. PubMed PMID: 17964077.
Fillingim RB, King CD, Ribeiro-Dasilva MC, Rahim-Williams B, Riley JL 3rd. Sex, gender, and pain:
a review of recent clinical and experimental findings. J Pain. 2009 May;10(5):447-85. Review. PubMed
PMID: 19411059.
Tan EC, Lim EC, Teo YY, Lim Y, Law HY, Sia AT. Ethnicity and OPRM variant independently predict
pain perception and patient-controlled analgesia usage for post-operative pain. Mol Pain. 2009 Jun
23;5:32. PubMed PMID: 19545447.

Question 159
A 38-year-old Hindu man underwent serial debridement for necrotizing fasciitis of the hand and forearm.
Although the infection has cleared, he has extensive areas of exposed tendon both volarly and dorsally.
An option for coverage is an acellular collagen matrix derived from fetal bovine dermis. In addition to
obtaining routine informed consent for this procedure, the physician should explain the material’s

1. origin.
2. durability.
3. permeability.
4. resorption rate.
5. tensile strength.

PREFERRED RESPONSE: 1

RECOMMENDED READINGS
Easterbrook C, Maddern G. Porcine and bovine surgical products: Jewish, Muslim, and Hindu
perspectives. Arch Surg. 2008 Apr;143(4):366-70; discussion 370. PubMed PMID: 18427024.
Enoch S, Shaaban H, Dunn KW. Informed consent should be obtained from patients to use products (skin
substitutes) and dressings containing biological material. J Med Ethics. 2005 Jan;31(1):2-6. PubMed
PMID: 15634745.

© 2013 American Academy of Orthopaedic Surgeons 2013 Orthopaedic In-Training Examination


SECTION 5: Medically Related Issues • 103

Question 178
Ganglion excision is scheduled for a 55-year-old male laborer born in Mexico. You communicate between
his limited English, your modest Spanish, and his daughter’s command of both languages. He agrees to
the procedure, and wishes no further information by a translator, despite your offering patient brochures
and use of a patient-oriented computer kiosk. What is the next most appropriate step?

1. Postpone the surgery


2. Obtain translator services anyway
3. Ask whether he and his daughter understand the procedure
4. Request further family members be present for the decision
5. Ask him and his daughter to, in their own words, explain the proposed plan

PREFERRED RESPONSE: 5

RECOMMENDED READINGS
Katz JN, Lyons N, Wolff LS, Silverman J, Emrani P, Holt HL, Corbett KL, Escalante A, Losina E.
Medical decision-making among Hispanics and non-Hispanic Whites with chronic back and knee pain: a
qualitative study. BMC Musculoskelet Disord. 2011 Apr 21;12:78. PubMed PMID: 21510880.
Gooberman-Hill R, Sansom A, Sanders CM, Dieppe PA, Horwood J, Learmonth ID, Williams S, Donovan
JL. Unstated factors in orthopaedic decision-making: a qualitative study. BMC Musculoskelet Disord.
2010 Sep 17;11:213. PubMed PMID: 20849636.
Christopher Gibbons M. Use of health information technology among racial and ethnic underserved
communities. Perspect Health Inf Manag. 2011 Jan 1;8:1f. PubMed PMID: 21307989.

Question 209
An otherwise healthy 25-year-old Hispanic man is seen in the emergency department; he is accompanied
by his supervisor. He has a cut to his right hand from a table saw and requires emergency surgery. All of
his responses are single-word answers, and when asked if he has any questions before proceeding with
surgery, he says “No.” The physician should now

1. proceed with surgery, considering he has signed the informed consent sheet.
2. ask the supervisor to explain the procedure to him.
3. ask him to explain in his own words his injury and the proposed procedure.
4. have a second surgeon examine him and sign the informed consent.
5. discuss his surgery and planned after-surgery care through a translator before proceeding with
the surgery.

PREFERRED RESPONSE: 3

© 2013 American Academy of Orthopaedic Surgeons 2013 Orthopaedic In-Training Examination


104 • American Academy of Orthopaedic Surgeons

RECOMMENDED READINGS
Poon AW, Gray KV, Franco GC, Cerruti DM, Schreck MA, Delgado ED. Cultural competence: serving
Latino patients. J Pediatr Orthop. 2003 Jul-Aug;23(4):546-9. PubMed PMID: 12826958.
Katz JN, Lyons N, Wolff LS, Silverman J, Emrani P, Holt HL, Corbett KL, Escalante A, Losina E.
Medical decision-making among Hispanics and non-Hispanic Whites with chronic back and knee pain: a
qualitative study. BMC Musculoskelet Disord. 2011 Apr 21;12:78. PubMed PMID: 21510880.
Gooberman-Hill R, Sansom A, Sanders CM, Dieppe PA, Horwood J, Learmonth ID, Williams S, Donovan
JL. Unstated factors in orthopaedic decision-making: a qualitative study. BMC Musculoskelet Disord.
2010 Sep 17;11:213. PubMed PMID: 20849636.

Question 237
A 21-year-old man has leg weakness after a motor vehicle collision. Examination reveals normal strength
in his upper extremities, with 2/5 strength in the quadriceps, 2/5 ankle plantar flexion, and 0/5 ankle and
great toe extension. Examination shows no rectal tone but intact perirectal sensation. A CT scan reveals a
T9-T10 dislocation. What best describes his spinal cord injury?

1. Complete, American Spinal Injury Association (ASIA) A


2. Complete, ASIA B
3. Incomplete, ASIA B
4. Incomplete, ASIA C
5. Incomplete, ASIA D

PREFERRED RESPONSE: 4

RECOMMENDED READINGS
Furlan JC, Fehlings MG, Tator CH, Davis AM. Motor and sensory assessment of patients in clinical trials
for pharmacological therapy of acute spinal cord injury: psychometric properties of the ASIA Standards.
J Neurotrauma. 2008 Nov;25(11):1273-301. Review. Erratum in: J Neurotrauma. 2009 Mar;26(3):469.
PubMed PMID: 19061373.
Sie I, Waters RL. Outcomes following spinal cord injury. In: Spinal Cord Medicine. Lin VW, Cardenas
DD, Cutter NC, et al, eds. New York, NY: Demos Medical Publishing; 2003:87-101 http://www.ncbi.nlm.
nih.gov/books/NBK9053/ (Accessed 8/5/2013)

© 2013 American Academy of Orthopaedic Surgeons 2013 Orthopaedic In-Training Examination


SECTION 5: Medically Related Issues • 105

Question 250
When using a presurgical safety checklist and timeout, which surgical team member has been shown to be
the most effective in reducing surgical complications?

1. Scrub nurse
2. Circulating nurse
3. Anesthesiologist
4. Surgeon
5. Surgeon delegating different aspects of the presurgical checklist/timeout to various members
of the team

PREFERRED RESPONSE: 4

RECOMMENDED READINGS
Haynes AB, Weiser TG, Berry WR, Lipsitz SR, Breizat AH, Dellinger EP, Herbosa T, Joseph S, Kibatala
PL, Lapitan MC, Merry AF, Moorthy K, Reznick RK, Taylor B, Gawande AA; Safe Surgery Saves Lives
Study Group. A surgical safety checklist to reduce morbidity and mortality in a global population. N Engl
J Med. 2009 Jan 29;360(5):491-9. Epub 2009 Jan 14. PubMed PMID: 19144931.

Question 263
After treating a supracondylar humeral fracture in a 4-year-old child, nerve palsy is identified. The
treating physician should acknowledge the nerve damage and should offer

1. an apology and accept blame.


2. an apology and not accept blame.
3. an apology and accept partial blame.
4. no apology and accept blame.
5. no apology and not accept blame.

PREFERRED RESPONSE: 2

RECOMMENDED READINGS
Tongue JR, Otsuka NY. Patient-centered care: communication skills and cultural competence. In: Flynn
JM, ed. Orthopaedic Knowledge Update 10. Rosemont, IL: American Academy of Orthopaedic Surgeons;
2011:109-119.
Mazor KM, Simon SR, Yood RA, Martinson BC, Gunter MJ, Reed GW, Gurwitz JH. Health plan
members' views about disclosure of medical errors. Ann Intern Med. 2004 Mar 16;140(6):409-18. PubMed
PMID: 15023706.
MacDonald N, Attaran A. Medical errors, apologies and apology laws. CMAJ. 2009 Jan 6;180(1):11, 13.
English, French. PubMed PMID: 19124780.

© 2013 American Academy of Orthopaedic Surgeons 2013 Orthopaedic In-Training Examination


106 • American Academy of Orthopaedic Surgeons

SECTION 6: Oncology

Figure 4a Figure 4b Figure 4c

Figure 4d Figure 4e

Question 4
A 60-year-old woman has a high-grade pleomorphic undifferentiated sarcoma of the thigh. Which figure
supports this pathologic diagnosis?

1. Figure 4a
2. Figure 4b
3. Figure 4c
4. Figure 4d
5. Figure 4e

PREFERRED RESPONSE: 4

RECOMMENDED READINGS
Randall RL. Malignant soft-tissue tumors. In: Schwartz HS, ed. Orthopaedic Knowledge Update:
Musculoskeletal Tumors 2. Rosemont, IL: American Academy of Orthopaedic Surgeons; 2007:277-287.

© 2013 American Academy of Orthopaedic Surgeons 2013 Orthopaedic In-Training Examination


SECTION 6: Oncology • 107

Question 17
Which soft-tissue sarcoma is most likely to develop lymphatic metastasis?

1. Liposarcoma
2. Leiomyosarcoma
3. Synovial sarcoma
4. Myxoid liposarcoma
5. Pleomorphic sarcoma

PREFERRED RESPONSE: 3

RECOMMENDED READINGS
Randall RL. Malignant soft-tissue tumors. In: Schwartz, HS, ed. Orthopaedic Knowledge Update:
Musculoskeletal Tumors 2. Rosemont, IL: American Academy of Orthopaedic Surgeons; 2007:277-287.
Riad S, Griffin AM, Liberman B, Blackstein ME, Catton CN, Kandel RA, O’Sullivan B, White LM, Bell
RS, Ferguson PC, Wunder JS. Lymph node metastasis in soft tissue sarcoma in an extremity. Clin Orthop
Relat Res 2004 Sep:(426): 129-34. PubMed PMID: 15346063.

© 2013 American Academy of Orthopaedic Surgeons 2013 Orthopaedic In-Training Examination


108 • American Academy of Orthopaedic Surgeons

Figure 25a Figure 25b

Figure 25c

Question 25
Figures 25a through 25c are the axial T1 and postcontrast MRI scans and biopsy specimen of a 35-year-
old man with a painless right thigh mass. He noticed the mass about 2 weeks ago and is unsure if it has
changed in size. Which translocation most commonly is associated with this type of tumor?

1. t(2;13)(q35;q14)
2. t(12;22)(q13;q12)
3. t(12;16)(q13;p11)
4. t(X;18)(p11;q11)
5. t(17;22)(q22;q13)

PREFERRED RESPONSE: 3

RECOMMENDED READINGS
Krishnan B, Khanna G, Clohisy D. Gene translocations in musculoskeletal neoplasms. Clin Orthop Relat
Res. 2008 Sep;466(9):2131-46. Epub 2008 Jun 20. Review. PubMed PMID: 18566876.
Bode-Lesniewska B, Frigerio S, Exner U, Abdou MT, Moch H, Zimmermann DR. Relevance of
translocation type in myxoid liposarcoma and identification of a novel EWSR1-DDIT3 fusion. Genes
Chromosomes Cancer. 2007 Nov;46(11):961-71. PubMed PMID: 17647282.

© 2013 American Academy of Orthopaedic Surgeons 2013 Orthopaedic In-Training Examination


SECTION 6: Oncology • 109

Figure 34a Figure 34b

Figure 34c Figure 34d

Question 34
Figures 34a through 34d are the radiographs and biopsy specimen of a 68-year-old woman with an
8-month history of a slowly enlarging, painful distal left thigh mass. What is the recommended treatment?

1. Surgery alone
2. Radiotherapy alone
3. Radiotherapy and surgery
4. Neoadjuvant chemotherapy and surgery
5. Neoadjuvant chemotherapy, radiotherapy, and surgery

PREFERRED RESPONSE: 1

RECOMMENDED READINGS
Seo SW, Remotti F, Lee FYI. Chondrosarcoma of bone. In: Schwartz HS, ed. Orthopaedic Knowledge
Update: Musculoskeletal Tumors 2. Rosemont, IL: American Academy of Orthopaedic Surgeons;
2007:185-195.
Greenspan A, Jundt G, Remagen W, eds. Differential Diagnosis in Orthopaedic Oncology. 2nd ed.
Philadelphia, PA: Lippincott Williams & Wilkins; 2007:158-256.

© 2013 American Academy of Orthopaedic Surgeons 2013 Orthopaedic In-Training Examination


110 • American Academy of Orthopaedic Surgeons

Figure 47a Figure 47b Figure 47c

Figure 47d

Question 47
Figures 47a through 47d are the plain radiographs, axial MRI scan, and biopsy specimen of an 8-year-
old boy with progressive right elbow pain that awakens him from sleep. Examination reveals soft-tissue
fullness around his elbow and pain with active or passive motion. What is the most likely diagnosis?

1. Lymphoma
2. Osteomyelitis
3. Osteogenic sarcoma
4. Ewing sarcoma
5. Langerhans cell histiocytosis

PREFERRED RESPONSE: 5

RECOMMENDED READINGS
Hoover KB, Rosenthal DI, Mankin H. Langerhans cell histiocytosis. Skeletal Radiol. 2007 Feb;36(2):95-
104. Epub 2006 Oct 7. Review. PubMed PMID: 17028900.
Lichtenstein L, Jeffe HL. Eosinophilic granuloma of bone: With report of a case. Am J Pathol 1940
Sep;16(5):595-604.3. PubMed PMID: 19970524.

© 2013 American Academy of Orthopaedic Surgeons 2013 Orthopaedic In-Training Examination


SECTION 6: Oncology • 111

Figure 60a Figure 60b Figure 60c

Figure 60d Figure 60e

Question 60
Figures 60a through 60e are the radiographs, MRI scan, and biopsy specimen of a 17-year-old boy with a
3-month history of left hip pain that is constant and not relieved by anti-inflammatory medication. What is
the most likely diagnosis?

1. Enchondroma
2. Ewing sarcoma
3. Osteoblastoma
4. Chondrosarcoma
5. Chondroblastoma

PREFERRED RESPONSE: 5

© 2013 American Academy of Orthopaedic Surgeons 2013 Orthopaedic In-Training Examination


112 • American Academy of Orthopaedic Surgeons

RECOMMENDED READINGS
Sailhan F, Chotel F, Parot R, SOFOP. Chondroblastoma of bone in a pediatric population. J Bone Joint
Surg Am. 2009 Sep;91(9):2159-68. PubMed PMID: 19723993.
Weber KL, O’Connor MI. Benign cartilage tumors. In: Schwartz HS, ed. Orthopaedic Knowledge Update:
Musculoskeletal Tumors 2. Rosemont, IL: American Academy of Orthopaedic Surgeons; 2007:103-120.
Greenspan A, Jundt G, Remagen W, eds. Differential Diagnosis in Orthopaedic Oncology. 2nd ed.
Philadelphia, PA: Lippincott Williams & Wilkins; 2007:158-256.

Question 71
What is the treatment recommendation for an American Joint Committee on Cancer stage IIB (Enneking
stage IIB) malignant fibrous histiocytoma of bone?

1. Wide excision alone


2. Radiotherapy and wide excision
3. Chemotherapy and radiation
4. Chemotherapy and wide excision
5. Chemotherapy, wide excision, and radiotherapy

PREFERRED RESPONSE: 4

RECOMMENDED READINGS
Wittig JC, Villalobos CE. Other skeletal sarcomas. In: Schwartz HS, ed. Orthopaedic Knowledge Update:
Musculoskeletal Tumors 2. Rosemont, IL: American Academy of Orthopaedic Surgeons; 2007:197-204.
Jeon DG, Song WS, Kong CB, Kim JR, Lee SY. MFH of bone and osteosarcoma show similar survival
and chemosensitivity. Clin Orthop Relat Res. 2011 Feb;469(2):584-90. Epub 2010 Jun 18. PubMed PMID:
20559764.

© 2013 American Academy of Orthopaedic Surgeons 2013 Orthopaedic In-Training Examination


SECTION 6: Oncology • 113

Figure 79a Figure 79b

Figure 79c Figure 79d

Question 79
Figures 79a through 79d are the plain radiographs and axial CT scans of an 80-year-old woman with
severe dementia and a newly noted thigh mass. Examination reveals a large, nonmobile anterior thigh
mass that is minimally tender. What is the best next treatment step?

1. Biopsy
2. Chest CT scan
3. Observation
4. Wide resection
5. Marginal resection

PREFERRED RESPONSE: 3

RECOMMENDED READINGS
Beiner JM, Jokl P. Muscle contusion injuries: current treatment options. J Am Acad Orthop Surg. 2001
Jul-Aug;9(4):227-37. Review. PubMed PMID: 11476532.
Cushner FD, Morwessel RM. Myositis ossificans traumatica. Orthop Rev. 1992 Nov;21(11):1319-26.
Review. PubMed PMID: 1461667.

© 2013 American Academy of Orthopaedic Surgeons 2013 Orthopaedic In-Training Examination


114 • American Academy of Orthopaedic Surgeons

Figure 86a Figure 86b

Figure 86c

Question 86
Figures 86a through 86c are the radiographs and biopsy specimen of a 14-year-old boy who has had left
knee pain for 4 weeks. What is the most likely diagnosis?

1. Chondroblastoma
2. Chondrosarcoma
3. Parosteal osteosarcoma
4. Osteoblastoma
5. Osteosarcoma

PREFERRED RESPONSE: 5

RECOMMENDED READINGS
Messerschmitt PJ, Garcia RM, Abdul-Karim FW, Greenfield EM, Getty PJ. Osteosarcoma. J Am Acad
Orthop Surg. 2009 Aug;17(8):515-27. Review. PubMed PMID: 19652033.
Hornicek FJ. Osteosarcoma of bone. In: Schwartz HS, ed. Orthopaedic Knowledge Update:
Musculoskeletal Tumors 2. Rosemont, IL: American Academy of Orthopaedic Surgeons; 2007:163-174.
Greenspan A, Jundt G, Remagen W, eds. Differential Diagnosis in Orthopaedic Oncology. 2nd ed.
Philadelphia, PA: Lippincott Williams & Wilkins; 2007:40-157.

© 2013 American Academy of Orthopaedic Surgeons 2013 Orthopaedic In-Training Examination


SECTION 6: Oncology • 115

Figure 109a Figure 109b

Figure 109c Figure 109d

Question 109
Figures 109a through 109d are the plain radiographs and axial T2-weighted MRI scans of a 30-year-old
woman who has had right hip pain for the past month. Examination reveals an antalgic gait and a firm,
fixed proximal femoral mass. What is the best next treatment step?

1. Biopsy
2. Observation
3. Chemotherapy
4. Internal fixation
5. Radiation therapy

PREFERRED RESPONSE: 1

RECOMMENDED READINGS
Gibbs CP Jr, Weber K, Scarborough MT. Malignant bone tumors. Instr Course Lect 2002:51;413-28.
PubMed PMID: 12064130.
Messerschmitt PJ, Garcia RM, Abdul-Karim FW, Greenfield EM, Getty PJ. Osteosarcoma. J Am Acad
Orthop Surg. 2009 Aug;17(8):515-27. Review. PubMed PMID: 19652033.

© 2013 American Academy of Orthopaedic Surgeons 2013 Orthopaedic In-Training Examination


116 • American Academy of Orthopaedic Surgeons

Figure 127a Figure 127b Figure 127c

Question 127
Figures 127a through 127c are the femur radiographs of a 76-year-old man who is a community
ambulator. This man, who has biopsy-proven metastatic prostate cancer, has been experiencing left hip
and thigh pain while walking for 3 weeks. What is the best next treatment step?

1. Total hip arthroplasty


2. Long cephalomedullary fixation
3. Long cephalomedullary fixation and radiotherapy
4. Plate and screw hip fixation
5. Plate and screw hip fixation and radiotherapy

PREFERRED RESPONSE: 3

RECOMMENDED READINGS
Bickels J, Dadia S, Lidar Z. Surgical management of metastatic bone disease. J Bone Joint Surg Am. 2009
Jun;91(6):1503-16. Review. PubMed PMID: 19487532.
Damron TA. Treatment principles and prediction of the impending pathologic fracture. In: Schwartz HS,
ed. Orthopaedic Knowledge Update: Musculoskeletal Tumors 2. Rosemont, IL: American Academy of
Orthopaedic Surgeons; 2007:369-374.

© 2013 American Academy of Orthopaedic Surgeons 2013 Orthopaedic In-Training Examination


SECTION 6: Oncology • 117

Question 147
A 70-year-old woman with multiple myeloma is scheduled to begin intravenous bisphosphonate treatment
and has concerns about the side effects of this medication. In addition to reviewing the common side
effects, the following is advised before therapy initiation

1. an echocardiogram.
2. a chest radiograph.
3. a complete blood count.
4. a gastric swallowing study.
5. completion of invasive dental work.

PREFERRED RESPONSE: 5

RECOMMENDED READINGS
Pazianas M, Abrahamsen B. Safety of bisphosphonates. Bone. 2011Jul;49(1):103-10. Epub 2011 Jan 12.
Review. PubMed PMID: 21236370.
Pazianas M, Miller P, Blumentals WA, Bernal M, Kothawala P. A review of the literature on osteonecrosis
of the jaw in patients with osteoporosis treated with oral bisphosphonates: prevalence, risk factors, and
clinical characteristics. Clin Ther. 2007 Aug;29(8):1548-58. Review. PubMed PMID: 17919538.

© 2013 American Academy of Orthopaedic Surgeons 2013 Orthopaedic In-Training Examination


118 • American Academy of Orthopaedic Surgeons

Figure 155a Figure 155b

Question 155
Figures 155a and 155b are the plain radiographs of a 17-year-old boy who recently noted painless swelling
in his distal thigh. Examination reveals a firm, fixed, deep distal thigh mass. There is no associated
tenderness. What is the best next treatment step?

1. Biopsy
2. Resection
3. A CT scan
4. An MRI scan
5. Observation

PREFERRED RESPONSE: 5

RECOMMENDED READINGS
Douis H, Saifuddin A. The imaging of cartilaginous bone tumours. I. Benign lesions. Skeletal Radiol.
2012 Jun 17. PubMed PMID: 22707094.
Wilkins RM. Treatment of benign bone tumors. In: Menendez LR, ed. Orthopaedic Knowledge Update:
Musculoskeletal Tumors. Rosemont, IL: American Academy of Orthopaedic Surgeons; 2002:79-85.

© 2013 American Academy of Orthopaedic Surgeons 2013 Orthopaedic In-Training Examination


SECTION 6: Oncology • 119

Figure 170a Figure 170b Figure 170c

Question 170
Figures 170a through 170c are the plain radiographs and coronal short inversion time inversion recovery
(STIR) MRI scan of a 44-year-old woman with metastatic thyroid carcinoma and right shoulder pain. She
reports no history of trauma. Examination reveals no masses and considerable tenderness of the proximal
humerus. What is the best next treatment step?

1. Observation
2. Physical therapy
3. Radiation therapy
4. Prophylactic internal fixation
5. Proximal humeral replacement

PREFERRED RESPONSE: 3

RECOMMENDED READINGS
Evans AR, Bottros J, Grant W, Chen BY, Damron TA. Mirels' rating for humerus lesions is both
reproducible and valid. Clin Orthop Relat Res. 2008 Jun;466(6):1279-84. Epub 2008 Mar 21. PubMed
PMID: 18357496.
Mirels H. Metastatic disease in long bones. A proposed scoring system for diagnosing impending
pathologic fractures. Clin Orthop Relat Res. 1989 Dec;(249):256-64. PubMed PMID: 2684463.

© 2013 American Academy of Orthopaedic Surgeons 2013 Orthopaedic In-Training Examination


120 • American Academy of Orthopaedic Surgeons

Figure 180a Figure 180b Figure 180c Figure 180d

Figure 180e Figure 180f

Question 180
Figures 180a through 180f are the plain radiographs, coronal short inversion time inversion recovery
MRI scans, and biopsy specimen of a 27-year-old woman with an enlarging painless calf mass. She first
noticed the area several months ago and is unsure if the mass has increased in size. What is the most
likely diagnosis?

1. Desmoids
2. Liposarcoma
3. Schwannoma
4. Synovial sarcoma
5. Pleomorphic sarcoma

PREFERRED RESPONSE: 4

RECOMMENDED READINGS
Bakri A, Shinagare AB, Krajewski KM, Howard SA, Jagannathan JP, Hornick JL, Ramaiya NH. Synovial
sarcoma: imaging features of common and uncommon primary sites, metastatic patterns, and treatment
response. AJR Am J Roentgenol. 2012 Aug;199(2):W208-15. PubMed PMID: 22826423.
Kilpatrick SE. Diagnostic Musculoskeletal Surgical Pathology. Philadelphia, PA: Saunders; 2004.

© 2013 American Academy of Orthopaedic Surgeons 2013 Orthopaedic In-Training Examination


SECTION 6: Oncology • 121

Figure 210a Figure 210b

Question 210
Figures 210a and 210b are the axial T1 and postgadolinium MRI scans of a 67-year-old woman with a
right thigh mass that has grown in size and has become increasingly symptomatic for 6 months. What is
the best next treatment step?

1. Observation
2. Core biopsy
3. Wide resection
4. Marginal resection
5. Radiation therapy

PREFERRED RESPONSE: 4

RECOMMENDED READINGS
Damron TA. What to do with deep lipomatous tumors. Instr Course Lect. 2004;53:651-5. Review.
PubMed PMID: 15116655.
Rougraff BT, Durbin M, Lawerence J, Buckwalter K. Histologic correlation with magnetic resonance
imaging for benign and malignant lipomatous masses. Sarcoma. 1997;1(3-4):175-9. PubMed PMID:
18521221.

© 2013 American Academy of Orthopaedic Surgeons 2013 Orthopaedic In-Training Examination


122 • American Academy of Orthopaedic Surgeons

Figure 220a Figure 220b

Figure 220c

Question 220
Figures 220a through 220c are the radiograph, MRI scan, and biopsy specimen of a 5-year-old boy
with a 4-week history of right hip pain, limp, and a low-grade fever. He also has diabetes insipidus,
exopthalmus, and multiple lesions in the skull. What is the most likely diagnosis?

1. Ollier disease
2. Gorham disease
3. Hand-Schuller-Christian disease
4. Mazabraud syndrome
5. McCune-Albright syndrome

PREFERRED RESPONSE: 3

RECOMMENDED READINGS
Pitcher JD Jr, Weber KL. Benign fibrous and histiocytic lesions. In: Schwartz HS, ed. Orthopaedic
Knowledge Update: Musculoskeletal Tumors 2. Rosemont, IL: American Academy of Orthopaedic
Surgeons; 2007:121-132.
Velez-Yanguas MC, Warrier RP. Langerhans’ cell histiocytosis. Orthop Clin North Am. 1996
Jul;27(3):615-23. Review. PubMed PMID: 8649742.
Islinger RB, Kuklo TR, Owens BD, Horan PJ, Choma TJ, Murphey MD, Temple HT. Langerhans’ cell
histiocytosis in patients older than 21 years. Clin Orthop Relat Res. 2000 Oct;(379):231-5. PubMed
PMID: 11039811.
© 2013 American Academy of Orthopaedic Surgeons 2013 Orthopaedic In-Training Examination
SECTION 6: Oncology • 123

Figure 233a Figure 233b

Figure 233c

Question 233
Figures 233a through 233c are the radiographs and biopsy specimen of a 32-year-old woman who has had
progressive ankle pain for 6 months. What is the most appropriate treatment option?

1. Radiotherapy alone
2. Intralesional curettage
3. Intralesional curettage with adjuvants
4. Wide surgical resection and radiotherapy
5. Neoadjuvant chemotherapy and wide resection

PREFERRED RESPONSE: 3

RECOMMENDED READINGS
McDonald DJ, Weber KL. Giant cell tumor of bone. In: Schwartz HS, ed. Orthopaedic Knowledge
Update: Musculoskeletal Tumors 2. Rosemont, IL: American Academy of Orthopaedic Surgeons;
2007:133-140.
Arbeitsgemeinschaft Knochentumoren, Becker WT, Dohle J, Bernd L, Braun A, Cserhati M, Enderle A,
Hovy L, Matejovsky Z, Szendroi M, Trieb K, Tunn PU. Local recurrence of giant cell tumor of bone after
intralesional treatment with and without adjuvant therapy. J Bone Joint Surg Am. 2008 May;90(5):1060-7.
PubMed PMID: 18451399.

© 2013 American Academy of Orthopaedic Surgeons 2013 Orthopaedic In-Training Examination


124 • American Academy of Orthopaedic Surgeons

Figure 242

Question 242
Figure 242 is the anteroposterior radiograph of a 28-year-old man who underwent resection and
reconstruction for an Ewing sarcoma. What is the most common functional deficit encountered during
rehabilitation?

1. Hip instability
2. Abductor weakness
3. Adductor weakness
4. Quadriceps weakness
5. Leg-length discrepancy

PREFERRED RESPONSE: 2

RECOMMENDED READINGS
Parvizi J, Sim FH. Proximal femoral replacements with megaprostheses. Clin Orthop Relat Res 2004;
420: 169-75. PubMed PMID: 15057093.
Zehr RJ, Enneking WF, Scarborough MT. Allograft-prosthesis composite versus megaprosthesis in
proximal femoral reconstruction. Clin Orthop Relat Res 1996; 322:207-23. PubMed PMID: 8542698.

© 2013 American Academy of Orthopaedic Surgeons 2013 Orthopaedic In-Training Examination


SECTION 6: Oncology • 125

Figure 258a Figure 258b

Question 258
Figures 258a and 258b are the radiographs of a 48-year-old woman who had knee pain after falling from
a standing height. Her pain subsequently resolved. She was treated 2 years ago for invasive ductal breast
carcinoma. What is the best next step in management?

1. An MRI scan
2. Biopsy
3. Bone scan
4. Observation
5. Prophylactic fixation

PREFERRED RESPONSE: 4

RECOMMENDED READINGS
Marco RA, Gitelis S, Brebach GT, Healey JH. Cartilage tumors: evaluation and treatment. J Am Acad
Orthop Surg 2000; 8: 292-304. PubMed PMID: 11029557.
Unni KK, Inwards CY, eds. Dahlin’s Bone Tumors: General Aspects and Data on 10,165 Cases.
Philadelphia, PA: Lippincott Williams & Wilkins, 2010.

© 2013 American Academy of Orthopaedic Surgeons 2013 Orthopaedic In-Training Examination


126 • American Academy of Orthopaedic Surgeons

Figure 267a Figure 267b Figure 267c

Question 267
Figures 267a through 267c are the radiographs and biopsy specimen of a 10-year-old boy who is
experiencing lateral ankle pain during sports and recreational activities. What is the best next treatment
step?

1. Wide resection
2. Curettage and bone graft
3. Aspiration and steroid injection
4. Chemotherapy and wide resection
5. Observation and activity restriction

PREFERRED RESPONSE: 2

RECOMMENDED READINGS
Weber KL, Heck RK Jr. Cystic and benign bone lesions. In: Schwartz HS, ed. Orthopaedic Knowledge
Update: Musculoskeletal Tumors 2. Rosemont, IL: American Academy of Orthopaedic Surgeons;
2007:87-102.
Greenspan A, Jundt G, Remagen W, eds. Differential Diagnosis In Orthopaedic Oncology. 2nd ed.
Philadelphia, PA: Lippincott Williams & Wilkins; 2007:387-457.
Steffner RJ, Liao C, Stacy G, Atanda A, Attar S, Avedian R, Peabody TD. Factors associated with
recurrence of primary aneurysmal bone cysts: is argon beam coagulation an effective adjuvant treatment?
J Bone Joint Surg Am. 2011 Nov 2;93(21):e1221-9. PubMed PMID: 22048101.

© 2013 American Academy of Orthopaedic Surgeons 2013 Orthopaedic In-Training Examination


SECTION 7: Pediatric Orthopaedics • 127

SECTION 7: Pediatric Orthopaedics

Figure 3a Figure 3b

Question 3
Based on the findings shown in Figures 3a and 3b, what is the most likely diagnosis?

1. Mucopolysaccharidosis
2. Osteogenesis imperfecta
3. Legg-Calvé-Perthes disease of both hips
4. A history of developmental hip dysplasia that has been treated
5. A history of developmental hip dysplasia that has not been treated

PREFERRED RESPONSE: 1

RECOMMENDED READINGS
Link B, de Camargo Pinto LL, Giugliani R, Wraith JE, Guffon N, Eich E, Beck M. Orthopedic
manifestations in patients with mucopolysaccharidosis type II (Hunter syndrome) enrolled in the Hunter
Outcome Survey. Orthop Rev (Pavia). 2010 Sep 23;2(2):e16. PubMed PMID: 21808707.
Crossan JF, Wynne-Davies R, Fulford GE. Bilateral failure of the capital femoral epiphysis: bilateral
Perthes disease, multiple epiphyseal dysplasia, pseudoachondroplasia, and spondyloepiphyseal dysplasia
congenita and tarda. J Pediatr Orthop. 1983 Jul;3(3):297-301. PubMed PMID: 6409926.
Kitoh H, Kitakoji T, Kawasumi M, Ishiguro N. A histological and ultrastructural study of the iliac crest
apophysis in Legg-Calve-Perthes disease. J Pediatr Orthop. 2008 Jun;28(4):435-9. PubMed PMID:
18520280.

© 2013 American Academy of Orthopaedic Surgeons 2013 Orthopaedic In-Training Examination


128 • American Academy of Orthopaedic Surgeons

Figure 13a Figure 13b Figure 13c

Figure 13d Figure 13e Figure 13f

Question 13
Figures 13a through 13f are the coronal and sagittal CT scans of a boy who sustained a Salter-Harris II
fracture through the physis of the distal tibia, with an associated Salter-Harris I distal fibula fracture at
10 years of age. He was treated with closed reduction and cast immobilization. Now at age 12, he is
asymptomatic and has a lower limb-length discrepancy that is 1 cm shorter on the involved side and bone
age consistent with standards for 13-year-olds. His parents should be informed that

1. his growth has stopped and no additional visits are necessary.


2. the areas of physeal closure are too complex for reliable bar resection.
3. differential continued growth between the tibia and the fibula is unlikely.
4. epiphyseodesis of the contralateral tibia and fibula is indicated to reestablish length equality.
5. physeal bar resection with interposition of fat or bone cement is indicated to permit continued
growth of the distal tibia.

PREFERRED RESPONSE: 2

© 2013 American Academy of Orthopaedic Surgeons 2013 Orthopaedic In-Training Examination


SECTION 7: Pediatric Orthopaedics • 129

RECOMMENDED READINGS
Rathjen KE, Birch JG. Physeal injuries and growth disturbances. In: Beaty JH, Kasser JR, eds. Rockwood
and Wilkins’ Fractures in Children, 6th ed. Philadelphia, PA: Lippincott Williams & Wilkins; 2006:99-131.
Kang HG, Yoon SJ, Kim JR. Resection of a physeal bar under computer-assisted guidance. J Bone Joint
Surg Br 2010 Oct;92(10):1452-5. PubMed PMID: 20884987.
Marsh JS, Polzhofer GK. Arthroscopically assisted central physeal bar resection. J Pediatr Orthop 2006
Mar-Apr;26(2):255-9. PubMed PMID: 16557145.

Question 21
A 10-year-old boy has had the insidious onset of anterior knee pain in both knees for 3 months. He had
no inciting traumatic event and no pain at night or when walking, but has pain when ascending stairs or
running. Examination reveals full active and passive range of motion of both knees, no hip pain with log
roll, mild swelling over the tibial tubercle of both knees, and reproduction of the knee pain with direct
palpation of the tibial tubercles. The most appropriate next step should include

1. a 3-phase bone scan.


2. an MRI scan of both knees.
3. core strengthening exercises.
4. bilateral tibial tubercle osteotomies.
5. nonsteroidal anti-inflammatory drugs, activity modification, and gentle quadriceps stretching.

PREFERRED RESPONSE: 5

RECOMMENDED READINGS
Frank JB, Jarit GJ, Bravman JT, Rosen JE. Lower extremity injuries in the skeletally immature athlete. J
Am Acad Orthop Surg. 2007 Jun;15(6):356-66. Review. PubMed PMID: 17548885.
Gholve PA, Scher DM, Khakharia S, Widmann RF, Green DW. Osgood Schlatter syndrome. Curr Opin
Pediatr. 2007 Feb;19(1):44-50. Review. PubMed PMID: 17224661.

Question 29
A 15-year-old girl involved in a motor vehicle collision has severe back pain and is unable to move or
feel her legs. The emergency medical technician noticed a large ecchymotic area on her back at the
thoracolumbar junction. What is the most appropriate initial evaluation?

1. A CT scan of the entire spine


2. Placement of a Foley catheter
3. Radiographs of the thoracolumbar junction
4. Evaluation of the bulbocavernosus reflex
5. Evaluation of the airway, breathing, and circulation

PREFERRED RESPONSE: 5

© 2013 American Academy of Orthopaedic Surgeons 2013 Orthopaedic In-Training Examination


130 • American Academy of Orthopaedic Surgeons

RECOMMENDED READINGS
Schouten R, Albert T, Kwon BK. The spine-injured patient: initial assessment and emergency treatment. J
Am Acad Orthop Surg. 2012 Jun;20(6):336-46. Review. PubMed PMID: 22661563.
Kay RM, Skaggs DL. Pediatric polytrauma management. J Pediatr Orthop. 2006 Mar-Apr;26(2):268-77.
PubMed PMID: 16557148.

Figure 35a Figure 35b

Question 35
Figures 35a and 35b are the radiographs of a 9-year-old boy who fell from a tree and sustained a left
elbow fracture. With open reduction and internal fixation, which technique would minimize after-surgery
lateral spurring?

1. Screw fixation of the fracture


2. Bone wax over the lateral metaphysis
3. Removal of the Kirschner wires by 3 weeks after surgery
4. Delayed range of motion until 6 weeks after surgery
5. Anatomical restoration of the lateral periosteum

PREFERRED RESPONSE: 5

RECOMMENDED READINGS
Beaty JH, Kasser JR. The elbow: physeal fractures, apophyseal injuries of the distal humerus,
osteonecrosis of the trochlea, and T-condylar fractures. In: Beaty JH, Kasser JR, eds. Rockwood & Wilkins’
Fractures in Children. 7th ed. Philadelphia, PA: Lippincott Williams & Wilkins; 2010:533-593.
Pribaz JR, Bernthal NM, Wong TC, Silva M. Lateral spurring (overgrowth) after pediatric lateral condyle
fractures. J Pediatr Orthop. 2012 Jul;32(5):456-60. PubMed PMID: 22706459.

© 2013 American Academy of Orthopaedic Surgeons 2013 Orthopaedic In-Training Examination


SECTION 7: Pediatric Orthopaedics • 131

Figure 43a Figure 43b

Question 43
A 10-year-old girl has the injury seen in Figures 43a and 43b. She is treated with closed reduction and
percutaneous pinning with smooth Kirschner wires. What is the most common complication of this
fracture?

1. Medial meniscus tear


2. Arthrofibrosis of the knee
3. Distal femoral growth arrest
4. Superficial peroneal nerve palsy
5. Lateral femoral condyle osteochondral fracture

PREFERRED RESPONSE: 3

RECOMMENDED READINGS
Garrett BR, Hoffman EB, Carrara H. The effect of percutaneous pin fixation in the treatment of distal
femoral physeal fractures. J Bone Joint Surg Br. 2011 May;93(5):689-94. Review. PubMed PMID:
21511937.
Arkader A, Warner WC Jr, Horn BD, Shaw RN, Wells L. Predicting the outcome of physeal fractures of
the distal femur. J Pediatr Orthop. 2007 Sep;27(6):703-8. PubMed PMID: 17717475.

© 2013 American Academy of Orthopaedic Surgeons 2013 Orthopaedic In-Training Examination


132 • American Academy of Orthopaedic Surgeons

Figure 49

Question 49
Figure 49 is the radiograph of a child with a forearm fracture. Access to follow-up care will be limited in
most regions of the United States by

1. body mass index.


2. patient comorbidities.
3. type of immobilization in place.
4. referral expertise.
5. the number of orthopaedic surgeons willing to see pediatric patients.

PREFERRED RESPONSE: 5

RECOMMENDED READINGS
Iobst C, King W, Baitner A, Tidwell M, Swirsky S, Skaggs DL. Access to care for children with fractures.
J Pediatr Orthop. 2010 Apr-May;30(3):244-7. PubMed PMID: 20357590.
Skaggs DL. Less access to care for children with Medicaid. Orthopedics. 2003 Dec;26(12):1184, 1186.
PubMed PMID: 14690285.

© 2013 American Academy of Orthopaedic Surgeons 2013 Orthopaedic In-Training Examination


SECTION 7: Pediatric Orthopaedics • 133

Figure 56a Figure 56b Figure 56c

Figure 56d Figure 56e Figure 56f Figure 56g

Question 56
A 13-year-old boy sustained the injury shown in Figure 56a. Closed treatment under general anesthesia
was performed within 3 hours. Posttreatment CT scans shown in Figures 56b through 56d revealed
acceptable results. At the 7-week visit, the boy has remained asymptomatic and has begun some running
despite being given instructions for activity restrictions. Figures 56e through 56g show current standing
anteroposterior, standing “false profile,” and supine internally rotated radiographs of the hip. Based on
these findings, the parents should be informed that

1. there is concern about possible chondrolysis.


2. there is no further concern about possible osteonecrosis.
3. the hip is not completely reduced.
4. the radiographs show evidence of osteonecrosis of the femoral head.
5. heterotopic ossification is the result of the patient’s early return to activity.

PREFERRED RESPONSE: 1

RECOMMENDED READINGS
Blasier RD, Hughes LO. Fractures and traumatic dislocations of the hip in children. In: Beaty JH, Kasser
JR, eds. Rockwood and Wilkins’ Fractures in Children, 6th ed. Philadelphia, PA: Lippincott Williams &
Wilkins; 2006:861-891.
Hamilton PR, Broughton NS. Traumatic hip dislocation in childhood. J Pediatr Orthop 1998;18:691-694.
PubMed PMID: 9746428.

© 2013 American Academy of Orthopaedic Surgeons 2013 Orthopaedic In-Training Examination


134 • American Academy of Orthopaedic Surgeons

Question 64
A 12-year-old right-handed girl has right shoulder pain after pitching baseball. Examination reveals
right shoulder tenderness to palpation over the anterolateral aspect and mild weakness with resisted
internal rotation and abduction. Radiographs of her shoulder reveal no abnormalities. What is the most
appropriate next step in management?

1. MRI scan of the right shoulder


2. MR arthrogram of the right shoulder
3. Glenohumeral corticosteroid injection
4. Arthroscopic evaluation and repair of the middle glenohumeral ligament
5. 2-month hiatus from pitching followed by a progressive throwing program

PREFERRED RESPONSE: 5

RECOMMENDED READINGS
Chen FS, Diaz VA, Loebenberg M, Rosen JE. Shoulder and elbow injuries in the skeletally immature
athlete. J Am Acad Orthop Surg. 2005 May-Jun;13(3):172-85. Review. PubMed PMID: 15938606.
McFarland EG, Ireland ML. Rehabilitation programs and prevention strategies in adolescent throwing
athletes. Instr Course Lect. 2003;52:37-42. Review. PubMed PMID: 12690839.

© 2013 American Academy of Orthopaedic Surgeons 2013 Orthopaedic In-Training Examination


SECTION 7: Pediatric Orthopaedics • 135

Figure 70a Figure 70b Figure 70c

Figure 70d Figure 70e

Question 70
Figures 70a through 70e are the radiographs and MRI scan of a 12-year-old boy with worsening thigh
pain. What is the most appropriate definitive surgical treatment?

1. Observation with repeat follow up in 3 months


2. Curettage and placement of a bone graft substitute
3. Incision and drainage and placement of antibiotic beads
4. Resection of the distal one-third of the femur and knee fusion
5. Resection of the distal two-thirds of the femur and custom implant

PREFERRED RESPONSE: 5

RECOMMENDED READINGS
Maheshwari AV, Bergin PF, Henshaw RM. Modes of failure of custom expandable repiphysis prostheses:
a report of three cases. J Bone Joint Surg Am. 2011 Jul 6;93(13):e72. PubMed PMID: 21776557.
Campanacci L, Manfrini M, Colangeli M, Alí N, Mercuri M. Long-term results in children with
massive bone osteoarticular allografts of the knee for high-grade osteosarcoma. J Pediatr Orthop. 2010
Dec;30(8):919-27. PubMed PMID: 21102223.

© 2013 American Academy of Orthopaedic Surgeons 2013 Orthopaedic In-Training Examination


136 • American Academy of Orthopaedic Surgeons

Question 74
For treatment of displaced supracondylar humeral fractures in children, the AAOS clinical practice
guideline, The Treatment of Pediatric Supracondylar Humerus Fractures, recommends closed reduction
and pin fixation using which pin configuration?

1. 1 medial and 1 lateral


2. 1 medial and 2 lateral
3. 2 medial and 1 lateral
4. 2 to 3 lateral
5. 2 to 3 medial

PREFERRED RESPONSE: 4

RECOMMENDED READINGS
Howard A, Mulpuri K, Abel MF, Braun S, Bueche M, Epps H, Hosalkar H, Mehlman CT, Scherl S,
Goldberg M, Turkelson CM, Wies JL, Boyer K; American Academy of Orthopaedic Surgeons. The
treatment of pediatric supracondylar humerus fractures. J Am Acad Orthop Surg. 2012 May;20(5):320-7.
PubMed PMID: 22553104.
Herring JA, ed. Tachdjian’s Pediatric Orthopaedics. 4th ed. Philadelphia, PA: WB Saunders; 2008:1431-
1435.
Weinstein SL, ed. The Pediatric Spine. 2nd ed. Philadelphia, PA: Lippincott Williams & Wilkins;
2001:753-769.
Beaty JH, Kasser JR, eds. Rockwood & Wilkins’ Fractures in Children. 7th ed. Philadelphia, PA: Lippincott
Williams & Wilkins; 2010:405-427.

Question 80
A 2-week-old infant has an Ortolani positive right hip. She is placed in a Pavlik harness with her hips
flexed to 120 degrees. Three days after the harness is started, her parents notice that she is not extending
her right knee. What is the most likely reason for the change?

1. Septic right knee


2. Right femoral nerve palsy
3. Avascular necrosis of the right hip
4. Compartment syndrome of the right leg
5. Development of right hip Pavlik harness disease

PREFERRED RESPONSE: 2

RECOMMENDED READINGS
Murnaghan ML, Browne RH, Sucato DJ, Birch J. Femoral nerve palsy in Pavlik harness treatment for
developmental dysplasia of the hip. J Bone Joint Surg Am. 2011 Mar 2;93(5):493-9. PubMed PMID:
21368082.
Weinstein SL, Mubarak SJ, Wenger DR. Developmental hip dysplasia and dislocation: Part II. Instr
Course Lect. 2004;53:531-42. Review. PubMed PMID: 15116642.

© 2013 American Academy of Orthopaedic Surgeons 2013 Orthopaedic In-Training Examination
SECTION 7: Pediatric Orthopaedics • 137

Question 91
An otherwise healthy 5-year-old girl underwent closed reduction and percutaneous pin fixation of an
uncomplicated supracondylar fracture of the distal humerus. Four weeks later, radiographs show bone
healing and the 2 smooth Kirschner wires are removed. The patient has range of motion from 30 degrees
of flexion to 90 degrees of flexion. You inform her parents that

1. spontaneous play and gentle household chores will almost always allow a patient to regain full
elbow range of motion.
2. progressive static splinting should be initiated immediately to regain full elbow extension.
3. constant-force (spring loaded) splinting should be initiated immediately to regain full elbow
flexion.
4. formal therapy sessions emphasizing forearm rotation should begin immediately.
5. arthroscopic anterior capsular release is commonly indicated following a pediatric
supracondylar humerus fracture.

PREFERRED RESPONSE: 1

RECOMMENDED READINGS
Zionts LE, Woodson CJ, Manjra N, Zalavras C. Time of return of elbow motion after percutaneous
pinning of pediatric supracondylar humerus fractures. Clin Orthop Relat Res 2009 Aug;467(8):2007-10.
PubMed PMID: 19198963.
Lee S, Park MS, Chung CY, Chung CY, Kwon DG, Sung KH, Kim TW, Choi IH, Cho TJ, Yoo WJ, Lee
MS. Consensus and different perspectives on treatment of supracondylar fractures of the humerus in
children. Clin Orthop Surg 2012 Mar;4(1):91-7. PubMed PMID: 22379561.
Keppler P, Salem K, Schwarting B, Kinzl L. The effectiveness of physiotherapy after operative treatment
of supracondylar humeral fractures in children. J Pediatr Orthop 2005 May-Jun;25(3):314-6. PubMed
PMID: 15832145.

© 2013 American Academy of Orthopaedic Surgeons 2013 Orthopaedic In-Training Examination


138 • American Academy of Orthopaedic Surgeons

Figure 94a Figure 94b

Figure 94c Figure 94d

Figure 94e Figure 94f

Question 94
An 8-year-old girl underwent a drainage procedure of her left hip joint 3 days ago. She is being treated
with appropriate antibiotics. She remains febrile, has left anterior groin pain, keeps her left hip flexed at
about 20 degrees, and resists any left hip extension. T1 and T2 STIR images in the axial, coronal, and
sagittal projections are shown in Figures 94a through 94f. What conclusions can be drawn from these
findings?

1. These are expected MRI scan findings following drainage of acute septic arthritis of the hip.
2. In addition to the septic hip joint, there is osteomyelitis of the iliac wing and involvement of
several muscle groups including the psoas.
3. There is sufficient recurrent fluid in the hip joint itself to explain the girl’s continued discomfort.
4. The MRI scans confirm osteomyelitis of the femoral head.
5. The septic arthritis of the hip is now proven to be secondary to a sarcoma of the iliacus muscle.

PREFERRED RESPONSE: 2

© 2013 American Academy of Orthopaedic Surgeons 2013 Orthopaedic In-Training Examination


SECTION 7: Pediatric Orthopaedics • 139

RECOMMENDED READINGS
Stans AA. Osteomyelitis and septic arthritis. In: Morrissy RT, Weinstein SL, eds. Lovell and Winter’s
Pediatric Orthopaedics, 6th ed. Philadelphia, PA: Lippincott Williams & Wilkins; 2006:439-491.
Dala-Ali BM, Lloyd MA, Janipireddy SB, Atkinson HD. A case report of a septic hip secondary to a psoas
abscess. J Orthop Surg Res 2010 Sep 16;5:70. doi:10.1186/1749-799X-5-70. PubMed PMID: 20846379.
Ibu Yacob Y, Amine B, Hajjaj-Hassouni N. Bilateral septic arthritis of the hip with osteitis and psoas
abscess in a 17-year-old adolescent. J Pediatr Orthop B 2011 Jul;20(4):238-41. PubMed PMID: 21076342.

© 2013 American Academy of Orthopaedic Surgeons 2013 Orthopaedic In-Training Examination


140 • American Academy of Orthopaedic Surgeons

Figure 97a Figure 97b

Question 97
A 2-week-old infant has had decreased spontaneous motion of the right upper limb since birth. There
were no birth fractures and no infection is present. Persistent posture is shown in Figure 97a, and the
posture during a Moro response is shown in Figure 97b. Examination reveals full passive and active
motion of the neck. Based on these findings, the parents should be instructed to implement what actions
for the next 3 months?

1. Perform passive stretching for the neck; most important: rotate the head toward the involved
limb and extend the cervical spine.
2. Perform passive stretching for the involved shoulder and elbow; most important: move the
shoulder to extension/adduction and internal rotation.
3. Perform passive stretching for the involved shoulder and elbow; most important: move the
shoulder to elevation/abduction and external rotation.
4. Perform passive stretching for the involved hand; most important: move the fingers into
flexion at the interphalangeal joints and the thumb into adduction-flexion.
5. Refrain from performing any passive stretching; let the infant move spontaneously and
monitor for improvement.

PREFERRED RESPONSE: 3

RECOMMENDED READINGS
Waters PM. Update on management of pediatric brachial plexus palsy. J Pediatr Orthop 2005 Jan-
Feb;25(1):116-26.PubMed PMID: 15614072.
Waters PM. The upper limb. In: Morrissy RT, Weinstein SL, eds. Lovell and Winter’s Pediatric
Orthopaedics, ed. 6. Philadelphia, PA: Lippincott Williams & Wilkins; 2006:921-985.
Herring JA. Disorders of the upper extremity. In: Herring JA, ed. Tachdjian’s Pediatric Orthopaedics, 3rd
ed. Philadelphia, PA: WB Saunders; 2002:379-512.

© 2013 American Academy of Orthopaedic Surgeons 2013 Orthopaedic In-Training Examination
SECTION 7: Pediatric Orthopaedics • 141

Question 104
An 18-month-old boy is evaluated because he is not walking. He is found to have generalized hypotonia,
asymmetry in his muscle strength, with his proximal muscles weaker than his distal muscles, absent
deep tendon reflexes, and tongue fasciculations. What is the most appropriate next step in determining a
diagnosis?

1. Obtain a skeletal survey.


2. Schedule a muscle biopsy.
3. Electromyography and nerve conduction velocity studies
4. Referral for genetic testing of the survival motor neuron 1 gene
5. Referral for genetic testing to evaluate for trisomy 21

PREFERRED RESPONSE: 4

RECOMMENDED READINGS
Mesfin A, Sponseller PD, Leet AI. Spinal muscular atrophy: manifestations and management. J Am Acad
Orthop Surg. 2012 Jun;20(6):393-401. Review. PubMed PMID: 22661569.
Shapiro F, Specht L. The diagnosis and orthopaedic treatment of childhood spinal muscular
atrophy, peripheral neuropathy, Friedreich ataxia, and arthrogryposis. J Bone Joint Surg Am. 1993
Nov;75(11):1699-714. Review. PubMed PMID: 8245065.

© 2013 American Academy of Orthopaedic Surgeons 2013 Orthopaedic In-Training Examination


142 • American Academy of Orthopaedic Surgeons

Figure 114a Figure 114b Figure 114c

Question 114
A physician is called to the well-baby nursery to consult regarding an otherwise healthy female newborn;
the clinician states that “one of the baby’s legs is backwards.” Examination of the involved limb reveals
intact circulation and motor functions. Radiographs are shown in Figures 114a through 114c. Based on
these findings, what is the most appropriate initial treatment?

1. Casting or bracing with the knee in flexion


2. Casting or bracing with the knee in extension
3. Anterior knee release should be performed in about 6 months.
4. An MRI scan of the knee should be obtained before beginning any attempt at treatment.
5. Observation should be conducted for 3 weeks with the expectation of gradual spontaneous
improvement.

PREFERRED RESPONSE: 1

RECOMMENDED READINGS
Schoenecker PL, Rich MM. The lower extremity. In: Morrissy RT, Weinstein SL, eds. Lovell and Winter’s
Pediatric Orthopaedics, 6th ed. Philadelphia, PA: Lippincott Williams & Wilkins; 2006:1158-1211.
Abdelaziz TH, Samir S. Congenital dislocation of the knee: a protocol for management based on degree of
knee flexion. J Child Orthop 2011 Apr;5(2):143-9. PubMed PMID: 22468158.
Klingele KE, Stephens S. Management of ACL elongation in the surgical treatment of congenital knee
dislocation. Orthopaedics 2012 Jul;35(7):e1094-8. PubMed PMID: 22784907.

© 2013 American Academy of Orthopaedic Surgeons 2013 Orthopaedic In-Training Examination


SECTION 7: Pediatric Orthopaedics • 143

Figure 122a Figure 122b

Question 122
Figures 122a and 122b are the radiographs of a 3-year-old girl with a flexed interphalangeal joint of
the thumb on the left hand. Her parents notice that she has been unable to extend the interphalangeal
joint of her thumb for 18 months; however, she has no pain and is able to fully use her hand. The
parents deny any previous trauma to her hand. Examination reveals no tenderness, full motion of the
metacarpophalangeal joint, and passive extension of the interphalangeal joint to 25 degrees short of
neutral. A small volar mass is palpated at the level of the metatarsophalangeal joint. What is the most
appropriate next step?

1. Observation for 6 months


2. Release of the A1 pulley of the thumb
3. Repair of the flexor pollicis longus tendon
4. Extension osteotomy of the proximal phalanx
5. Physical therapy for improved extensor pollicis longus strength

PREFERRED RESPONSE: 2

RECOMMENDED READINGS
Shah AS, Bae DS. Management of pediatric trigger thumb and trigger finger. J Am Acad Orthop Surg.
2012 Apr;20(4):206-13. Review. PubMed PMID: 22474090.
Marek DJ, Fitoussi F, Bohn DC, Van Heest AE. Surgical release of the pediatric trigger thumb. J Hand
Surg Am. 2011 Apr;36(4):647-652.e2. PubMed PMID: 21463727.

© 2013 American Academy of Orthopaedic Surgeons 2013 Orthopaedic In-Training Examination


144 • American Academy of Orthopaedic Surgeons

Figure 129a Figure 129b

Video 129c (click to play) Video 129d (click to play)

Question 129
Figures 129a through 129b are the radiographs and MRI and CT scans of the lumbar spine of a 10-year-
old premenarchal girl who has back pain and scoliosis. What is the most likely etiology of her scoliosis?

1. Olisthetic
2. Idiopathic
3. Tethered cord
4. Myelodysplasia
5. Osteoid osteoma

PREFERRED RESPONSE: 5

© 2013 American Academy of Orthopaedic Surgeons 2013 Orthopaedic In-Training Examination


SECTION 7: Pediatric Orthopaedics • 145

RECOMMENDED READINGS
Burn SC, Ansorge O, Zeller R, Drake JM. Management of osteoblastoma and osteoid osteoma of the spine
in childhood. J Neurosurg Pediatr. 2009 Nov;4(5):434-8. PubMed PMID: 19877775.
Jayakumar P, Harish S, Nnadi C, Noordeen H, Saifuddin A. Symptomatic resolution of spinal osteoid
osteoma with conservative management: imaging correlation. Skeletal Radiol. 2007 Jun;36 Suppl 1:S72-6.
Epub 2006 Sep 12. PubMed PMID: 16967288.

Figure 138

Question 138
The posteroanterior radiograph seen in Figure 138 is of a 15-year-old girl who is evaluated for scoliosis.
She has a slightly elevated right shoulder, a moderate rib prominence of forward bend test, and normal
strength and reflexes in her lower extremities. She is 2 years postmenarchal. The radiograph reveals a
30-degree right thoracic and 25-degree left lumbar scoliosis. What is the most appropriate treatment?

1. Observation
2. Obtain a total spine MRI scan
3. Apical vertebral body stapling
4. Posterior spinal fusion from T4-L1
5. Use of a custom thoracolumbar orthosis for 23 hours per day

PREFERRED RESPONSE: 1

RECOMMENDED READINGS
Richards BS, Bernstein RM, D'Amato CR, Thompson GH. Standardization of criteria for adolescent
idiopathic scoliosis brace studies: SRS Committee on Bracing and Nonoperative Management. Spine
(Phila Pa 1976). 2005 Sep 15;30(18):2068-75; discussion 2076-7. Review. PubMed PMID: 16166897.
Negrini S, Minozzi S, Bettany-Saltikov J, Zaina F, Chockalingam N, Grivas TB, Kotwicki T, Maruyama T,
Romano M, Vasiliadis ES. Braces for idiopathic scoliosis in adolescents. Spine (Phila Pa 1976). 2010 Jun
1;35(13):1285-93. Review. PubMed PMID: 20461027.
© 2013 American Academy of Orthopaedic Surgeons 2013 Orthopaedic In-Training Examination
146 • American Academy of Orthopaedic Surgeons

Figure 143a Figure 143b

Figure 143c Figure 143d

Question 143
Figures 143a through 143d are the radiographs of a 13-year-old girl who sustained a knee injury during a
volleyball game. She has been otherwise asymptomatic, denies any previous musculoskeletal injury, and
has been playing competitive team sports for several years. Examination of her forearms reveals neutral
rotation position with restricted pronation-supination on the dominant right and complete absence of
pronation-supination on her left arm. What is the most appropriate intervention?

1. Early total elbow arthroplasty


2. Immediate physical therapy and progressive splinting
3. Resection of the radial heads after skeletal maturity
4. Immediately avoid sports that require repetitive use or impact loading of the upper limbs.
5. Rotational osteotomy to position the dominant hand in pronation and the nondominant hand in
supination.

PREFERRED RESPONSE: 3

© 2013 American Academy of Orthopaedic Surgeons 2013 Orthopaedic In-Training Examination


SECTION 7: Pediatric Orthopaedics • 147

RECOMMENDED READINGS
Waters PM. The upper limb. In: Morrissy RT, Weinstein SL, eds. Lovell and Winter’s Pediatric
Orthopaedics, 6th ed. Philadelphia, PA: Lippincott Williams & Wilkins; 2006:921-985.
Yamazaki H, Kato H. Open reduction of the radial head with ulnar osteotomy and annular ligament
reconstruction for bilateral congenital radial head dislocation: a case with long-term follow-up. J Hand
Surg Eur Vol 2007 Feb;32(1):93-7. PubMed PMID: 17129644.

© 2013 American Academy of Orthopaedic Surgeons 2013 Orthopaedic In-Training Examination


148 • American Academy of Orthopaedic Surgeons

Figure 151a Figure 151b Figure 151c

Question 151
A 15-year-old patient sustained the injuries shown in Figures 151a through 151c in a motor vehicle
collision and is otherwise medically stable. What is the most appropriate treatment?

1. A spica cast on the right and hanging-arm cast on the left


2. A right cephalomedullary femoral nail, right long-leg cast, and left hanging-arm cast
3. A right cephalomedullary femoral nail, right intramedullary tibial rod, and left hanging-arm
cast
4. Multiple screws across the right femoral neck, right long-leg cast, and left humeral
intramedullary rod
5. Multiple screws across the right femoral neck, right intramedullary tibial rod, and left humeral
intramedullary rod

PREFERRED RESPONSE: 5

RECOMMENDED READINGS
American Academy of Pediatrics Section on Orthopaedics; American Academy of Pediatrics Committee
on Pediatric Emergency Medicine; American Academy of Pediatrics Section on Critical Care; American
Academy of Pediatrics Section on Surgery; American Academy of Pediatrics Section on Transport
Medicine; American Academy of Pediatrics Committee on Pediatric Emergency Medicine; Pediatric
Orthopaedic Society of North America, Krug SE, Tuggle DW. Management of pediatric trauma.
Pediatrics. 2008 Apr;121(4):849-54. Review. PubMed PMID: 18381551.
Boardman MJ, Herman MJ, Buck B, Pizzutillo PD. Hip fractures in children. J Am Acad Orthop Surg.
2009 Mar;17(3):162-73. Review. PubMed PMID: 19264709.

© 2013 American Academy of Orthopaedic Surgeons 2013 Orthopaedic In-Training Examination


SECTION 7: Pediatric Orthopaedics • 149

Question 161
A 10-year-old gymnast fell from the parallel bars and sustained an elbow dislocation. It is appropriately
reduced in the emergency department acutely. What is the most appropriate treatment option?

1. Splinting for 10 days, then begin protected range of motion


2. Immediate range of motion and return to activities as tolerated
3. Long-arm cast for 4 weeks, then begin protected range of motion
4. Long-arm cast for 6 weeks, then splint for an additional 2 to 3 weeks
5. Surgical repair of the medial collateral ligament and long-arm cast for 4 weeks

PREFERRED RESPONSE: 1

RECOMMENDED READINGS
Andrews JR, Wilk KE, Groh G. Elbow rehabilitation. In: Brotzman SB, ed. Clinical Orthopaedic
Rehabilitation. Philadelphia, PA: Mosby-Yearbook; 1996:67-71.
Harrelson GL, Leaver-Dunn D. Elbow rehabilitation. In: Andrews JR, Harrelson GL, Wilk KE, eds.
Physical Rehabilitation of the Injured Athlete. 2nd ed. Philadelphia, PA: WB Saunders; 1998:554-588.
Protzman RR. Dislocation of the elbow joint. J Bone Joint Surg Am. 1978 Jun;60(4):539-41. PubMed
PMID: 670278.

© 2013 American Academy of Orthopaedic Surgeons 2013 Orthopaedic In-Training Examination


150 • American Academy of Orthopaedic Surgeons

Figure 168a Figure 168b

Question 168
Figure 168a is the initial radiograph and Figure 168b is the radiograph taken after a reduction was
performed on a 15-year-old girl who fell from a horse. She has had persistant pain and swelling in her
left shoulder since presentation. She has full motor function and sensation in her left arm and is 1.5 years
postmenarchal. The most appropriate next treatment step is

1. immobilization in a hanging-arm cast.


2. fracture immobilization with a figure-of-8 brace.
3. fracture fixation with an antegrade locked intramedullary nail.
4. open reduction and internal fixation with a blade plate.
5. closed reduction and percutaneous pinning.

PREFERRED RESPONSE: 5

RECOMMENDED READINGS
Hutchinson PH, Bae DS, Waters PM. Intramedullary nailing versus percutaneous pin fixation of pediatric
proximal humerus fractures: a comparison of complications and early radiographic results. J Pediatr
Orthop. 2011Sep;31(6):617-22. PubMed PMID: 21841434.
Dobbs MB, Luhmann SL, Gordon JE, Strecker WB, Schoenecker PL. Severely displaced proximal
humeral epiphyseal fractures. J Pediatr Orthop. 2003 Mar-Apr;23(2):208-15. PubMed PMID: 12604953.

© 2013 American Academy of Orthopaedic Surgeons 2013 Orthopaedic In-Training Examination


SECTION 7: Pediatric Orthopaedics • 151

Question 171
An 18-month-old boy has a bowleg deformity. His parents have noticed an increase in the appearance of
the deformity since he started walking at 12 months of age. Examination reveals full range of motion of
his hips and knees with a mild bowleg deformity. He walks with a 10-degree internal foot progression
angle and has no lateral knee thrust. What is the most appropriate next treatment step?

1. Continued observation
2. Start bilateral antivarus bracing
3. Recommend guided growth surgery
4. Recommend bilateral proximal tibial osteotomies
5. Obtain bilateral knee MRI scans to evaluate for medial physeal bars

PREFERRED RESPONSE: 1

RECOMMENDED READINGS
Sabharwal S, Zhao C. The hip-knee-ankle angle in children: reference values based on a full-length
standing radiograph. J Bone Joint Surg Am. 2009 Oct;91(10):2461-8. PubMed PMID: 19797583.
Davids JR, Blackhurst DW, Allen BL Jr. Clinical evaluation of bowed legs in children. J Pediatr Orthop B.
2000 Oct;9(4):278-84. PubMed PMID: 11143472.

© 2013 American Academy of Orthopaedic Surgeons 2013 Orthopaedic In-Training Examination


152 • American Academy of Orthopaedic Surgeons

Figure 177a Figure 177b

Question 177
Figures 177a and 177b are the radiographs of a 7-year-old boy with spastic cerebral palsy. He has
quadriparetic involvement and is unable to ambulate. He has very limited abduction, 30 degrees of flexion
contractures, and pain on abduction. Bilateral varus osteotomies are scheduled with acetabular procedures
to improve stability. Which type of acetabular osteotomy should be performed?

1. Dega iliac
2. Salter iliac
3. Pemberton iliac
4. Steele triple
5. Ganz or Bernese periacetabular

PREFERRED RESPONSE: 1

RECOMMENDED READINGS
Karlen JW, Skaggs DL, Ramachandran M, Kay RM. The Dega osteotomy: a versatile osteotomy
in the treatment of developmental and neuromuscular hip pathology. J Pediatr Orthop. 2009 Oct-
Nov;29(7):676-82. PubMed PMID: 20104144.
Chung CY, Choi IH, Cho TJ, Yoo WJ, Lee SH, Park MS. Morphometric changes in the acetabulum after
Dega osteotomy in patients with cerebral palsy. J Bone Joint Surg Br. 2008 Jan;90(1):88-91. PubMed
PMID: 18160506.

© 2013 American Academy of Orthopaedic Surgeons 2013 Orthopaedic In-Training Examination


SECTION 7: Pediatric Orthopaedics • 153

Figure 182

Question 182
A child sustained the injury shown in Figure 182 1 month ago. The parents did not follow up with
orthopaedic care. What is the most likely reason for parents to not follow through?

1. Concern about cost


2. Instructions were too difficult to follow.
3. The parents do not understand the plan.
4. The parents do not agree with the physician’s treatment plan.
5. Recommendations conflict with the parents’ personal beliefs

PREFERRED RESPONSE: 4

RECOMMENDED READINGS
Tongue JR, Otsuka NY. Patient-centered care: communication skills and cultural competence. In: Flynn
JM, ed. Orthopaedic Knowledge Update 10. Rosemont, IL: American Academy of Orthopaedic Surgeons;
2011:109-119.
Levinson W, Gorawara-Bhat R, Lamb J: A study of patient clues and physician responses in primary care
and surgical settings. JAMA 2000; 284(4): 1021-1027. PubMed PMID: 10944650.
The Commonwealth Fund. 2001 Health Care Quality Survey. disparities in health care. Washington, D.C.:
National Academy Press; 2003. http://www.commonwealthfund.org/Surveys/2001/2001-Health-Care-
Quality-Survey.aspx (Accessed 8/2/2013).

© 2013 American Academy of Orthopaedic Surgeons 2013 Orthopaedic In-Training Examination


154 • American Academy of Orthopaedic Surgeons

Figure 187a Figure 187b Figure 187c

Question 187
The radiographs in Figures 187a through 187c were obtained 10 months after closed reduction and
percutaneous pin fixation of a minimally displaced (Jacobs type 1) lateral condyle fracture of the distal
humerus in an otherwise healthy 4-year-old girl. She has regained full elbow and forearm range of motion
and has resumed all preinjury activities without pain or swelling at the elbow. What information can be
given to the parents about the current radiographic findings?

1. A spike of bone at the tip of the lateral condyle is frequently seen, is attributed to periosteal
displacement from the injury, and should be functionally insignificant.
2. An osteochondroma at the lateral condyle has resulted from displacement of a piece of the
physeal plate that occurred at the instant of injury.
3. The irregularity at the lateral condyle is sometimes known as a “Pelkan spur,” and implies an
underlying vitamin C deficiency.
4. The irregularity may have a significant cartilaginous component with capitellar damage, and
an MRI arthrogram is indicated.
5. The minimal cubitus varus is an infrequent sequela, but will almost always remodel and
develop into a normal carrying angle.

PREFERRED RESPONSE: 1

RECOMMENDED READINGS
Skak SV, Olsen SD, Smaabrekke A. Deformity after fracture of the lateral humeral condyle in children. J
Pediatr Orthop B 2001 Apr;10(2):142-52. PubMed PMID: 11360781.
Beaty JH, Kasser JR. The elbow: physeal fractures, apophyseal injuries of the distal humerus,
osteonecrosis of the trochlea, and T-condylar fractures. In: Beaty JH, Kasser JR, eds. Rockwood and
Wilkins’ Fractures in Children, 6th ed. Philadelphia, PA: Lippincott Williams & Wilkins; 2006:591-660.

© 2013 American Academy of Orthopaedic Surgeons 2013 Orthopaedic In-Training Examination


SECTION 7: Pediatric Orthopaedics • 155

Question 212
A 5-year-old boy sustained a vertical shear fracture of the pelvis, such that his left hemipelvis is displaced
upward 2 cm. A CT scan reveals widening of the pubic symphysis, mild external rotation of the left
hemipelvis, and a small avulsion fracture of the left sacrum. He has no abdominal or urologic injury.
What is the best next treatment step?

1. Ambulation nonweight-bearing on the left for 6 weeks


2. Closed reduction with a spica cast for 2 months
3. Closed reduction with a posterior sacroiliac screw fixation
4. Closed reduction with application of an anterior external fixator
5. Closed reduction with an anterior external fixator and posterior sacroiliac screw fixation

PREFERRED RESPONSE: 2

RECOMMENDED READINGS
Sink AL, Blaiser D. Fractures of the pelvis. In: Beaty JH, Kasser JR, eds. Rockwood & Wilkins’ Fractures
in Children. 7th ed. Philadelphia, PA: Lippincott Williams & Wilkins; 2010:743-768.
Banerjee S, Barry MJ, Paterson JM. Paediatric pelvic fractures: 10 years experience in a trauma centre.
Injury. 2009 Apr;40(4):410-3. Epub 2009 Feb 20. PubMed PMID: 19232592.

© 2013 American Academy of Orthopaedic Surgeons 2013 Orthopaedic In-Training Examination


156 • American Academy of Orthopaedic Surgeons

Figure 215a Figure 215b

Figure 215c Figure 215d Figure 215e

Question 215
An otherwise healthy adolescent girl was treated for left slipped capital femoral epiphysis. The
contralateral hip had not slipped, but was stabilized prophylactically with a single cannulated screw. The
implants were removed after 1 year. The pelvic radiographs (Figures 215a and 215b) and the MRI scans
of the hip that had not originally slipped (Figures 215c through 215e) were obtained 10 months after screw
removal (22 months after the original fixation). Which findings are shown in these studies?

1. Both hips are normal and no further assessments will be needed.


2. A neoplasm has developed in the femoral head on the unslipped side.
3. There is now increased risk for a slip in the hip and a new screw should be inserted.
4. Osteonecrosis has developed in the unslipped hip adjacent to the previous screw position.
5. The screw track in the bone has not filled spontaneously as expected and grafting should be
considered.

PREFERRED RESPONSE: 4

RECOMMENDED READINGS
Woelfle JV, Fraitzl CR, Reichel H, Nelitz M. The asymptomatic contralateral hip in unilateral slipped
capital femoral epiphysis: morbidity of prophylactic fixation. J Pediatr Orthop B 2012 May;21(3):226-9.
PubMed PMID: 22406770.
Lubicky JP. Chondrolysis and avascular necrosis: complications of slipped capital femoral epiphysis. J
Pediatr Orthop B 1996;5(3):162-7.PubMed PMID: 8866280.

© 2013 American Academy of Orthopaedic Surgeons 2013 Orthopaedic In-Training Examination


SECTION 7: Pediatric Orthopaedics • 157

Figure 224a Figure 224b

Question 224
Figures 224a and 224b are the radiographs of a skeletally mature child with cerebral palsy and worsening
difficulty with sitting who meets Gross Motor Function Classification System level IV criteria. Surgery is
planned. What is the most appropriate treatment option?

1. Anterior fusion from T12 to L5


2. Anterior release from L1 to L4 and posterior fusion from T2 to L5
3. Anterior release from L1 to L4 and posterior fusion from T12 to L5
4. Posterior fusion from T2 to the sacrum
5. Posterior fusion alone from T2 to L5 with Aponte osteotomies from L1 to L4

PREFERRED RESPONSE: 4

RECOMMENDED READINGS
Imrie MN, Yaszay B. Management of spinal deformity in cerebral palsy. Orthop Clin North Am. 2010
Oct;41(4):531-47. Review. PubMed PMID: 20868883.
McCarthy JJ, D'Andrea LP, Betz RR, Clements DH. Scoliosis in the child with cerebral palsy. J Am Acad
Orthop Surg. 2006 Jun;14(6):367-75. Review. PubMed PMID: 16757676.

© 2013 American Academy of Orthopaedic Surgeons 2013 Orthopaedic In-Training Examination


158 • American Academy of Orthopaedic Surgeons

Figure 239a Figure 239b

Question 239
A 4-year-old boy sustained the fracture seen in Figures 239a and 239b. Examination reveals normal
sensation to light touch throughout his left hand. Specific motor testing shows he is able to extend his
ipsilateral thumb fully and cross his fingers, but is unable to actively flex the distal interphalangeal joint of
his ipsilateral index finger. What is the most likely etiology of his motor deficit?

1. Neuropraxia of the radial nerve


2. Neuropraxia of the anterior intraosseous nerve
3. Development of compartment syndrome
4. Laceration of the flexor digitorum profundus of the index finger
5. Laceration of the flexor digitorum superficialis of the index finger

PREFERRED RESPONSE: 2

RECOMMENDED READINGS
Babal JC, Mehlman CT, Klein G. Nerve injuries associated with pediatric supracondylar humeral
fractures: a meta-analysis. J Pediatr Orthop. 2010 Apr-May;30(3):253-63. PubMed PMID: 20357592.
Bashyal RK, Chu JY, Schoenecker PL, Dobbs MB, Luhmann SJ, Gordon JE. Complications after pinning
of supracondylar distal humerus fractures. J Pediatr Orthop. 2009 Oct-Nov;29(7):704-8. PubMed PMID:
20104149.

© 2013 American Academy of Orthopaedic Surgeons 2013 Orthopaedic In-Training Examination


SECTION 7: Pediatric Orthopaedics • 159

Figure 251a Figure 251b Figure 251c Figure 251d

Figure 251e

Question 251
Figures 251a and 251b are the radiographs of a 2-year-old boy who is otherwise healthy. Clinical
photographs of the prosthesis and the child are shown in Figures 251c through 251e. What
recommendations should be given to the parents?

1. Surgical hip reduction and acetabuloplasty should be performed now, anticipating staged
femoral lengthening.
2. Physical therapy to regain ankle dorsiflexion should begin to maximize the success of
prosthetic fitting.
3. Epiphyseodesis of the proximal tibia should be performed now to minimize the functional
discrepancy between the tibia and fibula.
4. Ankle equinus in the current prosthesis is acceptable because this may facilitate substitution
for knee function in an eventual Van Ness rotationplasty.
5. Syme’s amputation and fitting with an above-knee prosthetic design is recommended.

PREFERRED RESPONSE: 4

© 2013 American Academy of Orthopaedic Surgeons 2013 Orthopaedic In-Training Examination


160 • American Academy of Orthopaedic Surgeons

RECOMMENDED READINGS
Morrissy RT, Giavedoni BJ, Coulter-O’Berry C. The child with a limb deficiency. In: Morrissy RT,
Weinstein SL, eds. Lovell and Winter’s Pediatric Orthopaedics. 6th ed. Philadelphia, PA: Lippincott
Williams & Wilkins, 2006:1329-1381.
Alman BA, Krajbich JI, Hubbard S. Proximal femoral focal deficiency: results of rotationplasty and Syme
amputation. J Bone Joint Surg Am 1995;77:1786. PubMed PMID: 8550656.
Kant P, Koh SH, Neumann V, Elliot C, Cotter D. Treatment of longitudinal deficiency affecting the femur:
comparing patient mobility and satisfaction outcomes of Syme amputation against extension prosthesis. J
Pediatr Orthop 2003 Mar-Apr;23(2):236-42. Pub Med PMID: 12604957.

Figure 262

Question 262
Figure 262 is the radiograph of a 15-year-old with cerebral palsy who meets Gross Motor Function
Classification System level V criteria. What is the most appropriate treatment?

1. Bilateral open reduction and pelvic osteotomies


2. Hip abduction bracing with administration of onabotulinum toxin A
3. Hip abduction bracing without administration of onabotulinum toxin A
4. No treatment or radiographic follow up is needed unless the patient is in pain
5. No treatment at this point, but close radiographic follow up is needed to monitor for
progression

PREFERRED RESPONSE: 4

RECOMMENDED READINGS
Bischof FM, Chirwa TF. Daily care activities and hip pain in non-ambulatory children and young adults
with cerebral palsy. J Pediatr Rehabil Med. 2011;4(3):219-23. PubMed PMID: 22207098.
Gamble JG, Rinsky LA, Bleck EE. Established hip dislocations in children with cerebral palsy. Clin
Orthop Relat Res. 1990 Apr;(253):90-9. Review. PubMed PMID: 2180606.

© 2013 American Academy of Orthopaedic Surgeons 2013 Orthopaedic In-Training Examination


SECTION 8: Shoulder and Elbow • 161

SECTION 8: Shoulder and Elbow

Question 12
A 7-month-old infant is evaluated after right brachial plexus birth palsy. Examination reveals scapular
winging and no active elbow flexion. Chest radiographs reveal an elevated right hemidiaphragm. The
best next step should be

1. free functional gracilis muscle transfer.


2. observation and re-examination in 3 months.
3. therapy to maintain shoulder internal rotation and elevation.
4. direct brachial plexus repair with sural nerve grafting if necessary.
5. musculocutaneous neurotization from ulnar/median donor fascicles.

PREFERRED RESPONSE: 5

RECOMMENDED READINGS
Sedain G, Sharma MS, Sharma BS, Mahapatra AK. Outcome after delayed Oberlin transfer in brachial
plexus injury. Neurosurgery. 2011 Oct;69(4):822-7; discussion 827-8. PubMed PMID: 21670719.
Toussaint CP, Zager EL. The double fascicular nerve transfer for restoration of elbow flexion.
Neurosurgery. 2011 Mar;68(1 Suppl Operative):64-7; discussion 67. Review. PubMed PMID: 21206298.
Mackinnon SE, Novak CB, Myckatyn TM, Tung TH. Results of reinnervation of the biceps and brachialis
muscles with a double fascicular transfer for elbow flexion. J Hand Surg Am. 2005 Sep;30(5):978-85.
PubMed PMID: 16182054.

Question 26
A patient is unable to actively externally rotate the shoulder when the arm is placed into 90 degrees of
abduction and neutral rotation. This finding is most consistent with a tear of the

1. biceps tendon.
2. isolated subscapularis.
3. isolated supraspinatus.
4. superior and anterior labrum.
5. infraspinatus and teres minor.

PREFERRED RESPONSE: 5

RECOMMENDED READINGS
Walch G, Boulahia A, Calderone S, Robinson AH. The 'dropping' and 'hornblower's' signs in evaluation of
rotator-cuff tears. J Bone Joint Surg Br. 1998 Jul;80(4):624-8. PubMed PMID: 9699824.
Hertel R, Ballmer FT, Lombert SM, Gerber C. Lag signs in the diagnosis of rotator cuff rupture. J
Shoulder Elbow Surg. 1996 Jul-Aug;5(4):307-13. PubMed PMID: 8872929.

© 2013 American Academy of Orthopaedic Surgeons 2013 Orthopaedic In-Training Examination


162 • American Academy of Orthopaedic Surgeons

Figure 36

Question 36
The innervation to the upper portion of the structure noted in Figure 36 arises directly from what aspect of
the brachial plexus?

1. Medial cord
2. Lateral cord
3. Posterior cord
4. Upper trunk
5. C5-C7 roots

PREFERRED RESPONSE: 3

RECOMMENDED READINGS
Lyons RP, Green A. Subscapularis tendon tears. J Am Acad Orthop Surg. 2005 Sep;13(5):353-63. Review.
PubMed PMID: 16148361.
Kasper JC, Itamura JM, Tibone JE, Levin SL, Stevanovic MV. Human cadaveric study of subscapularis
muscle innervation and guidelines to prevent denervation. J Shoulder Elbow Surg. 2008 Jul-
Aug;17(4):659-62. Epub 2008 May 12. PubMed PMID: 18472282.

© 2013 American Academy of Orthopaedic Surgeons 2013 Orthopaedic In-Training Examination


SECTION 8: Shoulder and Elbow • 163

Figure 41a Figure 41b Figure 41c

Question 41
Figures 41a through 41c are the radiograph and MRI scans of a 76-year-old woman who has intractable
left shoulder pain. She was given 2 cortisone injections and oral pain medication without experiencing
lasting relief. Examination reveals 60 degrees of active forward elevation (120 degrees passively), 30
degrees of external rotation lag, and a positive Hornblower sign. Pain relief and improved functionality
will most likely be achieved with

1. continued nonsurgical treatment.


2. hemiarthroplasty with partial rotator cuff repair.
3. reverse total shoulder arthroplasty with latissimus dorsi transfer.
4. rotator cuff repair without acromioplasty, preserving the coracoacromial ligament.
5. limited-goals debridement of the rotator cuff and glenohumeral joint without rotator cuff
repair.

PREFERRED RESPONSE: 3

RECOMMENDED READINGS
Gerber C, Pennington SD, Lingenfelter EJ, Sukthankar A. Reverse Delta1III total shoulder replacement
combined with latissimus dorsi transfer. A preliminary report. J Bone Joint Surg Am. 2007 May;89(5):940-
7. PubMed PMID: 17473129.
Boileau P, Rumian AP, Zumstein MA. Reversed shoulder arthroplasty with modified L'Episcopo
for combined loss of active elevation and external rotation. J Shoulder Elbow Surg. 2010 Mar;19(2
Suppl):20-30. PubMed PMID: 20188265.

© 2013 American Academy of Orthopaedic Surgeons 2013 Orthopaedic In-Training Examination


164 • American Academy of Orthopaedic Surgeons

Figure 53

Question 53
Figure 53 is the CT scan of a 38-year-old woman who has pain with movement of her right arm and
shortness of breath after an assault. She is evaluated in the emergency department and her shoulder
radiograph findings are normal. The physician should recommend

1. a chest tube.
2. a sling and outpatient follow up.
3. closed reduction.
4. incision and drainage.
5. an MRI scan of the shoulder.

PREFERRED RESPONSE: 3

RECOMMENDED READINGS
Groh GI, Wirth MA. Management of traumatic sternoclavicular joint injuries. J Am Acad Orthop Surg.
2011 Jan;19(1):1-7. Review. PubMed PMID: 21205762.
Glass ER, Thompson JD, Cole PA, Gause TM II, Altman GT. Treatment of sternoclavicular joint
dislocations: a systematic review of 251 dislocations in 24 case series. J Trauma. 2011 May;70(5):1294-8.
Review. PubMed PMID: 21610444.

© 2013 American Academy of Orthopaedic Surgeons 2013 Orthopaedic In-Training Examination


SECTION 8: Shoulder and Elbow • 165

Question 66
A 40-year-old man with a history of a nondisplaced radial head fracture was initially treated with cast
immobilization for 3 weeks followed by a course of physical therapy. Six months later, he has limited
elbow range of motion. Examination reveals he lacks 30 degrees of extension and has flexion to only 90
degrees. To restore flexion, which structure must be released?

1. Triceps tendon
2. Anterior capsule
3. Ulnar part of the lateral collateral ligament
4. Anterior bundle of the medial collateral ligament
5. Posteromedial bundle of the medial collateral ligament

PREFERRED RESPONSE: 5

RECOMMENDED READINGS
Wada T, Ishii S, Usui M, Miyano S. The medial approach for operative release of post-traumatic
contracture of the elbow. J Bone Joint Surg Br. 2000 Jan;82(1):68-73. PubMed PMID: 10697317.
Morrey BF, An KN. Articular and ligamentous contributions to the stability of the elbow joint. Am J
Sports Med. 1983 Sep-Oct;11(5):315-9. PubMed PMID: 6638246.
Regan WD, Korinek SL, Morrey BF, An KN. Biomechanical study of ligaments around the elbow joint.
Clin Orthop Relat Res. 1991 Oct;(271):170-9. PubMed PMID: 1914292.

© 2013 American Academy of Orthopaedic Surgeons 2013 Orthopaedic In-Training Examination


166 • American Academy of Orthopaedic Surgeons

Figure 89a Figure 89b

Question 89
Figures 89a and 89b are the radiographs of an 18-year-old woman who has had elbow pain after falling
on an outstretched hand. She is evaluated 5 days after the injury. Examination reveals the wrist is normal
and her elbow has a limited arc of motion of 30 to 90 degrees of flexion/extension and 20 to 20 degrees
of pronation and supination, with tenderness isolated to the lateral side of the elbow. What is the most
appropriate treatment option?

1. Cast for 2 weeks


2. Initiate mobilization
3. Radial head excision
4. Radial head replacement
5. Open reduction and internal fixation

PREFERRED RESPONSE: 2

RECOMMENDED READINGS
Paschos NK, Mitsionis GI, Vasiliadis HS, Georgoulis AD. Comparison of early mobilization protocols in
radial head fractures. A prospective randomized controlled study. The effect of fracture characteristics on
outcome. J Orthop Trauma. 2012 Jul 11. PubMed PMID: 22576643.
Tejwani NC, Mehta H. Fractures of the radial head and neck: current concepts in management. J Am Acad
Orthop Surg. 2007 Jul;15(7):380-7. Review. PubMed PMID: 17602027.

© 2013 American Academy of Orthopaedic Surgeons 2013 Orthopaedic In-Training Examination


SECTION 8: Shoulder and Elbow • 167

Figure 101a Figure 101b

Question 101
Figures 101a and 101b are the radiographs of a 50-year-old man who has difficulty with overhead
work following a superior labrum anterior to posterior (SLAP) repair 12 months ago. He had no early
postsurgical complications and was in therapy for 9 months after surgery. Examination of the shoulder
reveals 110 and 45 degrees of active elevation and active external rotation with his arm at his side,
respectively. His passive range of motion is symmetric to his active range of motion. What is the best
treatment option?

1. Arthroplasty
2. Acromioplasty
3. Continue therapy
4. Revision SLAP repair
5. Arthroscopic capsular release

PREFERRED RESPONSE: 5

RECOMMENDED READINGS
Katz LM, Hsu S, Miller SL, Richmond JC, Khetia E, Kohli N, Curtis AS. Poor outcomes after SLAP
repair: descriptive analysis and prognosis. Arthroscopy. 2009 Aug;25(8):849-55. Erratum in: Arthroscopy.
2009 Nov;25(11):1361. Ketia, Eric [corrected to Khetia, Eric]. PubMed PMID: 19664504.
Holloway GB, Schenk T, Williams GR, Ramsey ML, Iannotti JP. Arthroscopic capsular release for the
treatment of refractory postoperative or post-fracture shoulder stiffness. J Bone Joint Surg Am. 2001
Nov;83-A(11):1682-7. PubMed PMID: 11701791.

© 2013 American Academy of Orthopaedic Surgeons 2013 Orthopaedic In-Training Examination


168 • American Academy of Orthopaedic Surgeons

Figure 118a Figure 118b

Video 118c (click to play) Video 118d (click to play)

Question 118
Figures 118a through 118d are the radiographs and selected CT sequences of a 24-year-old man who
had arthroscopic Bankart repair 1 year ago after a traumatic dislocation. He has recurrent instability
and denies any recent trauma. He has instability with even trivial activities and states his shoulder was
“never really stable” after his surgery. He has been to the emergency department on multiple occasions
for manipulative reduction. Examination reveals good muscle tone and bulk around the shoulder. He has
no external rotation weakness, and his abdominal compression test is normal. He has a markedly positive
anterior apprehension sign in the 90-degree abducted, externally rotated position and in midabduction.
The sulcus sign is negative and there is no evidence of posterior instability. What is the best treatment
option?

1. Physical therapy
2. Arthroscopic rotator cuff repair
3. Revision arthroscopic Bankart repair
4. Bone grafting of the Hill-Sachs lesion
5. Transfer of the coracoid process to the anterior glenoid neck

PREFERRED RESPONSE: 5
© 2013 American Academy of Orthopaedic Surgeons 2013 Orthopaedic In-Training Examination
SECTION 8: Shoulder and Elbow • 169

RECOMMENDED READINGS
Burkhart SS, De Beer JF. Traumatic glenohumeral bone defects and their relationship to failure of
arthroscopic Bankart repairs: significance of the inverted-pear glenoid and the humeral engaging Hill-
Sachs lesion. Arthroscopy. 2000 Oct;16(7):677-94. PubMed PMID: 11027751.
Hantes ME, Venouziou A, Bargiotas KA, Metafratzi Z, Karantanas A, Malizos KN. Repair of an
anteroinferior glenoid defect by the latarjet procedure: quantitative assessment of the repair by computed
tomography. Arthroscopy. 2010 Aug;26(8):1021-6. PubMed PMID: 20678698.

Question 131
A 45-year-old woman has elbow stiffness 3 months after treatment of an elbow dislocation consisting of
self-directed exercises. Examination reveals that her elbow is stable. Range-of-motion testing reveals a
35-degree flexion contracture, full flexion, and 80 degrees of both pronation and supination. What is the
best next treatment step?

1. Hinged elbow brace


2. Open contracture release
3. Arthroscopic debridement and release
4. Supervised therapy with splinting
5. Examination under anesthesia and manipulation
PREFERRED RESPONSE: 4
RECOMMENDED READINGS
Doornberg JN, Ring D, Jupiter JB. Static progressive splinting for posttraumatic elbow stiffness. J Orthop
Trauma. 2006 Jul;20(6):400-4. PubMed PMID: 16825965.
Lindenhovius AL, Doornberg JN, Brouwer KM, Jupiter JB, Mudgal CS, Ring D. A prospective
randomized controlled trial of dynamic versus static progressive elbow splinting for posttraumatic elbow
stiffness. J Bone Joint Surg Am. 2012 Apr 18;94(8):694-700. PubMed PMID: 22517385.

Question 148
A 22-year-old collegiate baseball pitcher has had posterior shoulder pain with decreased throwing velocity
and accuracy over the past several months. Examination of the abducted shoulder in the supine position
reveals 120 degrees of external rotation, 40 degrees of internal rotation on the throwing side, 100 degrees
of external rotation, and 70 degrees of internal rotation on the nonthrowing side. The remainder of the
clinical examination is unremarkable. An MRI scan shows a small partial articular-sided infraspinatus
tear. Initial treatment should consist of

1. arthroscopic rotator cuff repair.


2. arthroscopic anterior capsulorrhaphy.
3. arthroscopic selective posterior capsular release.
4. selective posterior rotator cuff strengthening.
5. posterior capsular stretching with scapular stabilization.
PREFERRED RESPONSE: 5
© 2013 American Academy of Orthopaedic Surgeons 2013 Orthopaedic In-Training Examination
170 • American Academy of Orthopaedic Surgeons

RECOMMENDED READINGS
Heyworth BE, Williams RJ 3rd. Internal impingement of the shoulder. Am J Sports Med. 2009
May;37(5):1024-37. Epub 2008 Dec 4. PubMed PMID: 19059895.
Tyler TF, Nicholas SJ, Lee SJ, Mullaney M, McHugh MP. Correction of posterior shoulder tightness
is associated with symptom resolution in patients with internal impingement. Am J Sports Med. 2010
Jan;38(1):114-9. Epub 2009 Dec 4. PubMed PMID: 19966099.

Figure 166

Question 166
Which factor is associated with an increased risk for the complication shown in Figure 166 following
reverse total shoulder arthroplasty?

1. Failed previous arthroplasty


2. Anterosuperior approach
3. Complete repair of the subscapularis
4. Inferior inclination of the glenosphere baseplate
5. Humeral stem placement in 10 degrees’ retroversion

PREFERRED RESPONSE: 1

RECOMMENDED READINGS
Trappey GJ 4th, O'Connor DP, Edwards TB. What are the instability and infection rates after reverse
shoulder arthroplasty? Clin Orthop Relat Res. 2011 Sep;469(9):2505-11. PubMed PMID: 21104354.
Favre P, Sussmann PS, Gerber C. The effect of component positioning on intrinsic stability of the reverse
shoulder arthroplasty. J Shoulder Elbow Surg. 2010 Jun;19(4):550-6. Epub 2010 Mar 23. PubMed PMID:
20335055.
Edwards TB, Williams MD, Labriola JE, Elkousy HA, Gartsman GM, O'Connor DP. Subscapularis
insufficiency and the risk of shoulder dislocation after reverse shoulder arthroplasty. J Shoulder Elbow
Surg. 2009 Nov-Dec;18(6):892-6. Epub 2009 Mar 17. PubMed PMID: 19282204.

© 2013 American Academy of Orthopaedic Surgeons 2013 Orthopaedic In-Training Examination


SECTION 8: Shoulder and Elbow • 171

Figure 174a Figure 174b

Video 174c (click to play) Video 174d (click to play)

Question 174
Figures 174a through 174d are the radiographs and selected MRI sequences of a 35-year-old man with
a history of alcoholism. He has right shoulder pain that has been progressively worsening over the past
several years. Examination reveals active forward elevation of 150 degrees, external rotation of 50
degrees with his arm by his side, and internal rotation to the T-12 vertebral level. He had a cortisone
injection, but experienced no relief. What is the best treatment option?

1. Hemiarthroplasty
2. Core decompression
3. Arthroscopy and capsular release
4. Intra-articular hyaluronate injections
5. Physical therapy and nonsteroidal anti-inflammatory medication

PREFERRED RESPONSE: 2

© 2013 American Academy of Orthopaedic Surgeons 2013 Orthopaedic In-Training Examination


172 • American Academy of Orthopaedic Surgeons

RECOMMENDED READINGS
Smith RG, Sperling JW, Cofield RH, Hattrup SJ, Schleck CD. Shoulder hemiarthroplasty for steroid-
associated osteonecrosis. J Shoulder Elbow Surg. 2008 Sep-Oct;17(5):685-8. Epub 2008 Jun 24. PubMed
PMID: 18572421.
LaPorte DM, Mont MA, Mohan V, Pierre-Jacques H, Jones LC, Hungerford DS. Osteonecrosis of the
humeral head treated by core decompression. Clin Orthop Relat Res. 1998 Oct;(355):254-60. PubMed
PMID: 9917611.

Figure 194a Figure 194b

Question 194
Figures 194a and 194b are the radiographs of a 59-year-old right-hand-dominant woman who has pain
in her dominant shoulder following a fall 1 day ago. Examination reveals tenderness over the proximal
humerus and ecchymosis about the midarm. She is distally neurovascularly intact. What is the most
appropriate treatment option?

1. Hemiarthroplasty
2. Reverse total shoulder arthroplasty
3. Open reduction and internal fixation
4. Sling immobilization for 6 weeks followed by passive range of motion
5. Symptomatic sling use followed by early active range-of-motion exercises

PREFERRED RESPONSE: 3

RECOMMENDED READINGS
Solberg BD, Moon CN, Franco DP, Paiement GD. Surgical treatment of three and four-part proximal
humeral fractures. J Bone Joint Surg Am. 2009 Jul;91(7):1689-97. PubMed PMID: 19571092.
Nho SJ, Brophy RH, Barker JU, Cornell CN, MacGillivray JD. Innovations in the management of
displaced proximal humerus fractures. J Am Acad Orthop Surg. 2007 Jan;15(1):12-26. Review. PubMed
PMID: 17213379.

© 2013 American Academy of Orthopaedic Surgeons 2013 Orthopaedic In-Training Examination


SECTION 8: Shoulder and Elbow • 173

Question 219
A woman who underwent an unconstrained total shoulder arthroplasty fell 6 weeks after surgery. She now
has a documented anterior shoulder dislocation and undergoes closed reduction. All 3 heads of the deltoid
contract, passive elevation is limited by pain to 30 degrees, and her internal rotation is to the side and
her passive external rotation is 80 degrees. At the 2-week postsurgical visit, her elevation is 60 degrees,
external rotation is 10 degrees, and internal rotation is to the side. Radiographs reveal no loosening,
fractures, or dislocations. Further evaluation should consist of

1. an indium scan.
2. C-reactive protein.
3. electromyography.
4. aspiration of the shoulder.
5. ultrasound of the shoulder.

PREFERRED RESPONSE: 5

RECOMMENDED READINGS
Miller BS, Joseph TA, Noonan TJ, Horan MP, Hawkins RJ. Rupture of the subscapularis tendon
after shoulder arthroplasty: diagnosis, treatment, and outcome. J Shoulder Elbow Surg. 2005 Sep-
Oct;14(5):492-6. PubMed PMID: 16194740.
Westhoff B, Wild A, Werner A, Schneider T, Kahl V, Krauspe R. The value of ultrasound after shoulder
arthroplasty. Skeletal Radiol. 2002 Dec;31(12):695- 701. Epub 2002 Oct 25. PubMed PMID: 12483430.

Question 232
A fall onto an outstretched arm places an axial load on the wrist and forearm. What other combination of
forces at the elbow leads to a terrible triad of radial head fracture, coronoid fracture, and lateral collateral
ligament injury?

1. Forearm supination and varus thrust


2. Forearm pronation and varus thrust
3. Forearm supination and valgus thrust
4. Forearm pronation and valgus thrust
5. No rotational forces, only axial loading

PREFERRED RESPONSE: 3

RECOMMENDED READINGS
O'Driscoll SW, Bell DF, Morrey BF. Posterolateral rotatory instability of the elbow. J Bone Joint Surg Am.
1991 Mar;73(3):440-6. PubMed PMID: 2002081.
O'Driscoll SW, Morrey BF, Korinek S, An KN. Elbow subluxation and dislocation. A spectrum of
instability. Clin Orthop Relat Res. 1992 Jul;(280):186-97. PubMed PMID: 1611741.

© 2013 American Academy of Orthopaedic Surgeons 2013 Orthopaedic In-Training Examination


174 • American Academy of Orthopaedic Surgeons

Figure 243a Figure 243b Figure 243c

Figure 243d

Question 243
Figures 243a through 243d are the plain radiographs and selected sequences from an MRI scan of a
52-year-old man with a history of prior arthroscopic rotator cuff repair. He has persistent pain and limited
range of motion. Examination reveals no deltoid atrophy, but significant atrophy of the infraspinatus.
He has active overhead elevation of 140 degrees with a painful arc and significant weakness of shoulder
external rotation both with his arm by his side and in abduction. Both the lift-off and abdominal
compression tests are within defined limits. What is the best treatment option?

1. Hemiarthroplasty
2. Superior labral repair
3. Latissimus dorsi transfer
4. Reverse total shoulder arthroplasty
5. Revision arthroscopic rotator cuff repair

PREFERRED RESPONSE: 3

RECOMMENDED READINGS
Gerber C, Maquieira G, Espinosa N. Latissimus dorsi transfer for the treatment of irreparable rotator cuff
tears. J Bone Joint Surg Am. 2006 Jan;88(1):113-20. PubMed PMID: 16391256.
Iannotti JP, Hennigan S, Herzog R, Kella S, Kelley M, Leggin B, Williams GR. Latissimus dorsi tendon
transfer for irreparable posterosuperior rotator cuff tears. Factors affecting outcome. J Bone Joint Surg
Am. 2006 Feb;88(2):342-8. PubMed PMID: 16452746.

© 2013 American Academy of Orthopaedic Surgeons 2013 Orthopaedic In-Training Examination


SECTION 8: Shoulder and Elbow • 175

Question 246
A 40-year-old man who had an intra-articular supracondylar humerus fracture was treated by bicolumnar
plating 1 year ago. He has pain and dysfunction of the elbow. His range of motion is from 30 to 90
degrees of flexion-extension and 80 to 80 degrees of pronation-supination. The mid arc of motion is pain
free. He has medial-sided elbow pain, reproduced with forced elbow flexion. Radiographs reveal a healed
fracture, no hardware breakage, and mild joint space incongruency with a well-maintained joint space.
What is the most appropriate treatment?

1. Hardware removal
2. Arthroscopic debridement
3. Osteotomy and reconstruction
4. Manipulation under anesthesia
5. Ulnar nerve decompression and capsular release

PREFERRED RESPONSE: 5

RECOMMENDED READINGS
Park MJ, Chang MJ, Lee YB, Kang HJ. Surgical release for posttraumatic loss of elbow flexion. J Bone
Joint Surg Am. 2010 Nov 17;92(16):2692-9. PubMed PMID: 21084579.
Williams BG, Sotereanos DG, Baratz ME, Jarrett CD, Venouziou AI, Miller MC. The contracted elbow: is
ulnar nerve release necessary? J Shoulder Elbow Surg. 2012 Jun 26. PubMed PMID: 22743068.

© 2013 American Academy of Orthopaedic Surgeons 2013 Orthopaedic In-Training Examination


176 • American Academy of Orthopaedic Surgeons

Figure 255a Figure 255b Figure 255c

Question 255
Figures 255a through 255c are the radiographs and MRI scan of a 73-year-old man who has severe pain
and functional disability of the right shoulder despite receiving several cortisone injections and physical
therapy. Examination reveals restricted shoulder range of motion in forward elevation and both internal
and external rotation. There is moderately diminished strength and pain with resisted forward elevation.
What is the best treatment option?

1. Reverse total shoulder arthroplasty


2. Unconstrained total shoulder arthroplasty
3. Hemiarthroplasty with biologic glenoid resurfacing
4. Arthroscopic subacromial decompression
5. Arthroscopic capsular release with manipulation under anesthesia

PREFERRED RESPONSE: 2

RECOMMENDED READINGS
Radnay CS, Setter KJ, Chambers L, Levine WN, Bigliani LU, Ahmad CS. Total shoulder replacement
compared with humeral head replacement for the treatment of primary glenohumeral osteoarthritis: a
systematic review. J Shoulder Elbow Surg. 2007 Jul-Aug;16(4):396-402. Epub 2007 Jun 20. Review.
PubMed PMID: 17582789.
Izquierdo R, Voloshin I, Edwards S, Freehill MQ, Stanwood W, Wiater JM, Watters WC 3rd, Goldberg
MJ, Keith M, Turkelson CM, Wies JL, Anderson S, Boyer K, Raymond L, Sluka P; American Academy
of Orthopedic Surgeons. Treatment of glenohumeral osteoarthritis. J Am Acad Orthop Surg. 2010
Jun;18(6):375-82. PubMedPMID: 20511443.

© 2013 American Academy of Orthopaedic Surgeons 2013 Orthopaedic In-Training Examination


SECTION 8: Shoulder and Elbow • 177

Figure 266

Question 266
Figure 266 is the anteroposterior radiograph of a 6-year-old boy who sustained an injury to his left elbow
after a fall. Examination of his elbow reveals intact skin. There is tenderness over the radial head, but
he is nontender elsewhere, including his wrist. His distal neurovascular examination is unremarkable. A
closed reduction was attempted; however, there was no improvement in position. What is the best next
treatment option?

1. Radial head arthroplasty


2. Long-arm cast for 4 weeks
3. Open reduction and plate fixation
4. Immediate range of motion and physical therapy
5. Percutaneous Kirschner wire-assisted reduction and casting

PREFERRED RESPONSE: 5

RECOMMENDED READINGS
Ursei M, Sales de Gauzy J, Knorr J, Abid A, Darodes P, Cahuzac JP. Surgical treatment of radial neck
fractures in children by intramedullary pinning. Acta Orthop Belg. 2006 Apr;72(2):131-7. PubMed PMID:
16768254.
Eilert RE, Erickson MA. Fractures of the proximal radius and ulna. In: Beaty JH, Kasser JR, eds.
Rockwood and Wilkins’ Fractures in Children. 6th ed. Philadelphia, PA: Lippincott Williams & Wilkins;
2006:443-490.

© 2013 American Academy of Orthopaedic Surgeons 2013 Orthopaedic In-Training Examination


178 • American Academy of Orthopaedic Surgeons

SECTION 9: Spine

Figure 7a Figure 7b Figure 7c

Question 7
Figures 7a through 7c are the radiograph and MRI scans of a 72-year-old woman who has had back and
leg pain for 3 months. Her pain is worse with prolonged walking and relieved with bending forward.
Examination reveals normal strength and sensation in her legs with intact pedal pulses. She has persistent
pain despite physical therapy, medications, and epidural injections. What is the most appropriate
treatment option?

1. Laminectomy
2. Laminectomy and instrumented fusion
3. Laminectomy and uninstrumented fusion
4. Endovascular aortic bypass
5. Anterior lumbar interbody fusion

PREFERRED RESPONSE: 2

RECOMMENDED READINGS
Weinstein JN, Lurie JD, Tosteson TD, Zhao W, Blood EA, Tosteson AN, Birkmeyer N, Herkowitz H,
Longley M, Lenke L, Emery S, Hu SS. Surgical compared with nonoperative treatment for lumbar
degenerative spondylolisthesis. Four-year results in the Spine Patient Outcomes Research Trial (SPORT)
randomized and observational cohorts. J Bone Joint Surg Am. 2009 Jun;91(6):1295-304. PubMed PMID:
19487505.
Abdu WA, Lurie JD, Spratt KF, Tosteson AN, Zhao W, Tosteson TD, Herkowitz H, Longely M, Boden
SD, Emery S, Weinstein JN. Degenerative spondylolisthesis: does fusion method influence outcome?
Four-year results of the spine patient outcomes research trial. Spine (Phila Pa 1976). 2009 Oct
1;34(21):2351-60. PubMed PMID: 19755935.

© 2013 American Academy of Orthopaedic Surgeons 2013 Orthopaedic In-Training Examination


SECTION 9: Spine • 179

Figure 19

Question 19
Figure 19 is the T2-weighted MRI scan of a 25-year-old man who is seen in the emergency department
after falling off of a roof. Examination revealed he has 3/5 strength in his bicep muscles bilaterally but no
motor or sensory function in his hands. For this type of injury, early decompression within 24 hours gives
what advantage?

1. Reduced mortality
2. Improved neurologic outcomes
3. Lower risk for pulmonary embolus
4. Decreased incidence of hospital readmission
5. Earlier discharge to a skilled rehabilitation facility

PREFERRED RESPONSE: 2

RECOMMENDED READINGS
Fehlings MG, Vaccaro A, Wilson JR, Singh A, W Cadotte D, Harrop JS, Aarabi B, Shaffrey C, Dvorak
M, Fisher C, Arnold P, Massicotte EM, Lewis S, Rampersaud R. Early versus delayed decompression for
traumatic cervical spinal cord injury: results of the Surgical Timing in Acute Spinal Cord Injury Study
(STASCIS). PLoS One. 2012;7(2):e32037. Epub 2012 Feb 23. PubMed PMID: 22384132.
Fehlings MG, Rabin D, Sears W, Cadotte DW, Aarabi B. Current practice in the timing of surgical
intervention in spinal cord injury. Spine (Phila Pa 1976). 2010 Oct 1;35(21 Suppl):S166-73. Review.
PubMed PMID: 20881458.

© 2013 American Academy of Orthopaedic Surgeons 2013 Orthopaedic In-Training Examination


180 • American Academy of Orthopaedic Surgeons

Figure 27a Figure 27b

Question 27
Figures 27a and 27b are the MRI scans of a 31-year-old woman with low-back and left leg pain radiating
into her posterior thigh and calf for 2 weeks. Examination reveals a positive straight-leg raise, normal
strength, and normal sensation in the lower extremities. What is the most appropriate treatment option?

1. Nonsurgical care
2. Microdiscectomy
3. Subtotal discectomy
4. Anterior decompression and fusion
5. Posterior decompression and fusion

PREFERRED RESPONSE: 1

RECOMMENDED READINGS
Weinstein JN, Lurie JD, Tosteson TD, Tosteson AN, Blood EA, Abdu WA, Herkowitz H, Hilibrand
A, Albert T, Fischgrund J. Surgical versus nonoperative treatment for lumbar disc herniation: four-
year results for the Spine Patient Outcomes Research Trial (SPORT). Spine (Phila Pa 1976). 2008 Dec
1;33(25):2789-800. PubMed PMID: 19018250.
Anderson PA, McCormick PC, Angevine PD. Randomized controlled trials of the treatment of lumbar
disk herniation: 1983-2007. J Am Acad Orthop Surg. 2008 Oct;16(10):566-73. Review. PubMed PMID:
18832600.

© 2013 American Academy of Orthopaedic Surgeons 2013 Orthopaedic In-Training Examination


SECTION 9: Spine • 181

Question 33
A physician shows interest in determining the evidence base for use of a specific interbody fusion
technique in the treatment of lumbar degenerative disc disease. A search of the literature reveals 4 studies
that retrospectively reviewed outcomes for series comprising fewer than 20 patients each. Another study
retrospectively compared results of the interbody fusion technique to posterolateral fusion. All of the
studies reported satisfactory outcomes for the interbody fusion technique, while the comparative study
found interbody fusion to be superior to posterolateral fusion. The quality of evidence supporting the use
of the interbody fusion technique would be graded as

1. B (fair-quality evidence), attributable to the fact that a single level III study supports use of the
interbody fusion technique
2. B (fair-quality evidence), attributable to the fact that multiple level IV studies and a single
level III study support its use
3. C (poor-quality evidence), attributable to the fact that multiple level IV studies and a single
level III study support its use
4. C (poor-quality evidence), attributable to the fact that multiple level V studies and a single
level III study support its use
5. I (no evidence), attributable to the fact that the studies found in the literature are of insufficient
quality to allow recommendation in support of the technique

PREFERRED RESPONSE: 3

RECOMMENDED READINGS
Wright JG, Swiontkowski MF, Heckman JD. Introducing levels of evidence to the journal. J Bone Joint
Surg Am. 2003 Jan;85-A(1):1-3. PubMed PMID: 12533564.
Wright JG. A practical guide to assigning levels of evidence. J Bone Joint Surg Am. 2007 May;89(5):1128-
30. PubMed PMID: 17473152.
Okike K, Kocher MS. Evidence-based orthopaedics: levels of evidence and guidelines in orthopaedic
surgery. In: Flynn JM, ed. Orthopaedic Knowledge Update 10. Rosemont, IL: American Academy of
Orthopaedic Surgeons; 2011:157-165.

© 2013 American Academy of Orthopaedic Surgeons 2013 Orthopaedic In-Training Examination


182 • American Academy of Orthopaedic Surgeons

Figure 45a Figure 45b

Question 45
Figures 45a and 45b are the sagittal and axial T2-weighted MRI scans of a 39-year-old man with a
3-month history of symptoms. Examination findings are most likely to indicate decreased sensation in
the left

1. upper arm, with weakness in the biceps.


2. index finger, with weakness in the hand intrinsics.
3. small finger, with weakness in the wrist extension.
4. middle finger, with weakness in the wrist flexion.
5. radial forearm, with weakness in shoulder abduction.

PREFERRED RESPONSE: 4

RECOMMENDED READINGS
Rhee JM, Yoon T, Riew KD. Cervical radiculopathy. J Am Acad Orthop Surg. 2007 Aug;15(8):486-94.
Review. PubMed PMID: 17664368.
Caridi JM, Pumberger M, Hughes AP. Cervical radiculopathy: a review. HSS J. 2011 Oct;7(3):265-72.
Epub 2011 Sep 9. PubMed PMID: 23024624.

© 2013 American Academy of Orthopaedic Surgeons 2013 Orthopaedic In-Training Examination


SECTION 9: Spine • 183

Figure 55a Figure 55b

Question 55
Figures 55a and 55b are the radiograph and CT scan of a 61-year-old woman who has had neck pain
after being involved in a high-speed motor vehicle collision. Examination reveals normal strength and
sensation in both upper and lower extremities, normal rectal tone, and no other injuries. The C1-C2 lateral
mass overhang measures 8.5 mm. What is the most appropriate treatment option?

1. Halo-vest orthosis
2. C1-C2 posterior cervical fusion
3. Occiput to C2 posterior cervical fusion
4. Cervical traction and closed reduction
5. Open reduction and internal fixation of C1

PREFERRED RESPONSE: 2

RECOMMENDED READINGS
Spence KF Jr, Decker S, Sell KW. Bursting atlantal fracture associated with rupture of the transverse
ligament. J Bone Joint Surg Am. 1970 Apr;52(3):543-9. PubMed PMID: 5425648.
Tan J, Li L, Sun G, Qian L, Yang M, Zeng C, Teng H, Jia L. C1 lateral mass-C2 pedicle screws and
crosslink compression fixation for unstable atlas fracture. Spine (Phila Pa 1976). 2009 Nov 1;34(23):2505-
9. PubMed PMID: 19927099.

© 2013 American Academy of Orthopaedic Surgeons 2013 Orthopaedic In-Training Examination


184 • American Academy of Orthopaedic Surgeons

Figure 67

Question 67
Figure 67 is the MRI scan of a 43-year-old man with an acute onset of neck pain, bilateral upper-extremity
weakness, and burning pain in his arms after hitting his head on a bookshelf. Examination initially
revealed 3/5 strength in both upper extremities, with normal motor strength in the lower extremities. What
is the best description of his spinal cord injury?

1. Central cord syndrome


2. Anterior cord syndrome
3. Posterior cord syndrome
4. Brown-Séquard syndrome
5. Complete spinal cord injury

PREFERRED RESPONSE: 1

RECOMMENDED READINGS
Nowak DD, Lee JK, Gelb DE, Poelstra KA, Ludwig SC. Central cord syndrome. J Am Acad Orthop Surg.
2009 Dec;17(12):756-65. Review. PubMed PMID: 19948700.
Dvorak MF, Fisher CG, Hoekema J, Boyd M, Noonan V, Wing PC, Kwon BK. Factors predicting motor
recovery and functional outcome after traumatic central cord syndrome: a long-term follow-up. Spine
(Phila Pa 1976). 2005 Oct 15;30(20):2303-11. Erratum in: Spine. 2006 May 15;31(11):1289. Kwon, Brian
[corrected to Kwon, Brian K]. PubMed PMID: 16227894.

© 2013 American Academy of Orthopaedic Surgeons 2013 Orthopaedic In-Training Examination


SECTION 9: Spine • 185

Figure 93a Figure 93b Figure 93c

Question 93
Figures 93a through 93c are the radiograph and CT and MRI scans of a 35-year-old man with diabetes
mellitus. He had severe neck pain for 6 weeks. He now has fevers and progressive weakness and
numbness in his upper extremities. Examination reveals 4/5 strength in both upper extremities, with
decreased sensation in both arms and hands and hyperreflexia. What is the most appropriate treatment
option?

1. Halo-vest fixation
2. Intravenous antibiotics
3. Posterior laminectomy
4. Percutaneous aspiration
5. Circumferential decompression and fusion

PREFERRED RESPONSE: 5

RECOMMENDED READINGS
Shousha M, Boehm H. Surgical treatment of cervical spondylodiscitis: a review of 30 consecutive
patients. Spine (Phila Pa 1976). 2012 Jan 1;37(1):E30-6. PubMed PMID: 21494200.
Heyde CE, Boehm H, El Saghir H, Tschöke SK, Kayser R. Surgical treatment of spondylodiscitis in the
cervical spine: a minimum 2-year follow-up. Eur Spine J. 2006 Sep;15(9):1380-7. Epub 2006 Jul 26.
PubMed PMID: 16868782.

© 2013 American Academy of Orthopaedic Surgeons 2013 Orthopaedic In-Training Examination


186 • American Academy of Orthopaedic Surgeons

Question 111
For patients undergoing posterior lumbar fusion, which risk factor is most responsible for development of
adjacent segment degeneration that necessitates further surgery?

1. Male gender
2. Single-level construct
3. Patient age younger than 45 years
4. Extension of the fusion to the sacrum
5. Laminectomy adjacent to the fusion

PREFERRED RESPONSE: 5

RECOMMENDED READINGS
Sears WR, Sergides IG, Kazemi N, Smith M, White GJ, Osburg B. Incidence and prevalence of surgery
at segments adjacent to a previous posterior lumbar arthrodesis. Spine J. 2011 Jan;11(1):11-20. PubMed
PMID: 21168094.
Cheh G, Bridwell KH, Lenke LG, Buchowski JM, Daubs MD, Kim Y, Baldus C. Adjacent segment
disease following lumbar/thoracolumbar fusion with pedicle screw instrumentation: a minimum 5-year
follow-up. Spine (Phila Pa 1976). 2007 Sep 15;32(20):2253-7. PubMed PMID: 17873819.

© 2013 American Academy of Orthopaedic Surgeons 2013 Orthopaedic In-Training Examination


SECTION 9: Spine • 187

Figure 124a Figure 124b

Question 124
Figures 124a and 124b are the MRI scans of a 74-year-old man who has difficulty walking distances
attributable to pain in both lower extremities. His leg pain is worse with lumbar extension and improves
with forward flexion. Examination reveals full strength and sensation in all 4 extremities. He shows
hyperreflexia and gait imbalance. He has tried physical therapy, medications, and epidural injections
without experiencing symptom relief. What is the most appropriate next step?

1. Daily lumbar traction


2. Referral to pain management
3. Lumbar decompression surgery
4. Spinal cord stimulator placement
5. An MRI scan of the cervical spine

PREFERRED RESPONSE: 5

RECOMMENDED READINGS
Bajwa NS, Toy JO, Young EY, Ahn NU. Is congenital bony stenosis of the cervical spine associated with
lumbar spine stenosis? An anatomical study of 1072 human cadaveric specimens. J Neurosurg Spine. 2012
Jul;17(1):24-9. Epub 2012 Apr 27. PubMed PMID: 22540170.
Lee MJ, Garcia R, Cassinelli EH, Furey C, Riew KD. Tandem stenosis: a cadaveric study in osseous
morphology. Spine J. 2008 Nov-Dec;8(6):1003-6. PubMed PMID: 18280216.

© 2013 American Academy of Orthopaedic Surgeons 2013 Orthopaedic In-Training Examination


188 • American Academy of Orthopaedic Surgeons

Figure 141a Figure 141b

Question 141
Figures 141a and 141b are the lumbar CT scans of a 16-year-old baseball pitcher who has had low-back
pain for 3 months. He has no radiating pain, numbness, or weakness. His pain is worsened by extension
and relieved with flexion. Examination reveals normal strength and sensation in his lower extremities.
What is the most likely diagnosis?

1. Spondylosis
2. Spondylolysis
3. Spondylolisthesis
4. Osteoid osteoma
5. Congenital dysplasia

PREFERRED RESPONSE: 2

RECOMMENDED READINGS
Cheung EV, Herman MJ, Cavalier R, Pizzutillo PD. Spondylolysis and spondylolisthesis in children and
adolescents: II. Surgical management. J Am Acad Orthop Surg. 2006 Aug;14(8):488-98. Review. PubMed
PMID: 16885480.
Saifuddin A, White J, Tucker S, Taylor BA. Orientation of lumbar pars defects: implications for
radiological detection and surgical management. J Bone Joint Surg Br. 1998 Mar;80(2):208-11. PubMed
PMID: 9546445.

© 2013 American Academy of Orthopaedic Surgeons 2013 Orthopaedic In-Training Examination


SECTION 9: Spine • 189

Figure 162

Question 162
Figure 162 is the CT scan of a 74-year-old woman who struck her head during a ground-level fall and has
severe neck pain. Examination reveals normal strength and sensation in her upper and lower extremities.
What is the most appropriate treatment option?

1. Cervical traction
2. Halo-vest orthosis
3. Anterior single-level fusion
4. Posterior single-level fusion
5. Posterior multilevel fusion

PREFERRED RESPONSE: 5

RECOMMENDED READINGS
Kubiak EN, Moskovich R, Errico TJ, Di Cesare PE. Orthopaedic management of ankylosing spondylitis. J
Am Acad Orthop Surg. 2005 Jul-Aug;13(4):267-78. PubMed PMID: 16112983.
Whang PG, Goldberg G, Lawrence JP, Hong J, Harrop JS, Anderson DG, Albert TJ, Vaccaro AR. The
management of spinal injuries in patients with ankylosing spondylitis or diffuse idiopathic skeletal
hyperostosis: a comparison of treatment methods and clinical outcomes. J Spinal Disord Tech. 2009
Apr;22(2):77-85. PubMed PMID: 19342927.

© 2013 American Academy of Orthopaedic Surgeons 2013 Orthopaedic In-Training Examination


190 • American Academy of Orthopaedic Surgeons

Figure 181

Question 181
Figure 181 is the MRI scan of a 59-year-old woman who has had no medical comorbidities but has had
difficulty with walking and balance for the past 6 months. She has severe pain in her neck and arms as
well as clumsiness and weakness in her arms. Examination reveals hyperreflexia in her upper and lower
extremities, a positive Hoffmann sign, and inability to perform rapid alternating movements. What
intervention would most likely produce the best long-term result?

1. Immobilization
2. Physical therapy
3. Surgical decompression
4. Neurology consultation
5. Cervical epidural injection

PREFERRED RESPONSE: 3

RECOMMENDED READINGS
Harrop JS, Naroji S, Maltenfort M, Anderson DG, Albert T, Ratliff JK, Ponnappan RK, Rihn JA, Smith
HE, Hilibrand A, Sharan AD, Vaccaro A. Cervical myelopathy: a clinical and radiographic evaluation and
correlation to cervical spondylotic myelopathy. Spine (Phila Pa 1976). 2010 Feb 10. [Epub ahead of print]
PubMed PMID: 20150835.
Emery SE. Cervical spondylotic myelopathy: diagnosis and treatment. J Am Acad Orthop Surg. 2001
Nov-Dec;9(6):376-88. Review. PubMed PMID: 11767723.

© 2013 American Academy of Orthopaedic Surgeons 2013 Orthopaedic In-Training Examination


SECTION 9: Spine • 191

Figure 190a Figure 190b

Question 190
Figures 190a and 190b are the sagittal and axial T2-weighted MRI scans of a 75-year-old man who is
experiencing progressively worsening bilateral lower-extremity pain and difficulty walking distances. In
another 4 years, nonsurgical treatment of his condition--compared to surgical treatment--is expected to
result in

1. equal improvement in pain.


2. equal improvement in function.
3. less improvement in pain.
4. more improvement in pain.
5. more improvement in function.

PREFERRED RESPONSE: 3

RECOMMENDED READINGS
Weinstein JN, Tosteson TD, Lurie JD, Tosteson A, Blood E, Herkowitz H, Cammisa F, Albert T, Boden
SD, Hilibrand A, Goldberg H, Berven S, An H. Surgical versus nonoperative treatment for lumbar spinal
stenosis four-year results of the Spine Patient Outcomes Research Trial. Spine (Phila Pa 1976). 2010 Jun
15;35(14):1329-38. PubMed PMID: 20453723.
Weinstein JN, Tosteson TD, Lurie JD, Tosteson AN, Blood EA, Hanscom B, Herkowitz H, Cammisa F,
Albert T, Boden SD, Hilibrand A, Goldberg H, Berven S, An H; SPORT Investigators. Surgical versus
nonsurgical therapy for lumbar spinal stenosis.N Engl J Med. 2008 Feb 21;358(8):794-810. doi: 10.1056/
NEJMoa0707136. PubMed PMID: 18287602.

© 2013 American Academy of Orthopaedic Surgeons 2013 Orthopaedic In-Training Examination


192 • American Academy of Orthopaedic Surgeons

Figure 198

Question 198
Figure 198 is the T2 sagittal MRI scan of a 47-year-old woman who has experienced pain in her lower
back and difficulty walking distances during the past 3 years. She has tried physical therapy, nonsteroidal
anti-inflammatory drugs, and multiple epidural injections without symptom relief. Which surgical
treatment is associated with the best outcome?

1. L4-L5 microdiscectomy
2. L4-L5 anterior interbody fusion
3. L4-L5 laminectomy
4. L4-L5 laminectomy and posterior uninstrumented fusion
5. L4-L5 laminectomy and posterior instrumented fusion

PREFERRED RESPONSE: 5

RECOMMENDED READINGS
Kornblum MB, Fischgrund JS, Herkowitz HN, Abraham DA, Berkower DL, Ditkoff JS. Degenerative
lumbar spondylolisthesis with spinal stenosis: a prospective long-term study comparing fusion and
pseudarthrosis. Spine (Phila Pa 1976). 2004 Apr 1;29(7):726-33; discussion 733-4. PubMed PMID:
15087793.
Majid K, Fischgrund JS. Degenerative lumbar spondylolisthesis: trends in management. J Am Acad
Orthop Surg. 2008 Apr;16(4):208-15. Review. PubMed PMID: 18390483.

© 2013 American Academy of Orthopaedic Surgeons 2013 Orthopaedic In-Training Examination


SECTION 9: Spine • 193

Figure 222a Figure 222b

Question 222
Figures 222a and 222b are the radiograph and MRI scan of a 41-year-old man who has had severe leg
pain for 6 months despite physical therapy and medications. Examination reveals normal strength and
sensation in both lower extremities. What is the most effective treatment option?

1. Lumbar interlaminar epidural injection


2. Lumbar transforaminal epidural injection
3. Posterior lumbar laminectomy
4. Posterior lumbar laminectomy and fusion
5. Posterior lumbar laminectomy and interbody fusion

PREFERRED RESPONSE: 3

RECOMMENDED READINGS
Weinstein JN, Tosteson TD, Lurie JD, Tosteson AN, Blood E, Hanscom B, Herkowitz H, Cammisa F,
Albert T, Boden SD, Hilibrand A, Goldberg H, Berven S, An H; SPORT Investigators. Surgical versus
nonsurgical therapy for lumbar spinal stenosis. N Engl J Med. 2008 Feb 21;358(8):794-810. PubMed
PMID: 18287602.
Deyo RA, Mirza SK, Martin BI, Kreuter W, Goodman DC, Jarvik JG. Trends, major medical
complications, and charges associated with surgery for lumbar spinal stenosis in older adults. JAMA. 2010
Apr 7;303(13):1259-65. PubMed PMID: 20371784.

© 2013 American Academy of Orthopaedic Surgeons 2013 Orthopaedic In-Training Examination


194 • American Academy of Orthopaedic Surgeons

Figure 236

Question 236
Figure 236 is the lateral radiograph of a 77-year-old man who had neck pain after a low-speed motor
vehicle collision. He had diffuse tenderness to palpation over his posterior cervical spine but the
remainder of the examination was unremarkable. Plain radiographs including Figure 236 were negative
for any evidence of fracture. What is the best next step in management?

1. Pulmonary function testing


2. An MRI scan of the cervical spine
3. Physical therapy and oral steroids
4. Immobilization in a rigid cervical collar for 6 weeks
5. Referral to a rheumatologist

PREFERRED RESPONSE: 2

RECOMMENDED READINGS
Whang PG, Goldberg G, Lawrence JP, Hong J, Harrop JS, Anderson DG, Albert TJ, Vaccaro AR. The
management of spinal injuries in patients with ankylosing spondylitis or diffuse idiopathic skeletal
hyperostosis: a comparison of treatment methods and clinical outcomes. J Spinal Disord Tech. 2009
Apr;22(2):77-85. PubMed PMID: 19342927.
Caron T, Bransford R, Nguyen Q, Agel J, Chapman J, Bellabarba C. Spine fractures in patients with
ankylosing spinal disorders. Spine (Phila Pa 1976). 2010 May 15;35(11):E458-64. PubMed PMID:
20421858.
Westerveld LA, Verlaan JJ, Oner FC. Spinal fractures in patients with ankylosing spinal disorders: a
systematic review of the literature on treatment, neurological status and complications. Eur Spine J. 2009
Feb;18(2):145-56. Epub 2008 Sep 13. Review. PubMed PMID: 18791749.

© 2013 American Academy of Orthopaedic Surgeons 2013 Orthopaedic In-Training Examination


SECTION 9: Spine • 195

Figure 247a Figure 247b Figure 247c

Figure 247d Figure 247e

Question 247
Figures 247a through 247e are the lateral radiographs of different spinal surgical procedures. Which
figure shows the surgical procedure for which rhBMP-2 has been FDA approved?

1. Figure 247a
2. Figure 247b
3. Figure 247c
4. Figure 247d
5. Figure 247e

PREFERRED RESPONSE: 1

RECOMMENDED READINGS
Epstein NE. Pros, cons, and costs of Infuse in spinal surgery. Surg Neurol Int. 2011 Jan 24;2:10. PubMed
PMID: 21297932.
Burkus JK, Transfeldt EE, Kitchel SH, Watkins RG, Balderston RA. Clinical and radiographic outcomes
of anterior lumbar interbody fusion using recombinant human bone morphogenetic protein-2. Spine (Phila
Pa 1976). 2002 Nov 1;27(21):2396-408.

© 2013 American Academy of Orthopaedic Surgeons 2013 Orthopaedic In-Training Examination


196 • American Academy of Orthopaedic Surgeons

Figure 260

Question 260
Figure 260 is the MRI scan of an 84-year-old woman who is admitted to the hospital with pain in her mid
back that started 8 weeks ago after a fall from a standing height. Examination reveals normal strength
and sensation. Her reflexes and rectal tone are normal. According to the most recent AAOS clinical
practice guideline, The Treatment of Symptomatic Osteoporotic Spinal Compression Fractures, which
recommendation is based on conclusive evidence for the treatment of her pain?

1. Brace
2. Calcitonin
3. Ibandronate
4. Bed rest and opioids
5. A supervised exercise program

PREFERRED RESPONSE: 3

RECOMMENDED READINGS
Esses SI, McGuire R, Jenkins J, Finkelstein J, Woodard E, Watters WC III, Goldberg MJ, Keith M,
Turkelson CM, Wies JL, Sluka P, Boyer KM, Hitchcock K. The treatment of symptomatic osteoporotic
spinal compression fractures. J Am Acad Orthop Surg. 2011 Mar;19(3):176-82. PubMed PMID:
21368099.
Sieber P, Lardelli P, Kraenzlin CA, Kraenzlin ME, Meier C. Intravenous bisphosphonates for
postmenopausal osteoporosis: safety profiles of zoledronic acid and ibandronate in clinical practice. Clin
Drug Investig. 2013 Feb;33(2):117-22. doi: 10.1007/s40261-012-0041-1. PubMed PMID: 23184667.

© 2013 American Academy of Orthopaedic Surgeons 2013 Orthopaedic In-Training Examination


SECTION 9: Spine • 197

Figure 271a Figure 271b

Question 271
Figures 271a and 271b are the MRI scans of a 49-year-old woman with pain that has been radiating down
her right lower extremity to the dorsum of her foot for the last 3 weeks. Examination reveals significant
pain with a straight-leg raise and decreased sensation in the right big toe. What is the most appropriate
next step in management?

1. Right L4-L5 microdiscectomy


2. Posterior L4-L5 decompression and fusion
3. Trial of nonsteroidal anti-inflammatory drugs
4. Neurology referral for nerve conduction studies
5. Transcutaneous electrical nerve stimulation treatment for 3 hours daily

PREFERRED RESPONSE: 3

RECOMMENDED READINGS
Weinstein JN, Tosteson TD, Lurie JD, Tosteson AN, Hanscom B, Skinner JS, Abdu WA, Hilibrand AS,
Boden SD, Deyo RA. Surgical vs nonoperative treatment for lumbar disk herniation: the Spine Patient
Outcomes Research Trial (SPORT): a randomized trial. JAMA. 2006 Nov 22;296(20):2441-50. PubMed
PMID: 17119140.
Weinstein JN, Lurie JD, Tosteson TD, Skinner JS, Hanscom B, Tosteson AN, Herkowitz H, Fischgrund J,
Cammisa FP, Albert T, Deyo RA. Surgical vs nonoperative treatment for lumbar disk herniation: the Spine
Patient Outcomes Research Trial (SPORT) observational cohort. JAMA. 2006 Nov 22;296(20):2451-9.
PubMed PMID: 17119141.
Weinstein JN, Lurie JD, Tosteson TD, Tosteson AN, Blood EA, Abdu WA, Herkowitz H, Hilibrand
A, Albert T, Fischgrund J. Surgical versus nonoperative treatment for lumbar disc herniation: four-
year results for the Spine Patient Outcomes Research Trial (SPORT). Spine (Phila Pa 1976). 2008 Dec
1;33(25):2789-800. PubMed PMID: 19018250.

© 2013 American Academy of Orthopaedic Surgeons 2013 Orthopaedic In-Training Examination


198 • American Academy of Orthopaedic Surgeons

Figure 274a Figure 274b

Question 274
Figures 274a and 274b are the MRI scans of a 53-year-old woman who has had severe, increasing mid-
back pain for 3 weeks. She had a small bowel transplant 10 years ago. Examination reveals tenderness to
palpation over the thoracic spine. She has full strength and sensation in all extremities, is normoreflexic
throughout, and is currently afebrile. What is the most appropriate next step in management?

1. Whole-body bone scan


2. Outpatient physical therapy
3. CT-guided biopsy of the thoracic spine
4. Thoracic decompression and stabilization
5. Referral to pain management for an epidural injection

PREFERRED RESPONSE: 3

RECOMMENDED READINGS
Tay BK, Deckey J, Hu SS. Spinal infections. J Am Acad Orthop Surg. 2002 May-Jun;10(3):188-97.
Review. PubMed PMID: 12041940.
Cousins JP, Haughton VM. Magnetic resonance imaging of the spine. J Am Acad Orthop Surg. 2009
Jan;17(1):22-30. Review. PubMed PMID: 19136424.

© 2013 American Academy of Orthopaedic Surgeons 2013 Orthopaedic In-Training Examination


SECTION 10: Sports Medicine • 199

SECTION 10: Sports Medicine

Question 11
A 20-year-old college basketball player has lateral ankle pain after sustaining an ankle sprain. His
pain persists despite allowing a sufficient period of rest and rehabilitation. He has a history of multiple
previous sprains, and describes this pain as being different than his usual pain after a sprain. He has
tenderness to palpation along the posterior fibula and reproducible pain with resisted eversion. What is the
most appropriate treatment option?

1. Core repair and tubularization of the peroneus brevis tendon


2. Direct repair of the anterior talofibular and calcaneofibular ligaments
3. Ankle arthroscopy and debridement of the lateral gutter and tibiofibular joint
4. Lateral ankle stabilization with a transfer of the peroneus brevis through the fibula
5. Continued physical therapy with proprioceptive training and peroneal strengthening

PREFERRED RESPONSE: 1

RECOMMENDED READINGS
Philbin TM, Landis GS, Smith B. Peroneal tendon injuries. J Am Acad Orthop Surg. 2009 May;17(5):306-
17. Review. PubMed PMID: 19411642.
Reed ME, Feibel JB, Donley BG, Giza E. Athletic ankle injuries. In: Kibler WB, ed. Orthopaedic
Knowledge Update: Sports Medicine 4. Rosemont, IL: American Academy of Orthopaedic Surgeons;
2009:199-214.

Question 23
During preparticipation physicals for college football, an athlete tests positive for the sickle-cell trait.
With regard to clearance to play, his team physician should

1. counsel the athlete about his personal risk for bone infarcts.
2. recommend a prophylactic splenectomy prior to participation.
3. bar the athlete from participating in National Collegiate Athletic Association-sanctioned
events.
4. assure the athlete that he can participate in football without concern.
5. ensure that the athlete is given adequate recovery time and remains hydrated.

PREFERRED RESPONSE: 5

RECOMMENDED READINGS
2011-2012 NCAA Sports Medicine Handbook. Indianapolis, IN: The National Collegiate Athletic
Association.
Kark JA, Posey DM, Schumacher HR, Ruehle CJ. Sickle-cell trait as a risk factor for sudden death in
physical training. N Engl J Med. 1987 Sep 24;317(13):781-7. PubMed PMID: 3627196.

© 2013 American Academy of Orthopaedic Surgeons 2013 Orthopaedic In-Training Examination


200 • American Academy of Orthopaedic Surgeons

Figure 39

Question 39
Figure 39 is the anteroposterior radiograph of a marathon runner who has left groin pain that prevents
her from running. She recently got back into her usual running routine after an ankle injury prevented
her from running for several months. She now has pain with any weight bearing. What is the most
appropriate treatment option?

1. Hip resurfacing arthroplasty


2. Hip arthroscopy with removal of the cam lesion
3. Internal fixation of the femoral neck with multiple screws
4. Trial of nonsurgical treatment with no weight bearing on the left leg
5. Vitamin D level assessment and supplementation with 50000 units weekly

PREFERRED RESPONSE: 3

RECOMMENDED READINGS
Shin AY, Gillingham BL. Fatigue fractures of the femoral neck in athletes. J Am Acad Orthop Surg. 1997
Nov;5(6):293-302. PubMed PMID: 10795065.
Hajek MR, Noble HB. Stress fractures of the femoral neck in joggers: case reports and review of the
literature. Am J Sports Med. 1982 Mar-Apr;10(2):112-6. PubMed PMID: 7081524.

© 2013 American Academy of Orthopaedic Surgeons 2013 Orthopaedic In-Training Examination


SECTION 10: Sports Medicine • 201

Video 42 (click to play)

Question 42
Denervation most typically associated with the finding seen in Figure 42 results in which characteristic
finding?

1. Internal rotation weakness


2. External rotation weakness in adduction
3. External rotation weakness in abduction
4. No noticeable weakness of the shoulder
5. Forward flexion weakness (more than 90 degrees)

PREFERRED RESPONSE: 2

RECOMMENDED READINGS
Piatt BE, Hawkins RJ, Fritz RC, Ho CP, Wolf E, Schickendantz M. Clinical evaluation and treatment of
spinoglenoid notch ganglion cysts. J Shoulder Elbow Surg. 2002 Nov-Dec;11(6):600-4. PubMed PMID:
12469086.
Westerheide KJ, Dopirak RM, Karzel RP, Snyder SJ. Suprascapular nerve palsy secondary to spinoglenoid
cysts: results of arthroscopic treatment. Arthroscopy. 2006 Jul;22(7):721-7. PubMed PMID: 16843807.
Piasecki DP, Romeo AA, Bach BR Jr, Nicholson GP. Suprascapular neuropathy. J Am Acad Orthop Surg.
2009 Nov;17(11):665-76. Review. PubMed PMID: 19880677.

© 2013 American Academy of Orthopaedic Surgeons 2013 Orthopaedic In-Training Examination


202 • American Academy of Orthopaedic Surgeons

Question 51
A 36-year-old man who was playing recreational basketball felt a pop in the back of his leg and is now
unable to walk. Rest, ice, and elevation have been ineffective at restoring his leg. Examination reveals
pain over the posterior calf, some ecchymosis, and weak plantar flexion strength. A Thompson test result
is positive. Compared with nonsurgical treatment, surgical treatment is more likely to

1. carry a lower risk for equinus contracture.


2. restore strength (closer to usual levels).
3. reduce risk for rerupture.
4. allow for quicker ambulation and recovery.
5. result in better outcomes on blind randomized studies.

PREFERRED RESPONSE: 3

RECOMMENDED READINGS
Keating JF, Will EM. Operative versus non-operative treatment of acute rupture of tendo Achillis: a
prospective randomised evaluation of functional outcome. J Bone Joint Surg Br. 2011 Aug;93(8):1071-8.
PubMed PMID: 21768631.
Willits K, Amendola A, Bryant D, Mohtadi NG, Giffin JR, Fowler P, Kean CO, Kirkley A. Operative
versus nonoperative treatment of acute Achilles tendon ruptures: a multicenter randomized trial using
accelerated functional rehabilitation. J Bone Joint Surg Am. 2010 Dec 1;92(17):2767-75. Epub 2010 Oct
29. PubMed PMID: 21037028.

Question 62
A 14-year-old girl has a 6-week history of diffuse pain in both knees after attending cheerleading camp
without trauma. She denies mechanical symptoms or swelling, but does state her knees “give-way”
and “click” occasionally. Examination and radiographs are unremarkable, with the exception of global
discomfort to palpation of both knees. What is the most appropriate next step?

1. MRI scans of both knees


2. Corticosteroid injection into both knees
3. Bone scan with pinhole views of both knees
4. Lab studies to rule out a rheumatologic condition
5. A physical therapy regimen to both lower extremities

PREFERRED RESPONSE: 5

RECOMMENDED READINGS
Dalton SE: Overuse injuries in adolescent athletes. Sports Med 1992;13:58-70. PubMed PMID: 1553456.
Earl JE, Vetter CS: Patellofemoral pain. Phys Med Rehabil Clin N Am 2007;18:439-458. PubMed PMID:
17678761.
Outerbridge AR, Micheli LJ: Overuse injuries in the young athlete. Clin Sports Med 1995;14:503-516.
PubMed PMID: 7553920.

© 2013 American Academy of Orthopaedic Surgeons 2013 Orthopaedic In-Training Examination
SECTION 10: Sports Medicine • 203

Question 83
A 20-year-old collegiate rower has pain along the left side of his chest just anterior to the midaxillary
line. The pain began approximately 4 weeks after he started preseason training. The pain occurs almost
immediately after he begins rowing and goes away when he stops. He has not noticed the pain while
running. Which study will most likely reveal the diagnosis?

1. Echocardiogram
2. Electrocardiogram
3. 3-phase bone scan
4. Rib series radiographs
5. Posteroanterior chest radiograph

PREFERRED RESPONSE: 3

RECOMMENDED READINGS
Karlson KA. Rib stress fractures in elite rowers. A case series and proposed mechanism. Am J Sports Med.
1998 Jul-Aug;26(4):516-9. Review. PubMed PMID: 9689370.
McDonnell LK, Hume PA, Nolte V. Rib stress fractures among rowers: definition, epidemiology,
mechanisms, risk factors and effectiveness of injury prevention strategies. Sports Med. 2011 Nov
1;41(11):883-901. doi: 10.2165/11593170-000000000-00000. Review. PubMed PMID: 21985212.

Question 99
A 42-year-old man has a chondral defect of the medial femoral condyle that is approximately 1 cm in
diameter. He has a very athletic lifestyle, wishes to remain active, and is now seeking a third opinion. He
has seen 2 orthopaedic surgeons; the first surgeon recommended microfracture for the chondral defect, and
the other recommended an osteochondral autograft transplantation (OATS). What should the patient be
told?

1. Recovery is faster with microfracture, the outcomes are better with OATS, and both techniques
produce the same reparative surface.
2. Recovery is faster with OATS, the outcomes are better with microfracture, and both techniques
produce the same reparative surface.
3. The outcomes are better with OATS, rehabilitation is faster with OATS, and the reparative
surface is articular cartilage with OATS.
4. Recovery time and outcomes are similar between these 2 techniques, and the reparative
surface with microfracture is fibrocartilage.
5. Recovery and outcomes are similar between these 2 techniques, and the reparative tissue with
OATS is fibrocartilage.

PREFERRED RESPONSE: 4

© 2013 American Academy of Orthopaedic Surgeons 2013 Orthopaedic In-Training Examination


204 • American Academy of Orthopaedic Surgeons

RECOMMENDED READINGS
Alfond JW, Cole BJ: Cartilage restoration, Part 1: Basic science, historical perspective, patient evaluation,
and treatment options. Am J Sports Med 2005;33:295-306. PubMed PMID: 15701618.
Magnussen RA, Dunn WR, Carey JL, Spindler KP: Treatment of focal articular cartilage defects in the
knee: a systemic review. Clin Orthop Relat Res 2008:466:952-962. PubMed PMID: 18196358.

Question 113
While performing an arthroscopic procedure, an instrument has a mechanical failure resulting in a 1.5-
mm segment of metallic debris incarcerated within the soft tissue. After 45 minutes of fluoroscopic
localization and special arthroscopic techniques, the fragment is determined to cause no harm to the
patient. Upon recognizing the event, the treating surgeon should

1. immediately abandon the procedure, close the portals, and obtain further imaging.
2. complete the surgery, determine the risk for potential injury to the patient, and immediately
notify the patient and family following the procedure.
3. complete the surgery, follow the patient clinically for any unintended consequences before
disclosing the event.
4. call the patient’s family from the operating room to explain the error before proceeding to
complete the procedure.
5. refrain from formal disclosure if the fragment is unlikely to cause any further damage.

PREFERRED RESPONSE: 2

RECOMMENDED READINGS
Lundy DW, Weinstein SL. Patient safety and risk management. In: Flynn JM, ed. Orthopaedic Knowledge
Update 10. Rosemont, IL: American Academy of Orthopaedic Surgeons; 2011:3-10.
http://www.aaos.org/about/papers/advistmt/1015.asp (Accessed 8/1/2012).

© 2013 American Academy of Orthopaedic Surgeons 2013 Orthopaedic In-Training Examination


SECTION 10: Sports Medicine • 205

Video 125 (click to play)

Question 125
Video 125 shows a subscapularis repair viewing from the posterior portal looking anteriorly at the lateral
border of the rotator interval. What serves as a landmark for appropriate placement of the anchor for the
upper border repair?

1. Biceps tendon
2. Middle glenohumeral ligament
3. Leading edge of the supraspinatus
4. Posterior band of the inferior glenohumeral ligament
5. Superior glenohumeral and coracohumeral ligaments

PREFERRED RESPONSE: 5

RECOMMENDED READINGS
Burkhart SS, Brady PC. Arthroscopic subscapularis repair: surgical tips and pearls A to Z. Arthroscopy.
2006 Sep;22(9):1014-27. PubMed PMID: 16952733.
Lo IK, Burkhart SS. The comma sign: An arthroscopic guide to the torn subscapularis tendon.
Arthroscopy. 2003 Mar;19(3):334-7. PubMed PMID: 12627163.

© 2013 American Academy of Orthopaedic Surgeons 2013 Orthopaedic In-Training Examination


206 • American Academy of Orthopaedic Surgeons

Question 145
A 23-year-old woman sustained an ankle inversion injury 6 weeks ago. She reports pain and difficulty
returning to recreational basketball. Examination reveals tenderness of the anterior talofibular ligament,
pain laterally with inversion stress, weakness without pain in eversion against resistance, and a negative
anterior drawer. What is the best next treatment step?

1. Physical therapy
2. Cast immobilization
3. Peroneal tendon repair
4. Chrisman-Snook reconstruction
5. Modified Brostrum reconstruction

PREFERRED RESPONSE: 1

RECOMMENDED READINGS
Digiovanni BF, Partal G, Baumhauer JF: Acute ankle injury and chronic lateral instability in the athlete.
Clin Sports Med 2004;23:1-19. PubMed PMID: 15062581.
Berlet GC, D'Onofrio MM, Lee TH: Acute ankle sprain, chronic ankle instability, and subtalar laxity. In:
Thordarson DB, ed. Foot and Ankle. Philadelphia, PA: Lippincott-Raven; 2004:242-249.

Question 158
An elite-level pitcher with a history of chronic moderate medial elbow pain reports a sudden pop and
severe pain along the medial elbow while throwing a pitch. Examination reveals a positive moving valgus
stress test. What is the most appropriate next treatment step?

1. Rehabilitation of the flexor-pronator musculature


2. Bracing of the elbow to facilitate a return to pitching
3. Early primary repair of the ulnar collateral ligament
4. Early ulnar collateral ligament reconstruction
5. Early ulnar collateral ligament reconstruction and ulnar nerve transposition

PREFERRED RESPONSE: 4

RECOMMENDED READINGS
Vitale MA, Ahmad CS. The outcome of elbow ulnar collateral ligament reconstruction in overhead
athletes: a systematic review. Am J Sports Med. 2008 Jun;36(6):1193-205. Review. PubMed PMID:
18490476.
Rettig AC, Sherrill C, Snead DS, Mendler JC, Mieling P. Nonoperative treatment of ulnar collateral
ligament injuries in throwing athletes. Am J Sports Med. 2001 Jan-Feb;29(1):15-7. PubMed PMID:
11206249.
Safran MR: Nonacute medial elbow injuries. In: Kibler WB, ed. Orthopaedic Knowledge Update: Sports
Medicine 4. Rosemont, IL: American Academy of Orthopaedic Surgeons; 2009:47-57.

© 2013 American Academy of Orthopaedic Surgeons 2013 Orthopaedic In-Training Examination


SECTION 10: Sports Medicine • 207

Question 184
At her 6-week follow-up visit after arthroscopic repair of a full-thickness rotator cuff tear, a patient has not
yet attended formal physical therapy. Which outcome at 1 year is expected?

1. Persistent stiffness of the shoulder, resulting in loss of function


2. High likelihood of needing a second procedure for a capsular release
3. Improved healing of the rotator cuff despite persistent stiffness of the shoulder
4. No long-term difference in motion compared to early physical therapy
5. A significant decrease in Constant score compared to early physical therapy

PREFERRED RESPONSE: 4

RECOMMENDED READINGS
Parsons BO, Gruson KI, Chen DD, Harrison AK, Gladstone J, Flatow EL. Does slower rehabilitation after
arthroscopic rotator cuff repair lead to long-term stiffness? J Shoulder Elbow Surg. 2010 Oct;19(7):1034-
9. Epub 2010 Jul 24. PubMed PMID: 20655763.
Trenerry K, Walton JR, Murrell GA. Prevention of shoulder stiffness after rotator cuff repair. Clin Orthop
Relat Res. 2005 Jan;(430):94-9. PubMed PMID: 15662309.

Question 193
Which of the following is the most important restraint to medial instability of the long head of the biceps
tendon?

1. Tendon of the subscapularis


2. Coracohumeral ligament
3. Superior glenohumeral ligament
4. Morphology of the bicipital groove
5. Origin of the long head of the biceps in the supraglenoid tubercle

PREFERRED RESPONSE: 1

RECOMMENDED READINGS
Walch G, Nové-Josserand L, Boileau P, Levigne C: Subluxations and dislocations of the tendon of the
long head of the biceps. J Shoulder Elbow Surg 1998;7:100-108. PubMed PMID: 9593086.
Maier D, Jaeger M, Suedkamp NP, Koestler W: Stabilization of the long head of the biceps tendon in the
context of early repair of traumatic subscapularis tendon tears. J Bone Joint Surg Am 2007;89:1763-1769.
PubMed PMID: 17671016.

© 2013 American Academy of Orthopaedic Surgeons 2013 Orthopaedic In-Training Examination


208 • American Academy of Orthopaedic Surgeons

Question 202
An inferior placement of the posterior shoulder portal endangers which vital structure?

1. Radial nerve
2. Axillary nerve
3. Long thoracic nerve
4. Thoracoacromial artery
5. Anterior humeral circumflex artery

PREFERRED RESPONSE: 2

RECOMMENDED READINGS
Lo IK, Lind CC, Burkhart SS. Glenohumeral arthroscopy portals established using an outside-in
technique: neurovascular anatomy at risk. Arthroscopy. 2004 20(6):596-602.PubMed PMID: 15241310.
Meyer M, Graveleau N, Hardy P, Landreau P. Anatomic risks of shoulder arthroscopy portals: anatomic
cadaveric study of 12 portals. Arthroscopy. 2007 23(5):529-536. PubMed PMID: 17478285.

Question 205
Toward the end of a preseason football practice, a player approaches his trainer with difficulty
remembering what he is supposed to do during his position drills. He is confused and disoriented, clearly
fatigued, soaked in sweat, and his skin is pale. What is the most appropriate next step?

1. Have the athlete lie down on the sidelines for administration of intravenous fluid.
2. Immediately perform a thorough neurologic evaluation on the sidelines.
3. Assure the athlete that he is simply dehydrated and can return after rehydrating.
4. Administer a Sideline Assessment of Concussion test to determine return to play.
5. Obtain a core temperature in a cooled training room while hydrating the athlete.

PREFERRED RESPONSE: 5

RECOMMENDED READINGS
American College of Sports Medicine, Armstrong LE, Casa DJ, Millard-Stafford M, Moran DS, Pyne SW,
Roberts WO. American College of Sports Medicine position stand. Exertional heat illness during training
and competition. Med Sci Sports Exerc. 2007 Mar;39(3):556-72. Review. PubMed PMID: 17473783.
American College of Sports Medicine, Sawka MN, Burke LM, Eichner ER, Maughan RJ, Montain SJ,
Stachenfeld NS. American College of Sports Medicine position stand. Exercise and fluid replacement.
Med Sci Sports Exerc. 2007 Feb;39(2):377-90. Review. PubMed PMID: 17277604.
Joy SM. Heat and hydration. In: Kibler WB, ed. Orthopaedic Knowledge Update: Sports Medicine 4.
Rosemont, IL: American Academy of Orthopaedic Surgeons; 2009:379-385.

© 2013 American Academy of Orthopaedic Surgeons 2013 Orthopaedic In-Training Examination


SECTION 10: Sports Medicine • 209

Question 227
A 29-year-old athlete reports recurrent anterior shoulder instability after surgery. Performing an
arthroscopic revision surgery is contraindicated when there is capsular attenuation or

1. glenoid bone loss of 15%.


2. an associated rotator cuff tear.
3. postthermal capsular necrosis.
4. a Hill-Sachs lesion involving 20% of the humeral head.
5. the patient participates in sports that involve contact and collision.

PREFERRED RESPONSE: 3

RECOMMENDED READINGS
Creighton RA, Romeo AA, Brown FM Jr, Hayden JK, Verma NN. Revision arthroscopic shoulder
instability repair. Arthroscopy 2007;23:703-709. PubMed PMID: 17637404.
Miniaci A, Codsi MJ: Thermal capsulorrhaphy for the treatment of shoulder instability. Am J Sports Med
2006:34:1356-63. PubMed PMID: 16685091.

Question 248
During routine knee arthroscopy, the anterior cruciate ligament is visualized with the knee in 95 degrees of
flexion. At this angle of knee flexion, the

1. posteromedial bundle is loose and the anterolateral bundle is tight.


2. posterolateral bundle is loose and the anteromedial bundle is tight.
3. anteromedial bundle is loose and the posterolateral bundle is tight.
4. anterolateral bundle is loose and the posteromedial bundle is tight.
5. anterolateral bundle is tight and the posteromedial bundle is tight.

PREFERRED RESPONSE: 2

RECOMMENDED READINGS
Tjoumakaris FP, Donegan DJ, Sekiya JK. Partial tears of the anterior cruciate ligament: diagnosis and
treatment. Am J Orthop (Belle Mead NJ). 2011 Feb;40(2):92-7. Review. PubMed PMID: 21720597.
Bicer EK, Lustig S, Servien E, Selmi TA, Neyret P. Current knowledge in the anatomy of the human
anterior cruciate ligament. Knee Surg Sports Traumatol Arthrosc. 2010 Aug;18(8):1075-84. Epub 2009
Dec 3. Review. PubMed PMID: 19956929.

© 2013 American Academy of Orthopaedic Surgeons 2013 Orthopaedic In-Training Examination


210 • American Academy of Orthopaedic Surgeons

Question 257
What is an absolute contraindication to meniscal transplantation?

1. Stable joint
2. Angular deformity
3. Inflammatory arthritis
4. Localized chondral defect
5. Anterior cruciate ligament deficiency

PREFERRED RESPONSE: 3

RECOMMENDED READINGS
West RV, Fu FH. Soft-tissue physiology and repair. In: Vaccaro AR ed: Orthopaedic Knowledge Update 8.
Rosemont, IL, American Academy of Orthopaedic Surgeons; 2005:15-27.
Cole BJ, Carter TR, Rodeo SA: Allograft meniscal transplantation: Background, techniques, and results.
Instr Course Lect 2003;52:383-396. PubMed PMID: 12690865.

Question 265
A 29-year-old athlete with postmeniscectomy pain syndrome after prior arthroscopic meniscectomy is
referred for a meniscal allograft. What is the most likely long-term outcome for a meniscal allograft
transplantation?

1. Rejection with early failure


2. Cartilage regeneration
3. Relative acellularity and possible tearing
4. Permanent reduction in pain and swelling
5. Acceleration in the progression of osteoarthritis

PREFERRED RESPONSE: 3

RECOMMENDED READINGS
Rath E, Richmond JC, Yassir W, Albright JD, Gundogan F. Meniscal allograft transplantation: Two to
eight year results. Am J Sports Med 2001;29:410-4. PubMed PMID: 11476377.
Sekiya JK, Ellingson CI: Meniscal allograft transplantation. J Am Acad Orthop Surg 2006:14:164-74.
PubMed PMID: 16520367.

© 2013 American Academy of Orthopaedic Surgeons 2013 Orthopaedic In-Training Examination


SECTION 10: Sports Medicine • 211

Figure 275a Figure 275b Figure 275c

Question 275
Figures 275a through 275c are the radiographs of a 28-year-old recreational basketball player who
underwent autograft anterior cruciate ligament reconstruction and a partial medial meniscectomy 4 years
ago. Although his initial results were favorable, he has persistent instability symptoms and “giving way”
when attempting to participate in desired sports activities. Examination reveals the following: a 2A
Lachman, 3+ pivot shift, negative external rotation dial, and a positive McMurray maneuver for the medial
compartment. His recurrent instability symptoms are most likely related to

1. an unstable lateral meniscal tear.


2. the development of posterolateral instability.
3. femoral tunnel placement that did not restore rotatory stability.
4. femoral tunnel placement that did not restore the posteromedial bundle.
5. femoral tunnel fixation that did not adequately address the anterolateral bundle.

PREFERRED RESPONSE: 3

RECOMMENDED READINGS
Yasuda K, van Eck CF, Hoshino Y, Fu FH, Tashman S. Anatomic single- and double-bundle anterior
cruciate ligament reconstruction, part 1: Basic science. Am J Sports Med August 2011 39 1789-1799.
PubMed PMID: 21596902.
Smith CK, Howell SM, Hull ML. Anterior laxity, slippage, and recovery of function in the first year after
tibialis allograft anterior cruciate ligament reconstruction. Am J Sports Med. 2011 Jan;39(1):78-88. Epub
2010 Oct 7. PubMed PMID: 20929933.

© 2013 American Academy of Orthopaedic Surgeons 2013 Orthopaedic In-Training Examination


212 • American Academy of Orthopaedic Surgeons

SECTION 11: Musculoskeletal Trauma

Question 2
An 85-year-old woman with a history of poorly controlled hypertension, orally controlled diabetes, and
atrial fibrillation with controlled rate is seen on a Thursday with an unstable intertrochanteric fracture.
Evaluation reveals she is slightly hypernatremic (sodium level 155 mEq/L) (reference range, 136-142
mEq/L). What is the most appropriate treatment option?

1. Traction and hydration because surgical intervention puts this patient at high risk
2. One liter of normal saline and immediate (Thursday) open reduction and internal fixation with
a plate
3. Rehydration, medical evaluation, and open reduction and internal fixation with a nail within
48 hours
4. Rehydration, cardiac stress testing, endocrine evaluation, and open reduction and internal
fixation with a plate on Monday
5. Immediate open reduction and internal fixation with a nail followed by admission to medicine
for treatment after surgery

PREFERRED RESPONSE: 3

RECOMMENDED READINGS
Switzer JA, Layman MD, Bogoch ER. Perioperative and postoperative considerations in the geriatric
patient. In: Schmidt AH, Teague DC, eds. Orthopaedic Knowledge Update: Trauma 4. Rosemont, IL:
American Academy of Orthopaedic Surgeons; 2010:535-544.
Zuckerman JD, Skovron ML, Koval KJ, Aharonoff G, Frankel VH. Postoperative complications and
mortality associated with operative delay in older patients who have a fracture of the hip. J Bone Joint
Surg Am. 1995 Oct;77(10):1551-6. PubMed PMID: 7593064.

Question 6
What is the most important determinant of the energy imparted to the soft tissues as a result of a gunshot
wound?

1. Yaw
2. Mass
3. Range
4. Caliber
5. Velocity

PREFERRED RESPONSE: 5

© 2013 American Academy of Orthopaedic Surgeons 2013 Orthopaedic In-Training Examination


SECTION 11: Musculoskeletal Trauma • 213

RECOMMENDED READINGS
Bartlett CS, Helfet DL, Hausman MR, Strauss E. Ballistics and gunshot wounds: effects on
musculoskeletal tissues. J Am Acad Orthop Surg. 2000 Jan-Feb;8(1):21-36. Review. PubMed PMID:
10666650.
Dougherty PJ, Najibi S, Silverton C, Vaidya R. Gunshot wounds: epidemiology, wound ballistics, and
soft-tissue treatment. Instr Course Lect. 2009;58:131-9. PubMed PMID: 19385526.

Figure 10

Question 10
Figure 10 is the radiograph of an 18-year-old man who sustained an isolated gunshot wound to his right
thigh. After appropriate evaluation and resuscitation, the fracture is repaired with a reamed intramedullary
nail. What is the most commonly encountered complication in this scenario?

1. Infection
2. Malunion
3. Nonunion
4. Fat embolism
5. Pulmonary embolism

PREFERRED RESPONSE: 2

RECOMMENDED READINGS
Lindsey JD, Krieg JC. Femoral malrotation following intramedullary nail fixation. J Am Acad Orthop
Surg. 2011 Jan;19(1):17-26. PubMed PMID: 21205764.
Ricci WM, Gallagher B, Haidukewych GJ. Intramedullary nailing of femoral shaft fractures: current
concepts. J Am Acad Orthop Surg. 2009 May;17(5):296-305. Review. PubMed PMID: 19411641.

© 2013 American Academy of Orthopaedic Surgeons 2013 Orthopaedic In-Training Examination


214 • American Academy of Orthopaedic Surgeons

Question 14
A 25-year-old thin man sustained a bimalleolar left ankle fracture, a comminuted spiral midshaft left
humeral fracture, and a grade IV splenic laceration during a motor vehicle collision. His left radial nerve
function is intact. He underwent splenectomy immediately and his fractures were splinted. In counseling
the patient regarding surgical vs nonsurgical treatment of the humerus fracture, you would advise that

1. the risk for radial nerve palsy is higher in spiral humeral shaft fractures that are treated
nonsurgically.
2. the patient may bear weight through the plated humeral fracture for the purpose of using
ambulatory aids.
3. a functional fracture brace will not adequately maintain humeral shaft fracture alignment
during the healing process.
4. surgical fixation of the humeral fracture will allow for earlier fracture union than nonsurgical
treatment with a functional fracture brace.
5. long-term outcomes for plated humeral shaft fractures are better than for fractures treated
nonsurgically.

PREFERRED RESPONSE: 2

RECOMMENDED READINGS
Tingstad EM, Wolinsky PR, Shyr Y, Johnson KD. Effect of immediate weightbearing on plated fractures
of the humeral shaft. J Trauma. 2000 Aug;49(2):278-80. PubMed PMID: 10963539.
Ekholm R, Tidermark J, Törnkvist H, Adami J, Ponzer S. Outcome after closed functional treatment of
humeral shaft fractures. J Orthop Trauma. 2006 Oct;20(9):591-6. PubMed PMID: 17088659.
Shao YC, Harwood P, Grotz MR, Limb D, Giannoudis PV. Radial nerve palsy associated with fractures of
the shaft of the humerus: a systematic review. J Bone Joint Surg Br. 2005 Dec;87(12):1647-52. Review.
PubMed PMID: 16326879.
Hak DJ. Radial nerve palsy associated with humeral shaft fractures. Orthopedics. 2009 Feb;32(2):111.
Review. PubMed PMID: 19301795.

© 2013 American Academy of Orthopaedic Surgeons 2013 Orthopaedic In-Training Examination


SECTION 11: Musculoskeletal Trauma • 215

Figure 32a Figure 32b Figure 32c

Question 32
Figures 32a through 32c are the radiographs of a 31-year-old man who was involved in a motor vehicle
collision. He has severe foot pain, marked swelling, and is unable to ambulate. What is the most
appropriate definitive treatment step?

1. External fixation
2. Closed reduction and casting
3. Closed reduction and percutaneous pinning
4. Open reduction and internal fixation with rigid fixation of the first to fifth tarsometatarsal
joints
5. Open reduction and internal fixation with rigid fixation of the first to third tarsometatarsal
joints and Kirschner wire fixation of the fourth and fifth tarsometatarsal joints

PREFERRED RESPONSE: 5

RECOMMENDED READINGS
Watson TS, Shurnas PS, Denker J. Treatment of Lisfranc joint injury: current concepts. J Am Acad Orthop
Surg. 2010 Dec;18(12):718-28. Review. PubMed PMID: 21119138.
Rammelt S, Schneiders W, Schikore H, Holch M, Heineck J, Zwipp H. Primary open reduction and
fixation compared with delayed corrective arthrodesis in the treatment of tarsometatarsal (Lisfranc)
fracture dislocation. J Bone Joint Surg Br. 2008 Nov;90(11):1499-506. PubMed PMID: 18978273.

© 2013 American Academy of Orthopaedic Surgeons 2013 Orthopaedic In-Training Examination


216 • American Academy of Orthopaedic Surgeons

Question 57
Advantages of a locking plate implant over a 95-degree angled blade plate for fixation of supracondylar
femur fractures include

1. a higher union rate.


2. a lower implant cost.
3. a lower overall complication rate.
4. a lower rate of prominent hardware requiring removal.
5. improved ability to use with associated coronal fractures.

PREFERRED RESPONSE: 5

RECOMMENDED READINGS
Vallier HA, Immler W. Comparison of the 95-degree angled blade plate and the locking condylar plate
for the treatment of distal femoral fractures. J Orthop Trauma. 2012 Jun;26(6):327-32. PubMed PMID:
22183200.
Gwathmey FW Jr, Jones-Quaidoo SM, Kahler D, Hurwitz S, Cui Q. Distal femoral fractures: current
concepts. J Am Acad Orthop Surg. 2010 Oct;18(10):597-607. Review. PubMed PMID: 20889949.

© 2013 American Academy of Orthopaedic Surgeons 2013 Orthopaedic In-Training Examination


SECTION 11: Musculoskeletal Trauma • 217

Figure 63a Figure 63b

Question 63
A 55-year-old man has a draining wound at the end of his transfemoral amputation residual limb. He
reports that he sustained a “compound fracture” of his thigh bone approximately 30 years ago, requiring
amputation and rodding of a fracture near his hip. His wound drains intermittently and has done so since
his amputation. Intermittent administration of oral antibiotics temporarily ceases wound drainage, but the
drainage returns after antibiotics are stopped. Wound culture reveals Pseudomonas aeruginosa, which is
sensitive to fluoroquinolones, carbapenems, aminoglycosides, and cephalosporins. Radiographs of the
residual limb are seen in Figures 63a and 63b. What is the recommended treatment?

1. Administration of oral ciprofloxacin for 3 months


2. Administration of oral ciprofloxacin for the rest of his life
3. Surgical debridement and irrigation with implant removal and postsurgical ciprofloxacin for 3
months
4. Surgical debridement and irrigation with implant removal, placement of a
gentamicin-impregnated polymethylmethacrylate medullary rod, and postsurgical
ciprofloxacin for 3 months
5. Surgical debridement and irrigation with implant removal, sinus tract biopsy, placement of
a gentamicin-impregnated polymethylmethacrylate medullary rod, and postsurgical
iprofloxacin for 3 months

PREFERRED RESPONSE: 5

RECOMMENDED READINGS
McGrory JE, Pritchard DJ, Unni KK, Ilstrup D, Rowland CM. Malignant lesions arising in chronic
osteomyelitis. Clin Orthop Relat Res. 1999 May;(362):181-9. PubMed PMID: 10335297.
Paley D, Herzenberg JE. Intramedullary infections treated with antibiotic cement rods: preliminary results
in nine cases. J Orthop Trauma. 2002 Nov-Dec;16(10):723-9. PubMed PMID: 12439196.
Riel RU, Gladden PB. A simple method for fashioning an antibiotic cement-coated interlocking
intramedullary nail. Am J Orthop (Belle Mead NJ). 2010 Jan;39(1):18-21. PubMed PMID: 20305835.

© 2013 American Academy of Orthopaedic Surgeons 2013 Orthopaedic In-Training Examination


218 • American Academy of Orthopaedic Surgeons

Figure 73a Figure 73b Figure 73c

Question 73
Figures 73a through 73c are the current radiographs of a 35-year-old woman who fractured her ankle 3
years ago. Her course after surgery was complicated by wound dehiscence over her fibula plate. She had
hardware removed and saucerization of her fibula at 9 months. She is now experiencing pain reproduced
with dorsiflexion/plantar flexion that limits all of her daily living activities. She is unable to obtain a
plantigrade foot with knee extension, has no pain with inversion/eversion, and has well-healed wounds.
Laboratory studies show that her erythrocyte sedimentation rate and C-reactive protein levels are within
defined limits. What is the best treatment option?

1. Total ankle replacement


2. Tibiotalar arthrodesis
3. Tibiotalar and subtalar arthrodesis
4. Tibiotalar arthrodesis with gastrocnemius recession
5. Tibiotalar and subtalar arthrodesis with gastrocnemius recession

PREFERRED RESPONSE: 4

RECOMMENDED READINGS
Hendrickx RP, Stufkens SA, de Bruijn EE, Sierevelt IN, van Dijk CN, Kerkhoffs GM. Medium- to long-
term outcome of ankle arthrodesis. Foot Ankle Int. 2011 Oct;32(10):940-7. PubMed PMID: 22224322.
Bai LB, Lee KB, Song EK, Yoon TR, Seon JK. Total ankle arthroplasty outcome comparison for
post-traumatic and primary osteoarthritis. Foot Ankle Int. 2010 Dec;31(12):1048-56. PubMed PMID:
21189204.

© 2013 American Academy of Orthopaedic Surgeons 2013 Orthopaedic In-Training Examination


SECTION 11: Musculoskeletal Trauma • 219

Figure 82

Question 82
The fracture shown in Figure 82 is scheduled to be fixed with a retrograde nail. An arthrotomy should be
performed during the procedure because it

1. ensures proper nail depth.


2. provides control of the distal fragment.
3. allows assessment for occult infection.
4. allows protection of the polyethylene liner.
5. allows assessment of the loosening component requiring revision.

PREFERRED RESPONSE: 4

RECOMMENDED READINGS
Ellis TJ, White RR, Lhowe DW. Periprosthetic fractures. In: Schmidt AH, Teague DC, eds. Orthopaedic
Knowledge Update: Trauma 4. Rosemont, IL: American Academy of Orthopaedic Surgeons; 2010:569-
577.
McLaren AC, Dupont JA, Schroeber DC. Open reduction internal fixation of supracondylar fractures
above total knee arthroplasties using the intramedullary supracondylar rod. Clin Orthop Relat Res. 1994
May;(302):194-8. PubMed PMID: 8168300.
Haidukewych GJ. Periprosthetic distal femur fracture: plate versus nail fixation. Opinion: open reduction
internal fixation. J Orthop Trauma. 2007 Mar;21(3):219-20. PubMed PMID: 17473762.

© 2013 American Academy of Orthopaedic Surgeons 2013 Orthopaedic In-Training Examination


220 • American Academy of Orthopaedic Surgeons

Figure 87a Figure 87b

Question 87
A 52-year-old woman sustained a closed bimalleolar ankle fracture. She was treated with open reduction
and internal fixation. A syndesmotic screw was added; however, there is persistent asymmetry of the
ankle mortise as shown in Figures 87a and 87b. What is the most likely reason for this finding?

1. The syndesmosis is malreduced.


2. The lateral malleolus is malreduced.
3. The posterior tibial tendon is entrapped in the medial joint.
4. The deltoid ligament is interposed in the medial joint space.
5. An osteochondral fragment is entrapped in the joint.

PREFERRED RESPONSE: 2

RECOMMENDED READINGS
Weber BG, Simpson LA: Corrective lengthening osteotomy of the fibula. Clin Orthop Relat Res 1985;
199:61-67. PubMed PMID: 4042497.
Chu A, Weiner L. Distal fibula malunions. J Am Acad Ortho Surg April 2009;17:220-230. PubMed
PMID: 19307671.

© 2013 American Academy of Orthopaedic Surgeons 2013 Orthopaedic In-Training Examination


SECTION 11: Musculoskeletal Trauma • 221

Question 90
The World Health Organization Fracture Risk Assessment Tool (FRAX) calculates which fracture risk?

1. 5-year risk for hip fracture


2. 5-year risk for distal radius fracture
3. 5-year risk for any fragility fracture
4. 10-year risk for hip fracture
5. 10-year risk for distal radius fracture

PREFERRED RESPONSE: 4

RECOMMENDED READINGS
Unnanuntana A, Gladnick BP, Donnelly E, Lane JM. The assessment of fracture risk. J Bone Joint Surg
Am. 2010 Mar;92(3):743-53. Review. PubMed PMID: 20194335.
Ekman EF. The role of the orthopaedic surgeon in minimizing mortality and morbidity associated with
fragility fractures. J Am Acad Orthop Surg. 2010 May;18(5):278-85. Review. PubMed PMID: 20435878.

© 2013 American Academy of Orthopaedic Surgeons 2013 Orthopaedic In-Training Examination


222 • American Academy of Orthopaedic Surgeons

Figure 102

Question 102
Figure 102 is an intraoperative figure taken during fixation of a right lateral tibial plateau fracture luxation.
Which structure is indicated by the arrow?

1. Iliotibial band
2. Popliteus tendon
3. Medial meniscus
4. Lateral meniscus
5. Lateral collateral ligament

PREFERRED RESPONSE: 4

RECOMMENDED READINGS
Higgins TF, Severson EP. Tibial plateau fractures. In: Schmidt AH, Teague DC, eds. Orthopaedic
Knowledge Update: Trauma 4. Rosemont, IL: American Academy of Orthopaedic Surgeons; 2010:475-
486.
Gardner MJ, Yacoubian S, Geller D, Pode M, Mintz D, Helfet DL, Lorich DG. Prediction of soft-tissue
injuries in Schatzker II tibial plateau fractures based on measurements of plain radiographs. J Trauma.
2006 Feb;60(2):319-23; discussion 324. PubMed PMID: 16508489.

© 2013 American Academy of Orthopaedic Surgeons 2013 Orthopaedic In-Training Examination


SECTION 11: Musculoskeletal Trauma • 223

Question 107
If a physician elects to shorten a femur by 4 cm for traumatic bone loss treatment and places an
intramedullary nail for fixation, which deformity will be created in the lower extremity?

1. Patella alta
2. Medial mechanical axis deviation
3. Lateral mechanical axis deviation
4. Increased anatomic tibiofemoral angle
5. Translation of the anatomical axis of the femur

PREFERRED RESPONSE: 2

RECOMMENDED READINGS
Paley DP. Principles of Deformity Correction. New York, NY: Springer-Verlag; 2002:1-17.
Kasis AG, Stockley I, Saleh M. External fixator-assisted acute shortening with internal fixation for leg
length discrepancy after total hip replacement. Strategies Trauma Limb Reconstr. 2008 Apr;3(1):35-8.
Epub 2008 Apr 4. PubMed PMID: 18427922.

© 2013 American Academy of Orthopaedic Surgeons 2013 Orthopaedic In-Training Examination


224 • American Academy of Orthopaedic Surgeons

Video 112 (click to play)

Question 112
The vessel ligated during the Stoppa approach for acetabular fracture fixation (Video 112) is an
anastomosis of which structures?

1. Femoral and obturator


2. Internal iliac and obturator
3. Internal and external obturator
4. External iliac and obturator
5. Superior gluteal and obturator

PREFERRED RESPONSE: 4

RECOMMENDED READINGS
Tornetta P III, Hochwald N, Levine R. Corona mortis. Incidence and location. Clin Orthop Relat Res.
1996 Aug;(329):97-101. PubMed PMID: 8769440.
Archdeacon MT, Kazemi N, Guy P, Sagi HC. The modified Stoppa approach for acetabular fracture. J Am
Acad Orthop Surg. 2011 Mar;19(3):170-5. PubMed PMID: 21368098.

© 2013 American Academy of Orthopaedic Surgeons 2013 Orthopaedic In-Training Examination


SECTION 11: Musculoskeletal Trauma • 225

Figure 117

Question 117
The radiograph seen in Figure 117 reveals a submuscular plate placement with locking screws for fixation.
The biomechanics of the construct can be best described as

1. stiff and axially stable.


2. stiff and axially unstable.
3. flexible and axially stable.
4. flexible and axially unstable.
5. flexible and rotationally unstable.

PREFERRED RESPONSE: 3

RECOMMENDED READINGS
Graves M, Nork SE. Fractures of the humerus. In: Schmidt AH, Teague DC, eds. Orthopaedic Knowledge
Update: Trauma 4. Rosemont, IL: American Academy of Orthopaedic Surgeons; 2010:201-224.
Livani B, Belangero WD. Bridging plate osteosynthesis of humeral shaft fractures. Injury. 2004
Jun;35(6):587-95. PubMed PMID: 15135278.

Question 120
A 22-year-old man was an unrestrained driver who was ejected from his car during a rollover motor
vehicle crash. He sustained a closed head injury, multiple closed right rib fractures with an ipsilateral
pneumothorax, and an open midshaft right tibia fracture. The tibia wound measures approximately 3 mm
in length and is free of gross contamination. What is the most important factor shown to minimize risk for
infection at the site of an open tibia fracture?

1. Transfer to a Level I trauma center within 3 hours


2. Intravenous antibiotic administration within 3 hours
3. Irrigation and debridement of the open fracture wound within 6 hours
4. Open reduction with plate-and-screw fixation at the index tip within 6 hours
5. Tibia wound irrigation within 3 hours with a solution containing bacitracin

PREFERRED RESPONSE: 2

© 2013 American Academy of Orthopaedic Surgeons 2013 Orthopaedic In-Training Examination


226 • American Academy of Orthopaedic Surgeons

RECOMMENDED READINGS
Pollak AN, Jones AL, Castillo RC, Bosse MJ, MacKenzie EJ; LEAP Study Group. The relationship
between time to surgical debridement and incidence of infection after open high-energy lower extremity
trauma. J Bone Joint Surg Am. 2010 Jan;92(1):7-15. PubMed PMID: 20048090.
Patzakis MJ, Wilkins J. Factors influencing infection rate in open fracture wounds. Clin Orthop Relat Res.
1989 Jun;(243):36-40. PubMed PMID: 2721073.

Question 123
A 68-year-old woman fell and sustained a displaced femoral neck fracture. She is a community ambulator
and enjoys playing tennis weekly. Which treatment will provide her with the best hip function?

1. Hip resurfacing
2. Hemiarthroplasty
3. Total hip arthroplasty
4. Internal fixation with cannulated screws
5. Internal fixation with a sliding hip screw and an antirotation screw

PREFERRED RESPONSE: 3

RECOMMENDED READINGS
Avery PP, Baker RP, Walton MJ, Rooker JC, Squires B, Gargan MF, Bannister GC. Total hip replacement
and hemiarthroplasty in mobile, independent patients with a displaced intracapsular fracture of the femoral
neck: a seven- to ten-year follow-up report of a prospective randomised controlled trial. J Bone Joint Surg
Br. 2011 Aug;93(8):1045-8. PubMed PMID: 21768626.
Hedbeck CJ, Enocson A, Lapidus G, Blomfeldt R, Törnkvist H, Ponzer S, Tidermark J. Comparison of
bipolar hemiarthroplasty with total hip arthroplasty for displaced femoral neck fractures: a concise four-
year follow-up of a randomized trial. J Bone Joint Surg Am. 2011 Mar 2;93(5):445-50. PubMed PMID:
21368076.

Question 128
A 23-year-old man was tackled while playing football. He felt a “pop” in his knee and noted significant
deformity. Examination reveals a closed posterior knee dislocation that is irreducible despite adequate
sedation. He is unable to dorsiflex his toes or ankle. His ankle-brachial index is 0.6. What is the next
most appropriate treatment step?

1. Surgical intervention
2. Splint and monitor peripheral pulse oximetry
3. Magnetic resonance angiography
4. Computed tomography angiography
5. Standard angiography

PREFERRED RESPONSE: 1
© 2013 American Academy of Orthopaedic Surgeons 2013 Orthopaedic In-Training Examination
SECTION 11: Musculoskeletal Trauma • 227

RECOMMENDED READINGS
Rihn JA, Groff YJ, Harner CD, Cha PS. The acutely dislocated knee: evaluation and management. J Am
Acad Orthop Surg. 2004 Sep-Oct;12(5):334-46. Review. PubMed PMID: 15469228.
Patterson BM, Agel J, Swiontkowski MF, Mackenzie EJ, Bosse MJ; LEAP Study Group. Knee
dislocations with vascular injury: outcomes in the Lower Extremity Assessment Project (LEAP) Study. J
Trauma. 2007 Oct;63(4):855-8. PubMed PMID: 18090017.

Question 132
A 24-year-old man sustained a medial tibial plateau fracture (Schatzker type IV) after being involved in a
motor vehicle-pedestrian collision. What is the best next step?

1. An MRI scan
2. Ankle brachial index
3. Immediate open reduction and internal fixation
4. Closed reduction and percutaneous screw fixation
5. Definitive treatment with a hybrid external fixator

PREFERRED RESPONSE: 2

RECOMMENDED READINGS
Berkson EM, Virkus WW. High-energy tibial plateau fractures. J Am Acad Orthop Surg. 2006
Jan;14(1):20-31. Review. PubMed PMID: 16394164.
Gardner MJ, Yacoubian S, Geller D, Suk M, Mintz D, Potter H, Helfet DL, Lorich DG. The incidence
of soft tissue injury in operative tibial plateau fractures: a magnetic resonance imaging analysis of 103
patients. J Orthop Trauma. 2005 Feb;19(2):79-84. PubMed PMID: 15677922.

© 2013 American Academy of Orthopaedic Surgeons 2013 Orthopaedic In-Training Examination


228 • American Academy of Orthopaedic Surgeons

Figure 135a Figure 135b Video 135c (click to play)

Question 135
A 45-year-old woman sustained a fall from height and has the injury shown in Figures 135a and 135b.
A 3-dimensional reconstruction CT scan is shown in Figure 135c. Joint-spanning external fixation is
applied on the day of injury. Ten days later, her skin is acceptable for definitive fixation. What is the most
appropriate type of fixation for her fracture?

1. Percutaneous screws and cast


2. Conversion to a circular fixator
3. Medial and anterolateral locked plates
4. Medial and anterolateral nonlocked plates
5. Lateral locked plate and medial malleolus screws

PREFERRED RESPONSE: 4

RECOMMENDED READINGS
Sirkin M, Sanders R, DiPasquale T, Herscovici D Jr. A staged protocol for soft tissue management in the
treatment of complex pilon fractures. J Orthop Trauma. 2004 Sep;18(8 Suppl):S32-8. PubMed PMID:
15472563.
Collinge C, Prayson M. Open reduction and internal fixation of pilon fractures. In: Tornetta P, Williams
GR, Ramsey ML, Hunt TR, Wiesel SW, eds. Operative Techniques in Orthopaedic Trauma Surgery.
Philadelphia, PA: Lippincott Williams & Wilkins; 2011:483-498.

© 2013 American Academy of Orthopaedic Surgeons 2013 Orthopaedic In-Training Examination


SECTION 11: Musculoskeletal Trauma • 229

Figure 137

Question 137
The fracture seen in Figure 137 was most likely caused by what type of mechanism?

1. Direct impact to the fibula


2. Abduction of the foot relative to the tibia
3. Adduction of the foot relative to the tibia
4. Internal rotation of the foot relative to the tibia
5. External rotation of the foot relative to the tibia

PREFERRED RESPONSE: 2

RECOMMENDED READINGS
Barei DP Crist BD. Fractures of the ankle and distal tibial pilon. In: Schmidt AH, Teague DC, eds.
Orthopaedic Knowledge Update: Trauma 4. Rosemont, IL: American Academy of Orthopaedic Surgeons;
2010:499-518.
Lauge-Hansen N. Fractures of the ankle. II. Combined experimental-surgical and experimental-
roentgenologic investigations. Arch Surg. 1950 May;60(5):957-85. PubMed PMID: 15411319.

© 2013 American Academy of Orthopaedic Surgeons 2013 Orthopaedic In-Training Examination


230 • American Academy of Orthopaedic Surgeons

Question 139
The best way to avoid sentinel event errors is through better

1. training.
2. staffing ratios.
3. communication.
4. patient assessment.
5. availability of information.

PREFERRED RESPONSE: 3

RECOMMENDED READINGS
The Joint Commission. 2007 National Patient Safety Goals. The Joint Commission, 2007. Viewed 27
October 2009. http://www.pharmacytimes.com/publications/issue/2007/2007-02/2007-02-6294 (Accessed
7/10/2012)
Leonard M, Graham S, Bonacum D. The human factor: the critical importance of effective teamwork
and communication in providing safe care. Qual Saf Health Care. 2004 Oct;13 Suppl 1:i85-90. PubMed
PMID: 15465961.
VA National Center for Patient Safety Executive Summary, 2007. Joint Commission Sentinel Event Alert
Issue 12 http://www.patientsafety.gov/ (Accessed 7/10/2012)

Question 144
A 22-year-old man wants a second opinion 3 weeks after intramedullary nailing of a comminuted
diaphyseal femoral shaft fracture. Examination reveals his injured leg has 26 degrees’ more external
rotation than the contralateral limb and is 3 cm shorter based on a block measurement. He should be
advised to

1. let the fracture unite as is because there is sufficient hip rotation to accommodate the external
rotation deformity, and a small shoe lift can accommodate for the limb length discrepancy.
2. let the fracture unite, and if he later finds it bothersome, consider a corrective osteotomy of the
injured femur for correction of the deformity.
3. let the fracture unite because there is sufficient hip rotation to accommodate the external
rotation deformity; if he later finds the leg length discrepancy bothersome, he should consider
contralateral closed femoral shortening.
4. consider revision surgery to correct the rotational deformity but not alter length because this
may impair fracture union if performed at this time.
5. consider revision surgery to correct both the rotational deformity and leg length discrepancy.

PREFERRED RESPONSE: 5

© 2013 American Academy of Orthopaedic Surgeons 2013 Orthopaedic In-Training Examination


SECTION 11: Musculoskeletal Trauma • 231

RECOMMENDED READINGS
Lindsey JD, Krieg JC. Femoral malrotation following intramedullary nail fixation. J Am Acad Orthop
Surg. 2011 Jan;19(1):17-26. PubMed PMID: 21205764.
Jaarsma RL, Pakvis DF, Verdonschot N, Biert J, van Kampen A. Rotational malalignment after
intramedullary nailing of femoral fractures. J Orthop Trauma. 2004 Aug;18(7):403-9. PubMed PMID:
15289684.

Question 149
On an anteroposteriorly directed fluoroscopic radiograph, the appropriate entry point for an intramedullary
tibia nail being used for fixation of a proximal third diaphyseal tibial fracture is ideally positioned

1. centered between the medial and lateral tibial eminences.


2. in line with the lateral border of the lateral tibial eminence.
3. in line with the medial border of the lateral tibial eminence.
4. in line with the lateral border of the medial tibial eminence.
5. in line with the medial border of the medial tibial eminence.

PREFERRED RESPONSE: 3

RECOMMENDED READINGS
McConnell T, Tornetta P III, Tilzey J, Casey D. Tibial portal placement: the radiographic correlate of the
anatomic safe zone. J Orthop Trauma. 2001Mar-Apr;15(3):207-9. PubMed PMID: 11265012.
Song SJ, Jeong BO. Three-dimensional analysis of the intramedullary canal axis of tibia: clinical
relevance to tibia intramedullary nailing. Arch Orthop Trauma Surg. 2010 Jul;130(7):903-7. Epub 2009
Nov 3. PubMed PMID: 19885665.

© 2013 American Academy of Orthopaedic Surgeons 2013 Orthopaedic In-Training Examination


232 • American Academy of Orthopaedic Surgeons

Figure 156a Figure 156b

Question 156
Figures 156a and 156b are the radiographs of a 38-year-old man with diabetes mellitus who fell 8 feet
from a ladder and sustained an isolated closed injury of his leg. Examination revealed swollen but
soft compartments. His neurovascular examination was unremarkable. A damage-control fixator was
initially applied, and his soft-tissue envelope is now amenable to further intervention. What is the most
appropriate treatment?

1. Conversion to a peri-articular hybrid frame


2. Open reduction and internal fixation with a lateral locking plate
3. Open reduction and internal fixation with a lateral nonlocking plate
4. Open reduction and internal fixation with medial and lateral plates
5. Open reduction and internal fixation with posteromedial and lateral plates

PREFERRED RESPONSE: 5

RECOMMENDED READINGS
Barei DP, O'Mara TJ, Taitsman LA, Dunbar RP, Nork SE. Frequency and fracture morphology of
the posteromedial fragment in bicondylar tibial plateau fracture patterns. J Orthop Trauma. 2008
Mar;22(3):176-82. PubMed PMID: 18317051.
Barei DP, Nork SE, Mills WJ, Coles CP, Henley MB, Benirschke SK. Functional outcomes of severe
bicondylar tibial plateau fractures treated with dual incisions and medial and lateral plates. J Bone Joint
Surg Am. 2006 Aug;88(8):1713-21. PubMed PMID: 16882892.

© 2013 American Academy of Orthopaedic Surgeons 2013 Orthopaedic In-Training Examination


SECTION 11: Musculoskeletal Trauma • 233

Figure 160

Question 160
Figure 160 is the intrasurgical photo of a 35-year-old woman with an open tibial fracture. Examination
reveals no Doppler signal of the peroneal artery or anterior tibial artery. However, flow in her posterior
tibial artery is detected by Doppler. According to the Gustilo-Anderson classification system, the fracture
should be classified as

1. type I.
2. type II.
3. type IIIA.
4. type IIIB.
5. type IIIC.

PREFERRED RESPONSE: 4

RECOMMENDED READINGS
Gustilo RB, Anderson JT. Prevention of infection in the treatment of one thousand and twenty-five
open fractures of long bones: retrospective and prospective analyses. J Bone Joint Surg Am. 1976
Jun;58(4):453-8. PubMed PMID: 773941.
Gustilo RB, Mendoza RM, Williams DN. Problems in the management of type III (severe) open fractures:
a new classification of type III open fractures. J Trauma. 1984 Aug;24(8):742-6. PubMed PMID: 6471139.

© 2013 American Academy of Orthopaedic Surgeons 2013 Orthopaedic In-Training Examination


234 • American Academy of Orthopaedic Surgeons

Question 163
To minimize complications and to maximize the likelihood of successful outcomes after percutaneous
fixation of displaced extension-type supracondylar humeral fractures in children, the physician should

1. use a divergent wire technique with wires placed medially.


2. use a divergent wire technique with wires placed laterally.
3. use a crossed-wire technique with wires placed laterally and medially.
4. apply a postsurgical circumferential cast with the elbow fully extended to prevent postsurgical
displacement.
5. apply a postsurgical circumferential cast with the elbow flexed past 90 degrees to prevent
postsurgical displacement.

PREFERRED RESPONSE: 2

RECOMMENDED READINGS
Slobogean BL, Jackman H, Tennant S, Slobogean GP, Mulpuri K. Iatrogenic ulnar nerve injury after
the surgical treatment of displaced supracondylar fractures of the humerus: number needed to harm, a
systematic review. J Pediatr Orthop. 2010 Jul-Aug;30(5):430-6. Review. PubMed PMID: 20574258.
Woratanarat P, Angsanuntsukh C, Rattanasiri S, Attia J, Woratanarat T, Thakkinstian A. Meta-analysis of
pinning in supracondylar fracture of the humerus in children. J Orthop Trauma. 2012 Jan;26(1):48-53.
PubMed PMID: 21909033.
Omid R, Choi PD, Skaggs DL. Supracondylar humeral fractures in children. J Bone Joint Surg Am. 2008
May;90(5):1121-32. Review. PubMed PMID: 18451407.
McKeon KE, O'Donnell JC, Bashyal R, Hou CC, Luhmann SJ, Dobbs MB, Gordon JE. Immobilization
after pinning of supracondylar distal humerus fractures in children: use of the A-frame cast. J Pediatr
Orthop. 2012 Jan-Feb;32(1):e1-5. PubMed PMID: 22173398.

Question 167
A 24-year-old football player sustained an injury to his left foot when another player fell directly on his
heel. He is unable to bear weight, but radiograph findings were negative. He is exquisitely tender at the
midfoot. What is the best next diagnostic study?

1. A CT scan
2. A bone scan
3. Weight-bearing views
4. Contralateral foot radiographs
5. Repeat radiograph in 2 weeks

PREFERRED RESPONSE: 3

© 2013 American Academy of Orthopaedic Surgeons 2013 Orthopaedic In-Training Examination


SECTION 11: Musculoskeletal Trauma • 235

RECOMMENDED READINGS
Nunley JA, Vertullo CJ. Classification, investigation, and management of midfoot sprains: Lisfranc
injuries in the athlete. Am J Sports Med. 2002 Nov-Dec;30(6):871-8. PubMed PMID: 12435655.
Watson TS, Shurnas PS, Denker J. Treatment of Lisfranc joint injury: current concepts. J Am Acad Orthop
Surg. 2010 Dec;18(12):718-28. Review. PubMed PMID: 21119138.

Figure 172a Figure 172b

Question 172
What is the most common nerve injury seen in Figures 172a and 172b?

1. Ulnar
2. Radial
3. Median
4. Anterior interosseous
5. Lateral antebrachial cutaneous

PREFERRED RESPONSE: 4

RECOMMENDED READINGS
Abzug JM, Herman MJ. Management of supracondylar humerus fractures in children: current concepts. J
Am Acad Orthop Surg. 2012 Feb;20(2):69-77. Review. PubMed PMID: 22302444.
Babal JC, Mehlman CT, Klein G. Nerve injuries associated with pediatric supracondylar humeral
fractures: a meta-analysis. J Pediatr Orthop. 2010 Apr-May;30(3):253-63. PubMed PMID: 20357592.

© 2013 American Academy of Orthopaedic Surgeons 2013 Orthopaedic In-Training Examination


236 • American Academy of Orthopaedic Surgeons

Question 176
The risk for developing complex regional pain syndrome after surgery to the foot and ankle or the wrist
can be decreased through the use of

1. capsaicin.
2. vitamin C.
3. vitamin D and calcium.
4. dexamethasone block.
5. multimodal pain therapy.

PREFERRED RESPONSE: 2

RECOMMENDED READINGS
Zollinger PE, Tuinebreijer WE, Breederveld RS, Kreis RW. Can vitamin C prevent complex regional pain
syndrome in patients with wrist fractures? A randomized, controlled, multicenter dose-response study. J
Bone Joint Surg Am. 2007 Jul;89(7):1424-31. PubMed PMID: 17606778.
Besse JL, Gadeyne S, Galand-Desmé S, Lerat JL, Moyen B. Effect of vitamin C on prevention of complex
regional pain syndrome type I in foot and ankle surgery. Foot Ankle Surg. 2009;15(4):179-82. Epub 2009
Apr 5. PubMed PMID: 19840748.

Question 186
What is the most common complication seen after patellar fracture open reduction and internal fixation?

1. Loss of reduction
2. Knee extensor lag
3. Symptomatic implants
4. Flexion contracture exceeding 5 degrees
5. Extension contracture exceeding 15 degrees

PREFERRED RESPONSE: 3

RECOMMENDED READINGS
Lebrun CT, Langford JR, Sagi HC. Functional outcomes after operatively treated patella fractures. J
Orthop Trauma. 2012 Jul;26(7):422-6. PubMed PMID: 22183197.
Melvin JS, Mehta S. Patellar fractures in adults. J Am Acad Orthop Surg. 2011 Apr;19(4):198-207.
Review. PubMed PMID: 21464213.

© 2013 American Academy of Orthopaedic Surgeons 2013 Orthopaedic In-Training Examination


SECTION 11: Musculoskeletal Trauma • 237

Question 188
An athletic 30-year-old sustained multiple injuries in a high-speed motor vehicle collision that resulted in
a loss of approximately 30% of blood volume. On arrival to the emergency department, the heart rate is
100 and blood pressure is 104/62. The best means with which to evaluate true hemodynamic status is

1. hematocrit.
2. serial heart rate.
3. serial blood pressure with a manual cuff.
4. serial blood pressure with an arterial line.
5. lactate and base deficit levels.

PREFERRED RESPONSE: 5

RECOMMENDED READINGS
Hak DJ, Stahel PF, Giannoudis P. Pathophysiology of the polytrauma patient. In: Schmidt AH, Teague
DC, eds. Orthopaedic Knowledge Update: Trauma 4. Rosemont, IL: American Academy of Orthopaedic
Surgeons; 2010:117–131.
Rossaint R, Cerny V, Coats TJ, Duranteau J, Fernández-Mondéjar E, Gordini G, Stahel PF, Hunt BJ,
Neugebauer E, Spahn DR. Key issues in advanced bleeding care in trauma. Shock. 2006 Oct;26(4):322-
31. Review. PubMed PMID: 16980877.

© 2013 American Academy of Orthopaedic Surgeons 2013 Orthopaedic In-Training Examination


238 • American Academy of Orthopaedic Surgeons

Figure 196

Question 196
Which virtual hinge shown in Figure 196 will gain the most length with the least amount of translation
and angulation at the end of deformity correction?

1. A
2. B
3. C
4. D
5. E

PREFERRED RESPONSE: 2

RECOMMENDED READINGS
Paley DP, ed. Principles of Deformity Correction. New York, NY: Springer-Verlag; 2002:99-154.
Feldman DS, Shin SS, Madan S, Koval KJ. Correction of tibial malunion and nonunion with six-axis
analysis deformity correction using the Taylor Spatial Frame. J Orthop Trauma. 2003 Sep;17(8):549-54.
PubMed PMID: 14504575.

© 2013 American Academy of Orthopaedic Surgeons 2013 Orthopaedic In-Training Examination


SECTION 11: Musculoskeletal Trauma • 239

Figure 200a Figure 200b

Question 200
Figures 200a and 200b are the radiographs of an 82-year-old woman who fell on a flexed knee. She has
no other injuries and was able to ambulate without assistance before her fall. The recommended treatment
to optimize her quality of life consists of

1. external fixation.
2. revision arthroplasty.
3. open reduction and internal fixation.
4. closed reduction and casting.
5. closed reduction and fracture bracing.

PREFERRED RESPONSE: 3

RECOMMENDED READINGS
Ricci WM, Bolhofner BR, Loftus T, Cox C, Mitchell S, Borrelli J Jr. Indirect reduction and plate fixation,
without grafting, for periprosthetic femoral shaft fractures about a stable intramedullary implant. J Bone
Joint Surg Am. 2005 Oct;87(10):2240-5. PubMed PMID: 16203889.
Streubel PN, Gardner MJ, Morshed S, Collinge CA, Gallagher B, Ricci WM. Are extreme distal
periprosthetic supracondylar fractures of the femur too distal to fix using a lateral locked plate? J Bone
Joint Surg Br. 2010 Apr;92(4):527-34. PubMed PMID: 20357329.

© 2013 American Academy of Orthopaedic Surgeons 2013 Orthopaedic In-Training Examination


240 • American Academy of Orthopaedic Surgeons

Figure 201a Figure 201b Figure 201c

Figure 201d

Question 201
Figure 201a is the radiograph of a patient with an open femur fracture who had debridement and nailing
with antibiotic beads as shown in Figure 201b. The patient notices leg deformity while lying in bed.
Subsequent CT scans are shown in Figures 201c and 201d. In addition to being fixed short, what other
malalignment, if any, is seen?

1. Fixed with approximately 24 degrees’ internal rotation deformity


2. Fixed with approximately 24 degrees’ external rotation deformity
3. Fixed with approximately 31 degrees’ internal rotation deformity
4. Fixed with approximately 31 degrees’ external rotation deformity
5. No malalignment; deformity is attributable to postsurgical pain and reflex relaxation

PREFERRED RESPONSE: 1

RECOMMENDED READINGS
Ricci WM, Gruen GS, Summers H, Siska PA. Fractures of the femoral diaphysis. In: Schmidt AH, Teague
DC, eds. Orthopaedic Knowledge Update: Trauma 4. Rosemont, IL: American Academy of Orthopaedic
Surgeons; 2010:431-444.
Jaarsma RL, van Kampen A. Rotational malalignment after fractures of the femur. J Bone Joint Surg Br.
2004 Nov;86(8):1100-4. Review. PubMed PMID: 15568519.

© 2013 American Academy of Orthopaedic Surgeons 2013 Orthopaedic In-Training Examination


SECTION 11: Musculoskeletal Trauma • 241

Question 204
A 23-year-old man had a laparotomy and splenectomy with packing of the abdomen after a motorcycle
collision. Laboratory studies show a hemoglobin level of 7.1 g/dL (reference range [rr], 14.0-17.5 g/
dL) and a lactate level of 8.0 mmol/L (rr, 0.6-1.7 mmol/L). He also has a left humeral fracture, an
anteroposterior compression I pelvic fracture, bilateral distal third femur fractures, and an open Gustilo-
type IIIA tibial diaphysis fracture with moderate contamination. What is the most appropriate treatment to
administer before leaving the operating room?

1. Saline lavage and splinting of the tibia and knee immobilizers of both femurs
2. Betadine dressing and splinting of the tibia with unlocked retrograde nailing of both femurs
3. Betadine dressing and external fixation of the tibia and knee immobilizers of both femurs
4. Irrigation and debridement and external fixation of the tibia and external fixation of both
femurs
5. Irrigation and debridement and external fixation of the tibia and unlocked retrograde nailing of
both femurs

PREFERRED RESPONSE: 4

RECOMMENDED READINGS
Hak DJ, Stahel PF, Giannoudis P. Pathophysiology of the polytrauma patients. In. Schmidt AH, Teague
DC, eds. Orthopaedic Knowledge Update: Trauma 4. Rosemont, IL: American Academy of Orthopaedic
Surgeons; 2010:117-131.
Ricci WM, Gruen GS, Summers H, Siska PA. Fractures of the femoral diaphysis. In: Schmidt AH, Teague
DC, eds. Orthopaedic Knowledge Update: Trauma 4. Rosemont, IL: American Academy of Orthopaedic
Surgeons; 2010:431-444.

© 2013 American Academy of Orthopaedic Surgeons 2013 Orthopaedic In-Training Examination


242 • American Academy of Orthopaedic Surgeons

Figure 207

Question 207
Which nerve identified by the arrow seen in Figure 207 is encountered during fixation of a tibial pilon
fracture?

1. Sural
2. Saphenous
3. Lateral cutaneous
4. Deep peroneal
5. Superficial peroneal

PREFERRED RESPONSE: 5

RECOMMENDED READINGS
Mehta S, Gardner MJ, Barei DP, Benirschke SK, Nork SE. Reduction strategies through the anterolateral
exposure for fixation of type B and C pilon fractures. J Orthop Trauma. 2011 Feb;25(2):116-22. PubMed
PMID: 21245716.
Crist BD, Khazzam M, Murtha YM, Della Rocca GJ. Pilon fractures: advances in surgical management. J
Am Acad Orthop Surg. 2011 Oct;19(10):612-22. Review. PubMed PMID: 21980026.

© 2013 American Academy of Orthopaedic Surgeons 2013 Orthopaedic In-Training Examination


SECTION 11: Musculoskeletal Trauma • 243

Question 208
What is the mechanism of action of tranexamic acid in controlling traumatic hemorrhage?

1. Inhibition of vitamin K reductase


2. Inhibition of topoisomerase II and IV
3. Antithrombin-III selective inhibition of Factor Xa
4. Competitive inhibition of plasminogen activation
5. Stimulation of integrin-mediated platelet adhesion and activation

PREFERRED RESPONSE: 4

RECOMMENDED READINGS
Godier A, Roberts I, Hunt BJ. Tranexamic acid: less bleeding and less thrombosis? Crit Care. 2012 Jun
29;16(3):135. [Epub ahead of print] PubMed PMID: 22748073.
Morrison JJ, Dubose JJ, Rasmussen TE, Midwinter MJ. Military Application of Tranexamic Acid in
Trauma Emergency Resuscitation (MATTERs) Study. Arch Surg. 2012 Feb;147(2):113-9. Epub 2011 Oct
17. PubMed PMID: 22006852.

© 2013 American Academy of Orthopaedic Surgeons 2013 Orthopaedic In-Training Examination


244 • American Academy of Orthopaedic Surgeons

Figure 211a Figure 211b

Question 211
Figures 211a and 211b are the radiographs of a 41-year-old construction worker who sustained a twisting
injury to his right leg. Which injury in the ipsilateral extremity is most commonly associated with this
type of fracture?

1. Lisfranc injury
2. Anterior cruciate ligament injury
3. Proximal tibiofibular joint dislocation
4. Tibial plateau fracture
5. Posterior malleolus fracture

PREFERRED RESPONSE: 5

RECOMMENDED READINGS
Boraiah S, Gardner MJ, Helfet DL, Lorich DG. High association of posterior malleolus fractures with
spiral distal tibial fractures. Clin Orthop Relat Res. 2008 Jul;466(7):1692-8. Epub 2008 Mar 18. PubMed
PMID: 18347885.
Stuermer EK, Stuermer KM. Tibial shaft fracture and ankle joint injury. J Orthop Trauma. 2008
Feb;22(2):107-12. PubMed PMID: 18349778.

© 2013 American Academy of Orthopaedic Surgeons 2013 Orthopaedic In-Training Examination


SECTION 11: Musculoskeletal Trauma • 245

Question 216
A 25-year-old man sustained a closed right knee dislocation in a motor vehicle collision. His pedal
pulses are symmetrical in the emergency department, both before and after reduction of the dislocation.
Angiography can be avoided if

1. his ipsilateral ankle-brachial index is 0.78.


2. he had an absent ipsilateral pedal pulse in the field before arriving at the hospital.
3. he has a slightly cool right foot that becomes warm again over the course of 3 hours.
4. he has normal color and warmth of the right foot with normal pedal pulses for 48 hours.
5. he has a large hematoma that has increased in size during the first 3 hours after admission.

PREFERRED RESPONSE: 4

RECOMMENDED READINGS
Stannard JP, Sheils TM, Lopez-Ben RR, McGwin G Jr, Robinson JT, Volgas DA. Vascular injuries in knee
dislocations: the role of physical examination in determining the need for arteriography. J Bone Joint Surg
Am. 2004 May;86-A(5):910-5. PubMed PMID: 15118031.
Mills WJ, Barei DP, McNair P. The value of the ankle-brachial index for diagnosing arterial injury after
knee dislocation: a prospective study. J Trauma. 2004 Jun;56(6):1261-5. PubMed PMID: 15211135.
Nicandri GT, Chamberlain AM, Wahl CJ. Practical management of knee dislocations: a selective
angiography protocol to detect limb-threatening vascular injuries. Clin J Sport Med. 2009 Mar;19(2):125-
9. Review. PubMed PMID: 19451767.

Question 221
A 24-year-old man had multisystem injuries, including an open left femoral shaft fracture he sustained
after a motorcycle collision. He received 3 liters of crystalloid and 2 units of packed red blood cells.
Urgent debridement and irrigation of his open left femur fracture is planned. Which finding would support
proceeding with definitive fixation of the fracture at the time of debridement?

1. Lactate level of 2.2 mg/dL


2. Platelet count of 70,000
3. Urine output of 20 cc/hour
4. Systolic blood pressure of 90
5. Body temperature of 34.5°C

PREFERRED RESPONSE: 1

RECOMMENDED READINGS
Pape HC, Tornetta P 3rd, Tarkin I, Tzioupis C, Sabeson V, Olson SA. Timing of fracture fixation in
multitrauma patients: the role of early total care and damage control surgery. J Am Acad Orthop Surg
September 2009; 17:541-549. PubMed PMID: 19726738.
Pape HC, Giannoudis PV, Krettek C, Trentz O. Timing of fixation of major fractures in blunt polytrauma:
role of conventional indicators in clinical decision making. J Orthop Trauma Sep 2005;19(8):551-562.
PubMed PMID: 16118563.
© 2013 American Academy of Orthopaedic Surgeons 2013 Orthopaedic In-Training Examination
246 • American Academy of Orthopaedic Surgeons

Question 223
The World Health Organization Safe Surgery Guidelines Checklist requires that when prophylactic
antibiotics are indicated, they should be administered

1. within 30 minutes prior to incision.


2. within 60 minutes prior to incision.
3. within 30 minutes prior to or after incision.
4. within 60 minutes prior to or after incision.
5. only in the operating room once the patient’s allergies, if any, have been confirmed by the
anesthesiologist and circulating nurse.

PREFERRED RESPONSE: 2

RECOMMENDED READINGS
Haynes AB, Weiser TG, Berry WR, Lipsitz SR, Breizat AH, Dellinger EP, Herbosa T, Joseph S, Kibatala
PL, Lapitan MC, Merry AF, Moorthy K, Reznick RK, Taylor B, Gawande AA; Safe Surgery Saves Lives
Study Group. A surgical safety checklist to reduce morbidity and mortality in a global population. N Engl
J Med. 2009 Jan 29;360(5):491-9. Epub 2009 Jan 14. PubMed PMID: 19144931.
World Alliance for Patient Safety. WHO guidelines for safe surgery. Geneva, Switzerland: World Health
Organization; 2008.

© 2013 American Academy of Orthopaedic Surgeons 2013 Orthopaedic In-Training Examination


SECTION 11: Musculoskeletal Trauma • 247

Figure 226a Figure 226b

Figure 226c Video 226d (click to play)

Question 226
A 55-year-old man sustained a right acetabular fracture after a fall from a ladder. Anteroposterior and
Judet radiographs of the pelvis are shown in Figures 226a through 226c, and an axial CT scan of the pelvis
is shown in Figure 226d. The acetabular fracture is best classified as

1. associated T type.
2. associated both column.
3. associated transverse and posterior wall.
4. associated posterior column and posterior wall.
5. associated anterior and posterior hemitransverse.

PREFERRED RESPONSE: 3

© 2013 American Academy of Orthopaedic Surgeons 2013 Orthopaedic In-Training Examination


248 • American Academy of Orthopaedic Surgeons

RECOMMENDED READINGS
Matta JM. Fractures of the acetabulum: accuracy of reduction and clinical results in patients managed
operatively within three weeks after the injury. J Bone Joint Surg Am. 1996 Nov;78(11):1632-45. PubMed
PMID: 8934477.
Borrelli J Jr, Peelle M, McFarland E, Evanoff B, Ricci WM. Computer-reconstructed radiographs are as
good as plain radiographs for assessment of acetabular fractures. Am J Orthop (Belle Mead NJ). 2008
Sep;37(9):455-9; discussion 460. PubMed PMID: 18982180.
Beaulé PE, Dorey FJ, Matta JM. Letournel classification for acetabular fractures. Assessment of
interobserver and intraobserver reliability. J Bone Joint Surg Am. 2003 Sep;85-A(9):1704-9. PubMed
PMID: 12954828.
Letournel E. Acetabulum fractures: classification and management. Clin Orthop Relat Res. 1980
Sep;(151):81-106. PubMed PMID: 7418327.

Question 228
A 78-year-old woman sustained a periprosthetic supercondylar femoral fracture. What is the advantage of
submuscular plating compared with an extensile lateral approach?

1. Decreased rate of infection


2. Decreased risk for nonunion
3. Decreased risk for iatrogenic fracture
4. Improved functional outcome
5. Increased longevity of the component

PREFERRED RESPONSE: 2

RECOMMENDED READINGS
Hou Z, Bowen TR, Irgit K, Strohecker K, Matzko ME, Widmaier J, Smith WR. Locked plating of
periprosthetic femur fractures above total knee arthroplasty. J Orthop Trauma. 2012 Jul;26(7):427-32.
PubMed PMID: 22357080.
Hoffmann MF, Jones CB, Sietsema DL, Koenig SJ, Tornetta P 3rd. Outcome of periprosthetic distal
femoral fractures following knee arthroplasty. Injury. 2012 Jul;43(7):1084-9. Epub 2012 Feb 18. PubMed
PMID: 22348954.

© 2013 American Academy of Orthopaedic Surgeons 2013 Orthopaedic In-Training Examination


SECTION 11: Musculoskeletal Trauma • 249

Figure 235

Question 235
Figure 235 is the radiograph of a 75-year-old woman who is seen in the emergency department following
a low-energy fall. What is the most appropriate treatment based on her radiographic findings?

1. Perform a biopsy of the lesion


2. Stabilize with an intramedullary nail
3. Initiate immediate bisphosphonate therapy
4. Treat with chemotherapy followed by wide resection
5. Obtain a chest CT scan, urine protein electrophoresis, and serum protein electrophoresis

PREFERRED RESPONSE: 2

RECOMMENDED READINGS
Weil YA, Rivkin G, Safran O, Liebergall M, Foldes AJ. The outcome of surgically treated femur fractures
associated with long-term bisphosphonate use. J Trauma. 2011 Jul;71(1):186-90. PubMed PMID:
21610533.
Prasam ML, Ahn J, Helfet DL, Lane JM, Lorich DG. Bisphosphonate-associated femur fractures have
high complication rates with operative fixation. Clin Orthop Relat Res. 2012 Aug;470(8):2295-301.
PubMed PMID: 22669553.

© 2013 American Academy of Orthopaedic Surgeons 2013 Orthopaedic In-Training Examination


250 • American Academy of Orthopaedic Surgeons

Figure 238a Figure 238b

Question 238
Figures 238a and 238b are the radiographs of a 60-year-old woman who fell and sustained a right midshaft
humeral fracture 1 year ago. She was treated in a functional brace for 6 months and has used an electrical
bone stimulator for the past 6 months. She has arm pain and limited use of her left shoulder and elbow.
What is the best treatment option?

1. A reamed intramedullary nail


2. A change to an ultrasound bone stimulator
3. Continued nonsurgical treatment with both functional bracing and electrical bone stimulator
4. Systemic administration of 1-34 teriparatide
5. Compression plating with or without bone graft

PREFERRED RESPONSE: 5

RECOMMENDED READINGS
Abboud JA, Boardman ND III. Shoulder trauma: bone. In: Flynn JM, ed. Orthopaedic Knowledge Update
10. Rosemont, IL: American Academy of Orthopaedic Surgeons; 2011:271-284.
Ring D, Chin K, Taghinia AH, Jupiter JB. Nonunion after functional brace treatment of diaphyseal
humerus fractures. J Trauma. 2007 May;62(5):1157-8. PubMed PMID: 17495717.

© 2013 American Academy of Orthopaedic Surgeons 2013 Orthopaedic In-Training Examination


SECTION 11: Musculoskeletal Trauma • 251

Figure 249a Figure 249b

Figure 249c

Question 249
A 75-year-old woman fell at home and sustained the injury seen in Figures 249a through 249c. What is
the most appropriate treatment option?

1. Stand pivot transfer only


2. Bed rest with bathroom privileges
3. Partial weight bearing on the right
4. Weight bearing only after surgical intervention
5. Bilateral weight bearing as tolerated

PREFERRED RESPONSE: 5

RECOMMENDED READINGS
Sagi HC, Liporace FA. Fractures of the pelvis and acetabulum. In: Flynn JM, ed. Orthopaedic Knowledge
Update 10. Rosemont, IL: American Academy of Orthopaedic Surgeons; 2011:379-397.
Sembler Soles GL, Lien J, Tornetta P III. Nonoperative immediate weightbearing of minimally displaced
lateral compression sacral fractures does not result in displacement. J Orthop Trauma. 2012 Apr 10.
PubMed PMID: 22495523.

© 2013 American Academy of Orthopaedic Surgeons 2013 Orthopaedic In-Training Examination


252 • American Academy of Orthopaedic Surgeons

Figure 261

Question 261
A 30-year-old man sustained the injury seen in Figure 261. According to the Lauge-Hansen Classification
System, the fracture should be classified as

1. pronation-abduction.
2. pronation-adduction.
3. pronation-external rotation.
4. supination-adduction.
5. supination-external rotation.

PREFERRED RESPONSE: 4

RECOMMENDED READINGS
Davidovitch RI, Egol KA. Fractures of the ankle. In: Bucholz RW, Court-Brown CM, Heckman JD,
Tornetta P, eds. Fractures and Dislocations. Philadelphia, PA: Lippincott; 2009:1975-2021.
Graves M. Ankle fractures. In: Flynn JM, ed. Orthopaedic Knowledge Update 10. Rosemont, IL:
American Academy of Orthopaedic Surgeons; 2011:493-505.

© 2013 American Academy of Orthopaedic Surgeons 2013 Orthopaedic In-Training Examination


SECTION 11: Musculoskeletal Trauma • 253

Figure 268

Question 268
The condition shown in Figure 268 has been subject to 2 nailing attempts. The patient is seen 8 months
after the second surgery. What is the most appropriate treatment method?

1. Bone stimulator with vitamin D supplementation


2. In situ noncompressive plating with a bone graft
3. In situ repeat intramedullary nailing with a bone graft
4. Corrective alignment with exchange nailing with a bone graft
5. Corrective alignment and compression plating without a bone graft

PREFERRED RESPONSE: 5

RECOMMENDED READINGS
Bolhofner BR, Finnegan M, Lundy DW. Nonunions and malunions. In: Schmidt AH, Teague DC, eds.
Orthopaedic Knowledge Update: Trauma 4. Rosemont, IL: American Academy of Orthopaedic Surgeons;
2010:145-157.
Bellabarba C, Ricci WM, Bolhofner BR. Results of indirect reduction and plating of femoral shaft
nonunions after intramedullary nailing. J Orthop Trauma. 2001 May;15(4):254-63. PubMed PMID:
11371790.

© 2013 American Academy of Orthopaedic Surgeons 2013 Orthopaedic In-Training Examination


NON-EXCLUSIVE LICENSE FOR USE OF AAOS MATERIAL

The American Academy of Orthopaedic Surgeons ("AAOS”) grants this non-exclusive license
(License) to the participant (Participant) in the 2013 Orthopaedic In-Training Examination® to
use AAOS copyrighted material. The material (Material) is as follows: The 2013 Orthopaedic
In-Training Examination (OITE) which includes test questions, test figures (images, videos,
slides, etc), recommended readings, and preferred responses. The AAOS and Participant both
benefit from this License and agree as follows:

1. This License shall begin on the date signed below by the AAOS and shall be in effect for
the life of the edition.

2. The AAOS grants Participant a non-exclusive license to use the Material in connection with
orthopaedic surgery resident training. Participant’s use of the Material is limited to the uses
described in this paragraph and the Material cannot be used by Participant in any other manner.
The following are examples of appropriate use of the Material:
a. Display the Material on Participant’s computer screen and/or another equivalent
device
b. Make a single fixed copy for the sole purpose of Participant’s personal reference
c. Use the Material in resident research project(s), provided the Material used is solely
for non-commercial research purposes. The Materials shall not be used in any
commercial endeavors.
d. Use the Material in educational presentations for residents and faculty
e. Provide an appropriate number of handouts for a single teaching session with the
appropriate citation

3. When using the Material, Participant agrees that:


a. Permission is granted only for orthopaedic surgery resident training.
b. The Material used in educational sessions must contain an acknowledgement that the
Material is being used with permission of the AAOS. Please use the following
citation:
© 2013 American Academy of Orthopaedic Surgeons. Reprinted from the
2013 Orthopaedic In-Training Examination with permission.
c. At no time are Materials (test items or figures) permitted to be posted to the internet
d. At no time are Materials to be reproduced in multiple copies and distributed beyond
the uses permitted under Paragraph 2 herein above
e. At no time are Materials to be used in materials created for sale.
f. At no time are Materials to be modified and marketed as a new product.
g. At no time are Materials to be used in commercial or industry sponsored lectures.
h. At no time are Materials to be used in educational settings where payment is received
for instruction.
i. At no time are Materials to be posted to a “personal web page (eg, Facebook, twitter,
etc.) or facsimile”

AMERICAN ACADEMY OF ORTHOPAEDIC SURGEONS


2012/10
Updated 8/9/2013
NON-EXCLUSIVE LICENSE FOR USE OF AAOS MATERIAL

4. This License:
a. Is subject to the laws of the State of Illinois;
b. May not be modified except by express written consent of the AAOS;
c. Constitutes the complete understanding of the parties and supersedes all prior oral or
written communication;
d. May not be assigned by Participant except by prior written consent of the AAOS; and
e. Is not effective until signed below by both parties.

5. The failure of the AAOS at any time to require performance of any provision of this License
shall not affect the right to later enforce the same provision.

6. The fee for Participant to use the Material as described above is $0 (zero).

7. This Agreement may not be assigned to another party or residency program.

8. This Agreement shall be construed in accordance with and governed by the laws of the
State of Illinois. The Parties agree to the jurisdiction and venue of the Circuit Courts of
Cook County, Illinois, or the United States District Court of the Northern District of
Illinois, for resolving disputes arising under this Agreement.

9. If Participant detects any misuse of this AAOS material, which means another individual is
using the materials in a manner that does not adhere to the terms of this license, send the
information about the misuse to AAOS at 6300 North River Road, Rosemont, IL 60018,
Attention: Laura Hruska, Manager, Examinations or email them to exams@aaos.org.

Name: Date:

BY CHECKING THIS BOX, I AGREE TO THE TERMS LISTED ABOVE

By Date: 11/09/2013
Laura Hruska, M.Ed., Manager, Examinations
Fax: 847-823-8024 Email: hruska@aaos.org

AMERICAN ACADEMY OF ORTHOPAEDIC SURGEONS


2012/10
Updated 8/9/2013
2013 Orthopaedic In‐Training Examination Score Key by Content Domain
 
{Excluded Items from Scoring: 19, 38, 65, 92, 178, 182, 207, 212, 244, 245, 257} 
 
Question    PR    Question    PR    Question    PR    Question    PR    Question    PR 
9    4    37    1    17    3    26    5    205    5 
22    4    48    5    25    3    36    3    227    3 
24    3    58    1    34    1    41    3    248    2 
38    3    65    4    47    5    53    3    257    3 
52    4    75    3    60    5    66    5    265    3 
61    4    78    5    71    4    89    2    275    3 
72    2    88    5    79    3    101    5    2    3 
84    3    95    5    86    5    118    5    6    5 
100    2    103    5    109    1    131    4    10    2 
110    5    115    5    127    3    148    5    14    2 
116    4    133    5    147    5    166    1    32    5 
119    2    140    4    155    5    174    2    57    5 
126    1    213    5    170    3    194    3    63    5 
134    3    225    4    180    4    219    5    73    4 
142    2    230    3    210    4    232    3    82    4 
152    1    240    2    220    3    243    3    87    2 
154    4    256    4    233    3    246    5    90    4 
165    3    269    4    242    2    255    2    102    4 
169    1    5    4    258    4    266    5    107    2 
179    5    20    3    267    2    7    2    112    4 
185    1    31    1    3    1    19    2    117    3 
192    2    44    1    13    2    27    1    120    2 
195    3    50    2    21    5    33    3    123    3 
199    2    59    3    29    5    45    4    128    1 
203    2    76    5    35    5    55    2    132    2 
206    1    98    3    43    3    67    1    135    4 
218    4    105    1    49    5    93    5    137    2 
229    5    108    3    56    1    111    5    139    3 
244    3    121    5    64    5    124    5    144    5 
254    5    136    3    70    5    141    2    149    3 
270    5    153    4    74    4    162    5    156    5 
8    3    164    1    80    2    181    3    160    4 
16    1    173    3    91    1    190    3    163    2 
30    2    183    4    94    2    198    5    167    3 
46    2    189    4    97    3    222    3    172    4 
54    5    197    4    104    4    236    2    176    2 
68    5    214    1    114    1    247    1    186    3 
69    3    234    3    122    2    260    3    188    5 
81    1    241    3    129    5    271    3    196    2 
92    1    253    2    138    1    274    3    200    3 
106    2    259    1    143    3    11    1    201    1 
130    4    272    2    151    5    23    5    204    4 
146    5    18    3    161    1    39    3    207    5 
157    5    40    2    168    5    42    2    208    4 
175    1    77    4    171    1    51    3    211    5 
191    3    85    4    177    1    62    5    216    4 
217    3    96    5    182    4    83    3    221    1 
231    1    150    5    187    1    99    4    223    2 
245    1    159    1    212    2    113    2    226    3 
252    5    178    5    215    4    125    5    228    2 
264    5    209    3    224    4    145    1    235    2 
273    5    237    4    239    2    158    4    238    5 
1    1    250    4    251    4    184    4    249    5 
15    1    263    2    262    4    193    1    261    4 
28    1    4    4    12    5    202    2    268    5 

 
2013 Orthopaedic In‐Training Examination—Question Numbers by Content Domain
 
(*Excluded from Scoring: 19, 38, 65, 92, 178, 182, 207, 212, 244, 245, 257) 
 
 
Basic Science (31) 
9, 22, 24, *38, 52, 61, 72, 84, 100, 110, 116, 119, 126, 134, 142, 152, 154, 165, 169, 179, 185, 
192, 195, 199, 203, 206, 218, 229, *244, 254, 270 
 
Foot and Ankle (21) 
8, 16, 30, 46, 54, 68, 69, 81, *92, 106, 130, 146, 157, 175, 191, 217, 231, *245, 252, 264, 273 
 
Hand (21) 
1, 15, 28, 37, 48, 58, *65, 75, 78, 88, 95, 103, 115, 133, 140, 213, 225, 230, 240, 256, 269 
 
Hip & Knee Reconstruction (24) 
5, 20, 31, 44, 50, 59, 76, 98, 105, 108, 121, 136, 153, 164, 173, 183, 189, 197, 214, 234, 241, 
253, 259, 272 
 
System Based Practice (12) 
18, 40, 77, 85, 96, 150, 159, *178, 209, 237, 250, 263 
 
Oncology (21) 
4, 17, 25, 34, 47, 60, 71, 79, 86, 109, 127, 147, 155, 170, 180, 210, 220, 233, 242, 258, 267 
 
Pediatrics (34) 
3, 13, 21, 29, 35, 43, 49, 56, 64, 70, 74, 80, 91, 94, 97, 104, 114, 122, 129, 138, 143, 151, 161, 
168, 171, 177, *182, 187, *212, 215, 224, 239, 251, 262 
 
Shoulder and Elbow (20) 
12, 26, 36, 41, 53, 66, 89, 101, 118, 148, 166, 174, 194, 219, 232, 243, 246, 255, 266 
 
Spine (21) 
7, *19, 27, 33, 45, 55, 67, 93, 111, 124, 141, 162, 181, 190, 198, 222, 236, 247, 260, 271, 274 
 
Sports Medicine (21) 
11, 23, 39, 42, 51, 62, 83, 99, 113, 125, 145, 158, 184, 193, 202, 205, 227, 248, *257, 265, 275 
 
Trauma (52) 
2, 6, 10, 14, 32, 57, 63, 73, 82, 87, 90, 102, 107, 112, 117, 120, 123, 128, 132, 135, 137, 139, 
144, 149, 156, 160, 163, 167, 172, 176, 186, 188, 196, 200, 201, 204, *207, 208, 211, 216, 221, 
223, 226, 228, 235, 238, 249, 261, 268 
2013 Orthopaedic In‐Training Examination

EXAMINATION WORKSHEET

This worksheet is for your use only.


DO NOT return this page to AAOS.

Basic Science 199. 1 2 3 4 5 217. 1 2 3 4 5 230. 1 2 3 4 5

9. 1 2 3 4 5 203. 1 2 3 4 5 231. 1 2 3 4 5 240. 1 2 3 4 5

22. 1 2 3 4 5 206. 1 2 3 4 5 245. 1 2 3 4 5 256. 1 2 3 4 5

24. 1 2 3 4 5 218. 1 2 3 4 5 252. 1 2 3 4 5 269. 1 2 3 4 5

38. 1 2 3 4 5 229. 1 2 3 4 5 264. 1 2 3 4 5 Hip/Knee Reconst


52. 1 2 3 4 5 244. 1 2 3 4 5 273. 1 2 3 4 5 5. 1 2 3 4 5

61. 1 2 3 4 5 254. 1 2 3 4 5 Hand 20. 1 2 3 4 5

72. 1 2 3 4 5 270. 1 2 3 4 5 1. 1 2 3 4 5 31. 1 2 3 4 5

84. 1 2 3 4 5 Foot and Ankle 15. 1 2 3 4 5 44. 1 2 3 4 5

100. 1 2 3 4 5 8. 1 2 3 4 5 28. 1 2 3 4 5 50. 1 2 3 4 5

110. 1 2 3 4 5 16. 1 2 3 4 5 37. 1 2 3 4 5 59. 1 2 3 4 5

116. 1 2 3 4 5 30. 1 2 3 4 5 48. 1 2 3 4 5 76. 1 2 3 4 5

119. 1 2 3 4 5 46. 1 2 3 4 5 58. 1 2 3 4 5 98. 1 2 3 4 5

126. 1 2 3 4 5 54. 1 2 3 4 5 65. 1 2 3 4 5 105. 1 2 3 4 5

134. 1 2 3 4 5 68. 1 2 3 4 5 75. 1 2 3 4 5 108. 1 2 3 4 5

142. 1 2 3 4 5 69. 1 2 3 4 5 78. 1 2 3 4 5 121. 1 2 3 4 5

152. 1 2 3 4 5 81. 1 2 3 4 5 88. 1 2 3 4 5 136. 1 2 3 4 5

154. 1 2 3 4 5 92. 1 2 3 4 5 95. 1 2 3 4 5 153. 1 2 3 4 5

165. 1 2 3 4 5 106. 1 2 3 4 5 103. 1 2 3 4 5 164. 1 2 3 4 5

169. 1 2 3 4 5 130. 1 2 3 4 5 115. 1 2 3 4 5 173. 1 2 3 4 5

179. 1 2 3 4 5 146. 1 2 3 4 5 133. 1 2 3 4 5 183. 1 2 3 4 5

185. 1 2 3 4 5 157. 1 2 3 4 5 140. 1 2 3 4 5 189. 1 2 3 4 5

192. 1 2 3 4 5 175. 1 2 3 4 5 213. 1 2 3 4 5 197. 1 2 3 4 5

195. 1 2 3 4 5 191. 1 2 3 4 5 225. 1 2 3 4 5 214. 1 2 3 4 5

Page 1 of 3
2013 Orthopaedic In‐Training Examination

EXAMINATION WORKSHEET

This worksheet is for your use only.


DO NOT return this page to AAOS.

234. 1 2 3 4 5 60. 1 2 3 4 5 56. 1 2 3 4 5 239. 1 2 3 4 5

241. 1 2 3 4 5 71. 1 2 3 4 5 64. 1 2 3 4 5 251. 1 2 3 4 5

253. 1 2 3 4 5 79. 1 2 3 4 5 70. 1 2 3 4 5 262. 1 2 3 4 5

259. 1 2 3 4 5 86. 1 2 3 4 5 74. 1 2 3 4 5 Shoulder & Elbow


272. 1 2 3 4 5 109. 1 2 3 4 5 80. 1 2 3 4 5 12. 1 2 3 4 5

System Based Pract 127. 1 2 3 4 5 91. 1 2 3 4 5 26. 1 2 3 4 5

18. 1 2 3 4 5 147. 1 2 3 4 5 94. 1 2 3 4 5 36. 1 2 3 4 5

40. 1 2 3 4 5 155. 1 2 3 4 5 97. 1 2 3 4 5 41. 1 2 3 4 5

77. 1 2 3 4 5 170. 1 2 3 4 5 104. 1 2 3 4 5 53. 1 2 3 4 5

85. 1 2 3 4 5 180. 1 2 3 4 5 114. 1 2 3 4 5 66. 1 2 3 4 5

96. 1 2 3 4 5 210. 1 2 3 4 5 122. 1 2 3 4 5 89. 1 2 3 4 5

150. 1 2 3 4 5 220. 1 2 3 4 5 129. 1 2 3 4 5 101. 1 2 3 4 5

159. 1 2 3 4 5 233. 1 2 3 4 5 138. 1 2 3 4 5 118. 1 2 3 4 5

178. 1 2 3 4 5 242. 1 2 3 4 5 143. 1 2 3 4 5 131. 1 2 3 4 5

209. 1 2 3 4 5 258. 1 2 3 4 5 151. 1 2 3 4 5 148. 1 2 3 4 5

237. 1 2 3 4 5 267. 1 2 3 4 5 161. 1 2 3 4 5 166. 1 2 3 4 5

250. 1 2 3 4 5 Pediatrics 168. 1 2 3 4 5 174. 1 2 3 4 5

263. 1 2 3 4 5 3. 1 2 3 4 5 171. 1 2 3 4 5 194. 1 2 3 4 5

Oncology 13. 1 2 3 4 5 177. 1 2 3 4 5 219. 1 2 3 4 5

4. 1 2 3 4 5 21. 1 2 3 4 5 182. 1 2 3 4 5 232. 1 2 3 4 5

17. 1 2 3 4 5 29. 1 2 3 4 5 187. 1 2 3 4 5 243. 1 2 3 4 5

25. 1 2 3 4 5 35. 1 2 3 4 5 212. 1 2 3 4 5 246. 1 2 3 4 5

34. 1 2 3 4 5 43. 1 2 3 4 5 215. 1 2 3 4 5 255. 1 2 3 4 5

47. 1 2 3 4 5 49. 1 2 3 4 5 224. 1 2 3 4 5 266. 1 2 3 4 5

Page 2 of 3
2013 Orthopaedic In‐Training Examination

EXAMINATION WORKSHEET

This worksheet is for your use only.


DO NOT return this page to AAOS.

Spine 23. 1 2 3 4 5 14. 1 2 3 4 5 167. 1 2 3 4 5

7. 1 2 3 4 5 39. 1 2 3 4 5 32. 1 2 3 4 5 172. 1 2 3 4 5

19. 1 2 3 4 5 42. 1 2 3 4 5 57. 1 2 3 4 5 176. 1 2 3 4 5

27. 1 2 3 4 5 51. 1 2 3 4 5 63. 1 2 3 4 5 186. 1 2 3 4 5

33. 1 2 3 4 5 62. 1 2 3 4 5 73. 1 2 3 4 5 188. 1 2 3 4 5

45. 1 2 3 4 5 83. 1 2 3 4 5 82. 1 2 3 4 5 196. 1 2 3 4 5

55. 1 2 3 4 5 99. 1 2 3 4 5 87. 1 2 3 4 5 200. 1 2 3 4 5

67. 1 2 3 4 5 113. 1 2 3 4 5 90. 1 2 3 4 5 201. 1 2 3 4 5

93. 1 2 3 4 5 125. 1 2 3 4 5 102. 1 2 3 4 5 204. 1 2 3 4 5

111. 1 2 3 4 5 145. 1 2 3 4 5 107. 1 2 3 4 5 207. 1 2 3 4 5

124. 1 2 3 4 5 158. 1 2 3 4 5 112. 1 2 3 4 5 208. 1 2 3 4 5

141. 1 2 3 4 5 184. 1 2 3 4 5 117. 1 2 3 4 5 211. 1 2 3 4 5

162. 1 2 3 4 5 193. 1 2 3 4 5 120. 1 2 3 4 5 216. 1 2 3 4 5

181. 1 2 3 4 5 202. 1 2 3 4 5 123. 1 2 3 4 5 221. 1 2 3 4 5

190. 1 2 3 4 5 205. 1 2 3 4 5 128. 1 2 3 4 5 223. 1 2 3 4 5

198. 1 2 3 4 5 227. 1 2 3 4 5 132. 1 2 3 4 5 226. 1 2 3 4 5

222. 1 2 3 4 5 248. 1 2 3 4 5 135. 1 2 3 4 5 228. 1 2 3 4 5

236. 1 2 3 4 5 257. 1 2 3 4 5 137. 1 2 3 4 5 235. 1 2 3 4 5

247. 1 2 3 4 5 265. 1 2 3 4 5 139. 1 2 3 4 5 238. 1 2 3 4 5

260. 1 2 3 4 5 275. 1 2 3 4 5 144. 1 2 3 4 5 249. 1 2 3 4 5

271. 1 2 3 4 5 Trauma 149. 1 2 3 4 5 261. 1 2 3 4 5

274. 1 2 3 4 5 2. 1 2 3 4 5 156. 1 2 3 4 5 268. 1 2 3 4 5

Sports Medicine 6. 1 2 3 4 5 160. 1 2 3 4 5

11. 1 2 3 4 5 10. 1 2 3 4 5 163. 1 2 3 4 5

Page 3 of 3

You might also like